Sunteți pe pagina 1din 216

• Unit and Dimension

• Vector and Calculus

• Kinematics
Copyright © reserved with Motion Edu. Pvt. Ltd. and Publications

All rights reserved. No part of this work herein should be reproduced or used either
graphically, electronically, mechanically or by recording, photocopying, taping, web
distributing or by storing in any form and retrieving without the prior written permission of
the publisher. Anybody violating this is liable to be legally prosecuted.

Corporate Head Office


394 - Rajeev Gandhi Nagar Kota, (Raj.)
Ph. No. : 08003899588, 0744-2209671
IVRS No : 0744-2439051, 52, 53,
www. motioniitjee.com , info@motioniitjee.com
CONTENTS

UNIT AND DIMENSION


THEORY AND EXERCISE BOOKLET
S.NO. TOPIC ......................... PAGE NO.
 THEORY WITH SOLVED EXAMPLES ................................................... 05 – 25
 EXERCISE - 1 (JEE MAIN) ............................................... 26 – 30
 EXERCISE - 2 (JEE ADVANCED-OBJECTIVE) ........................ 31 – 40
 EXERCISE - 3 (PRIVIOUS YEAR QUESTIONS) ................................ 41 – 44
 ANSWER KEY ....................................................................................... 45 – 45

VECTOR AND CALCULUS


THEORY AND EXERCISE BOOKLET
S.NO. TOPIC .............. PAGE NO.
 THEORY WITH SOLVED EXAMPLES ................................................... 46 – 81
 EXERCISE - 1 (JEE MAIN) ............................................... 82 – 93
 EXERCISE - 2 (JEE ADVANCED-OBJECTIVE) ........................ 94 – 104
 EXERCISE - 3 (JEE ADVANCED) ........................................ 105 – 117
 ANSWER KEY ....................................................................................... 118 – 121

KINEMATICS
THEORY AND EXERCISE BOOKLET
S.NO. TOPIC .............. PAGE NO.

 THEORY WITH SOLVED EXAMPLES ................................................... 122 – 168


 EXERCISE - 1 (JEE MAIN) ............................................... 172 – 179
 EXERCISE - 2 (JEE ADVANCED-OBJECTIVE) ........................ 180 – 195
 EXERCISE - 3 (JEE ADVANCED) ........................................ 196 – 203
 EXERCISE - 4 (PRIVIOUS YEAR QUESTIONS) ................................ 204 – 212
 ANSWER KEY ....................................................................................... 213 – 216
Page # 4 UNITS AND DIMENSIONS

Syllabus
Unit Dimension and Basic Mathematics
Physics, SI units, Fundamental and derived units. Dimensions of
Physical qunatities, dimensional analysis and its applications.
Kinematics
Kinematics in one and two dimensions (Cartesian coordinates only),
Projectiles; Relative velocity.

Corporate Head Office : Motion Education Pvt. Ltd., 394 - Rajeev Gandhi Nagar, Kota-5 (Raj.)
UNITS AND DIMENSIONS Page # 5

UNITS AND DIMENSIONS


1. PHYSICAL QUANTITY
The quantites which can be measured by an instrument and by means of which we can describe the
laws of physics are called physical quantities.
Types of physical quantities :

Fundamental Derived Supplementry

1.1 Fundamental
Although the number of physical quantities that we measure is very large, we need only a limited
number of units for expressing all the physical quantities since they are interrelated with one another.
So, certain physical quantities have been chosen arbitrarily and their units are used for expressing all
the physical quantities, such quantities are known as Fundamental, Absolute or Base Quantities
(such as length, time and mass in mechanics)
(i) All other quantites may be expressed in terms of fundamental quantities.
(ii) They are independent of each other and cannot be obtained from one another.

An international body named General Conference on Weights and Measures chose seven physical
quantities as fundamental :
(1) length (2) mass (3) time (4) electric current,
(5) thermodynamic temperature (6) amount of substance
(7) luminous intensity.
Note : These are also called as absolute or base quantities.
In mechanics, we treat length, mass and time as the three basic or fundamental quantities.

1.2 Derived : Physical quantities which can be expressed as combination of base quantities are called as
derived quantities.

For example : Speed, velocity, acceleration, force, momentum, pressure, energy etc.

dis tance length


Ex.1 Speed = =
time time
1.3 Supplementary : Beside the seven fundamental physical quantities two supplementary quantities are
also defined, they are :
(1) Plane angle (2) Solid angle.

Note : The supplementary quantities have only units but no dimensions.

2. MAGNITUDE :
Magnitude of physical quantity = (numerical value) × (unit)
Magnitude of a physical quantity is always constant. It is independent of the type of unit.

1
 numerical value 
unit
or n1u1 = n2u2 = constant
Ex.2 Length of a metal rod bar is unchanged whether it is measured as 2 metre or 200 cm.
Observe the change in the Numerical value (from 2 to 200) as unit is changed from metre to
cm.

: 0744-2209671, 08003899588 | url : www.motioniitjee.com, : info@motioniitjee.com


Page # 6 UNITS AND DIMENSIONS

3. UNIT :
Measurement of any physical quantity is expressed in terms of an internationally accepted certain
basic reference standard called unit.
The units for the fundamental or base quantities are called fundamental or base unit. Other physical
quantities are expressed as combination of these base units and hence, called derived units.
A complete set of units, both fundamental and derived is called a system of unit.

3.1. Principle systems of Unit


There are various system in use over the world : CGS, FPS, SI (MKS) etc
Table 1 : Units of some physical quantities in different systems.

Syste m
Physica l
Qua ntity CGS MKS FPS
(Ga ussia n) (SI) (British)
Length centimeter meter foot

Funda me nta l Mass gram kilogram pound

Time second second second

Force dyne newton  N poundal

De rive d W ork or Energy erg joule  J ft-poundal

Power erg/s watt  W ft-poundal/s

3.2 Supplementary units :


(1) Plane angle : radian (rad)
(2) Solid angle : steradian (sr)
* The SI system is at present widely used throughout the world. In IIT JEE only SI system is followed.

3.3 Definitions of some important SI Units


(i) Metre : 1 m = 1,650, 763.73 wavelengths in vaccum, of radiation corresponding to organ-red light of
krypton-86.
(ii) Second : 1 s = 9,192, 631,770 time periods of a particular from Ceasium - 133 atom.
(iii) Kilogram : 1kg = mass of 1 litre volume of water at 4°C
(iv) Ampere : It is the current which when flows through two infinitely long straight conductors of
negligible cross-section placed at a distance of one metre in vacuum produces a force of 2 × 10–7 N/m
between them.
(v) Kelvin : 1 K = 1/273.16 part of the thermodynamic temperature of triple point of water.
(vi) Mole : It is the amount of substance of a system which contains as many elementary particles
(atoms, molecules, ions etc.) as there are atoms in 12g of carbon - 12.

 1  2
(vii) Candela : It is luminous intensity in a perpendicular direction of a surface of  m of a
 600000 
black body at the temperature of freezing point under a pressure of 1.013 × 105 N/m2.
(viii) Radian : It is the plane angle between two radiia of a circle which cut-off on the circumference,
an arc equal in length to the radius.
(ix) Steradian : The steradian is the solid angle which having its vertex at the centre of the sphere,
cut-off an area of the surface of sphere equal to that of a square with sides of length equal to the
radius of the sphere.

Corporate Head Office : Motion Education Pvt. Ltd., 394 - Rajeev Gandhi Nagar, Kota-5 (Raj.)
UNITS AND DIMENSIONS Page # 7

Ex.3 Find the SI unit of speed, acceleration


dis tan ce meter(m)
Sol. speed = = = m/s (called as meter per second)
time sec ond(s)
velocity displacement / time
acceleration = =
time time
displacement meter
= = = m/s2 (called as meter per second square)
( time) 2 sec ond2

4. S I PREFIXES
The magnitudes of physical quantities vary order a wide range. The CGPM recommended standard
prefixes for magnitude too large or too small to be expressed more compactly for certain power of 10.

Power of 10 Prefix Symbol Power of 10 Prefix Symbol


18 –1
10 exa E 10 deci d
15 –2
10 peta P 10 centi c
12 –3
10 tera T 10 milli m
109 giga G 10–6 micro 
106 mega M 10–9 nano n
3 –12
10 kilo k 10 pico p
2 –15
10 hecto h 10 femto f
101 deca da 10–18 atto a

5. GENERAL GUIDELINES FOR USING SYMBOLS FOR SI UNITS, SOME OTHER UNITS, SOME
OTHER UNITS, AND SI PREFIXES
(a) Symbols for units of physical quantities are printed/written in Roman (upright type), and not in
italics
For example : 1 N is correct but 1 N is incorrect
(b) (i) Unit is never written with capital initial letter even if it is named after a scientist.
For example : SI unit of force is newton (correct) Newton (incorrect)
(ii) For a unit named after a scientist, the symbol is a capital letter.
But for other units, the symbol is NOT a capital letter.
For example :
force  newton (N)
energy  joule (J)
electric current  ampere (A)
temperature  kelvin (K)
frequency  hertz (Hz)
For example :
length  meter (m)
mass  kilogram (kg)
luminous intensity  candela (cd)
time  second (s)
Note : The single exception is L, for the unit litre.
(c) Symbols for units do not contain any final full stop all the end of recommended letter and remain
unaltered in the plural, using only singular form of the unit.

: 0744-2209671, 08003899588 | url : www.motioniitjee.com, : info@motioniitjee.com


Page # 8 UNITS AND DIMENSIONS

For example :

Quantity Correct Incorrect


25 c m
25 centimeters 25 cm
25 cms.

(d) Use of solidus ( / ) is recommended only for indicating a division of one letter unit symbol by another
unit symbol. Not more than one solidus is used.

For example :

Correct Incorrect
2
m/s m/s/s
2
N s/m N s / m/ m
J/K mol J / K / mol
kg/m s kg / m / s

(e) Prefix symbols are printed in roman (upright) type without spacing between the prefix symbol and the
unit symbol. Thus certain approved prefixes written very close to the unit symbol are used to indicate
decimal fractions or multiples of a SI unit, when it is inconveniently small or large.

For example

megawatt 1 MW = 10 6 W
centim etre 1 cm = 10–2 m
kilom etre 1 km = 103 m
millivolt 1 mV = 10 –3 V
kilowatt-hour 1 kW h = 10 3 W h = 3.6 M J = 3.6 × 10 6 J
microampere 1 A  1 0
–6
A
– 10
angstrom 1 Å  0 .1n m   1 0 m

nanosecond 1 ns = 10 –9 s
picofarad 1 pF = 10 –12 F
microsecond 1 s  1 0 – 6 s

gigahertz 1 GHz = 109 Hz


micron 1 m  10 –6 m

The unit 'fermi', equal to a femtometre or 10–15 m has been used as the convenient length unit in
nuclear studies.

(f) When a prefix is placed before the symbol of a unit, the combination of prefix and symbol is considered
as a new symbol, for the unit, which can be raised to a positive or negative power without using
brackets. These can be combined with other unit symbols to form compound unit.

Corporate Head Office : Motion Education Pvt. Ltd., 394 - Rajeev Gandhi Nagar, Kota-5 (Raj.)
UNITS AND DIMENSIONS Page # 9

For example :

Quantity Correct Incorrect

cm3 (cm)3 = (0.01 m)3 = (10–2m)3 = 10–6 m3 0.01 m3 or 10–2 m3

mA2 (mA)2 = (0.001 A)2 = (10–3A)2 = 10–6 A2 0.001 A2

(g) A prefix is never used alone. It is always attached to a unit symbol and written or fixed before the unit
symbol.

For example :
103/m3 = 1000/m3 or 1000 m–3, but not k/m3 or k m–3.

(h) Prefix symbol is written very close to the unit symbol without spacing between them, while unit
symbols are written separately with spacing with units are multiplied together.

For example :

Quantity Correct Incorrect


–1
1 ms 1 metre per second 1 milli per second
1 ms 1 millisecond 1 metre second
1Cm 1 coulomb metre 1 centimetre
1 cm 1 centimetre 1 coulomb metre
(i) The use of double profixes is avoided when single prefixes are available.
For example :

Quantity Correct Incorrect


10–9 m 1 nm (nanometre) 1 m m (milli micrometre)
10–6 m 1m (micron) 1 m m m (milli millimetre)
10–12 F 1 pF (picofarad) 1   F (micro microfarad)
109 F 1 GW (giga watt) 1 kM W (kilo megawatt)

(j) The use of a combination of unit and the symbols for unit is avoided when the physical quantity is
expressed by combining two or more units.

Quantity Correct Incorrect

J/mol K Joule / mole K


joule per mole Kelvin or J mol–1 K–1 or J/mol Kelvin
or J/mole K
newton m second
newton metre second Nms or N m second
or N metre s
or newton metre s

: 0744-2209671, 08003899588 | url : www.motioniitjee.com, : info@motioniitjee.com


Page # 10 UNITS AND DIMENSIONS

5.1. Characteristics of base units or standards :


(A) Well defined (B) Accessibility (C) Invariability (D) Convenience in use

5.2 Some special types of units :


1. 1 Micron (1) = 10–4 cm = 10–6 m (length)
2. 1 Angstrom (1 Å) = 10–8 cm = 10–10m (length)
3. 1 fermi (1f) = 10 cm
–13
= 10–15 m (length)
4. 1 inch = 2.54 cm (length)
5. 1 mile = 5280 feet = 1.609 km (length)
6. 1 atmosphere = 105 N/m2 = 76 torr = 76 mm of Hg pressure (pressure)
7. 1 litre = 10–3 m3 = 1000 cm3 (volume)
8. 1 carat = 0.0002 kg (weight)
9. 1 pound (Ib) = 0.4536 kg (weight)

6. DIMENSIONS
Dimensions of a physical quantity are the power to which the fundamental quantities must be raised to
represent the given physical quantity.

mass mass
For example, density = =
volume ( length )3
or density = (mass) (length)–3 ...(i)
Thus, the dimensions of density are 1 in mass and –3 in length. The dimensions of all other fundamental
quantities are zero.
For convenience, the fundamental quantities are represented by one letter symbols. Generally mass is
denoted by M, length by L, time by T and electric current by A.

The thermodynamic temperature, the amount of substance and the luminous intensity are denoted by
the symbols of their units K, mol and cd respectively. The physical quantity that is expressed in terms
of the base quantities is enclosed in square brackets.
[sin] = [cos] = [tan] = [ex] = [M0L0T0]

7. DIMENSIONAL FORMULA
It is an expression which shows how and which of the fundalmental units are required to represent the
unit of physical quantity.
Different quantities with units. symbol and dimensional formula.

Quantity Symbol Formula S.I. Unit D.F.


Displacement s  Metre or m M0LT0
Area A ×b (Metre)2 or m2 M0L2T0
Volume V ×b×h (Metre)3 or m3 M0L3T0
s
Velocity v v m/s M0LT–1
t
Momentum p p = mv kgm/s MLT–1
v
Acceleration a a m/s2 M0LT–2
t
Force F F = ma Newton or N MLT–2
Impulse - F× t N.sec MLT–1
Work W F.d N.m ML2T–2

Corporate Head Office : Motion Education Pvt. Ltd., 394 - Rajeev Gandhi Nagar, Kota-5 (Raj.)
UNITS AND DIMENSIONS Page # 11

1
Energy KE or U K.E.  mv 2 Joule or J ML2T–2
2
P.E. = mgh
W
Power P P watt or W ML2T–3
t
Density d d = mass/volume kg/m3
ML–3T0
Pressure P P = F/A Pascal or Pa ML–1T–2
Torque  =r×F N.m. ML2T–2
arc
Angular displacement   radian or rad M0L0T0
radius


Angular velocity   rad/sec
t
MLT
0 0 –1


Angular acceleration   rad/sec2 M0L0T–2
t
Moment of Inertia I I = mr2 kg-m2 ML2T0
1
Frequency v or f f hertz or Hz
T
M0L0T–1
Stress - F/A N/m2 ML–1T–2
 A V
Strain - ; ; - M0L0T0
 A V

F/ A
Youngs modulus Y Y N/m2
 / 
ML–1T–2
(Bulk modulus of rigidity)

F W N J
Surface tension T or ; ML0T–2
 A m m2
Force constant (spring) k F = kx N/m ML0T–2

 dv 
Coefficient of viscosity  F =  A kg/ms(poise in C.G.S.) ML–1T–1
 dx 

Gm1 m 2 N  m2
Gravitation constant G F M–1L3T–2
r2 kg 2

PE J
Gravitational potential Vg Vg 
m kg
M0L2T–2
Temperature  - Kelvin or K
M0L0T0+1
Heat Q Q = m × S × t Joule or Calorie
ML2T–2

: 0744-2209671, 08003899588 | url : www.motioniitjee.com, : info@motioniitjee.com


Page # 12 UNITS AND DIMENSIONS

Joule
Specific heat S Q = m × S × t kg .Kelvin
M 0L 2T –
2 –1

Joule
Latent heat L Q = mL kg
M0L2T–2

KA (1   2 )t Joule
Coefficient of thermal K Q MLT –3 –
d m sec K
1

conductivity

Joule
Universal gas constant R PV = nRT
mol.K
ML2T–2–1
Mechanical equivalent J W = JH -
M0L0T0
of heat

Q
Charge Q or q I Coulomb or C M0L0TA
t
Current I - Ampere or A
M0L0T0A

1 q1q2 (coul.)2 C2
Electric permittivity 0 0  . or M – 1
L –
4F r 2 N.m2 N  m2
3
T4A 2

W
Electric potential V V Joule/coul
q
ML2T–3A–1

F
Intensity of electric field E E N/coul. MLT–3A–
q
1

Capacitance C Q = CV Farad M – 1
L –

2 4 2
TA


Dielectric constant r r  - M0L0T0
0
or relative permittivity
Resistance R V = IR Ohm
ML2T–3A–2

1
Conductance S S Mho M – 1
L –
R
2
T–3A2

RA
Specific resistance   Ohm × meter

ML3T–3A–2
or resistivity

Corporate Head Office : Motion Education Pvt. Ltd., 394 - Rajeev Gandhi Nagar, Kota-5 (Raj.)
UNITS AND DIMENSIONS Page # 13

1
Conductivity or s  Mho/meter

specific conductance
Magnetic induction B F = qvBsin Tesla or weber/m2 M
MT–2L AT
–1 A
–1 –3 3 2

or F = BIL

d
Magnetic flux  e Weber ML2T–2A–1
dt
Magnetic intensity H B = H A/m M0L–1T0A

 0 Idl sin  N
Magnetic permeability 0 B MLT–2A–2
4 r 2 amp 2
of free space or medium

dI
Coefficient of self or L e  L. Henery ML2T–2A–2
dt
Mutual inductance
Electric dipole moment p p = q × 2 C.m. M0LTA
Magnetic dipole moment M M = NIA amp.m2 M0L2AT0

8. USE OF DIMENSIONS

Theory of dimensions have following main uses :


8.1 Conversion of units :
This is based on the fact that the product of the numerical value (n) and its corresponding unit (u) is a
constant, i.e.,
n[u] = constant
or n1[u1] = n2 [u2]
Suppose the dimensions of a physical quantity are a in mass, b in length and c in time. If the fundamental
units in one system are M1, L1 and T1 and in the other system are M2, L2 and T2 respectively. Then we
can write.
n1[M1a Lb1 T1c ]  n2 [Ma2 Lb2 T2c ] ...(i)
Here n1 and n2 are the numerical values in two system of units respectively. Using Eq. (i), we can
convert the numerical value of a physical quantity from one system of units into the other system.

Ex.4 The value of gravitation constant is G = 6.67 × 10–11 Nm2/kg2 in SI units. Convert it into CGS
system of units.
Sol. The dimensional formula of G is [M–1 L3 T–2].
Using equation number (i), i.e.,
n1[M1–1 L31 T1–2 ]  n2 [M2–1 L32 T2–2 ]

–1 3 –2
M  L   T 
n 2  n1  1   1   1 
 M2   L 2   T2 
Here, n1 = 6.67 × 10–11
M1 = 1 kg, M2 = 1 g = 10–3 kg L1 = 1 m, L2 = 1cm = 10–2 m, T1 = T2 = 1s
Substituting in the above equation, we get

: 0744-2209671, 08003899588 | url : www.motioniitjee.com, : info@motioniitjee.com


Page # 14 UNITS AND DIMENSIONS

–1 3 –2
 1kg   1m   1s 
n2 = 6.67 × 10 –11  –3   –2   
10 kg  10 m   1s 
or n2 = 6.67 × 10–8
Thus, value of G in CGS system of units is 6.67 × 10–8 dyne cm2/g2.
8.2 To check the dimensional correctness of a given physical equation :
Every physical equation should be dimensionally balanced. This is called the 'Principle of Homogeneity'.
The dimensions of each term on both sides of an equation must be the same. On this basis we can
judge whether a given equation is correct or not. But a dimensionally correct equation may or may not
be physically correct.
Ex.5 Show that the expression of the time period T of a simple pendulum of length l given by T =
l
2 is dimensionally correct.
g

l
Sol. T  2
g

[L]
Dimensionally [T ]   [T]
[LT – 2 ]

As in the above equation, the dimensions of both sides are same. The given formula is dimensionally
correct.
8.3 Principle of Homogeneity of Dimensions.
This principle states that the dimensions of all the terms in a physical expression should be same. For
1 2 1
example, in the physical expression s = ut + at , the dimensions of s, ut and at2 all are same.
2 2
Note : The physical quantities separated by the symbols +, –, =, >, < etc., have the same dimensions.

c
Ex.6 The velocity v of a particle depends upon the time t according to the equation v = a + bt + .
d t
Write the dimensions of a, b, c and d.
Sol. From principle of homogeneity
[a] = [v]
or [a] = [LT–1] Ans.
[bt] = [v]

[ v ] [LT –1 ]
or [b ]  
[t ] [T]
or [b] = [LT–2]
Similarly, [d] = [t] = [T] Ans.

[c ]
Further,  [v ]
[d  t]
or [c] = [v] [d + t]
or [c] = [LT–1] [T]
or [c] = [L] Ans.

Corporate Head Office : Motion Education Pvt. Ltd., 394 - Rajeev Gandhi Nagar, Kota-5 (Raj.)
UNITS AND DIMENSIONS Page # 15

8.4 To establish the relation among various physical quantities :


If we know the factors on which a given physical quantity may depend, we can find a formula relating
the quantity with those factors. Let us take an example.
Ex.7 The frequency (f) of a stretched string depends upon the tension F (dimensions of force), length
l of the string and the mass per unit length  of string. Derive the formula for frequency.
Sol. Suppose, that the frequency f depends on the tension raised to the power a, length raised to the power
b and mass per unit length raised to the power c. Then.

f  [F]a [l ]b []c

or f  k [F]a [l ]b []c
Here, k is a dimensionless constant. Thus,
[f ] = [F]0 [l]b []c
or [M0 L0 T–1] = [MLT–2]a [L]b [ML–1]c
or [M0L0T–1] = [Ma + c La + b – c T–2a]
For dimensional balance, the dimension on both sides should be same.
Thus, a+c=0 ...(ii)
a+b–c=0 ...(iii)
and – 2a = – 1 ...(iv)
Solving these three equations, we get

: 0744-2209671, 08003899588 | url : www.motioniitjee.com, : info@motioniitjee.com


Page # 16 UNITS AND DIMENSIONS

1 1
a , c– and b = – 1
2 2
Substituting these values in Eq. (i), we get

f  k(F)1/ 2 (l ) –1() –1/ 2

k F
or f 
l 
1
Experimentally, the value of k is found to be
2
1 F
Hence, f 
2l 

8.5 Limitations of Dimensional Analysis


The method of dimensions has the following limitations :
(i) By this method the value of dimensionless constant can not be calculated.
(ii) By this method the equation containing trigonometrical, exponential and logarithmic terms cannot
be analysed.
(iii) If a physical quantity depends on more than three factors, then relation among them cannot be
established because we can have only three equations by equalising the powers of M, L and T.

BASIC MATHEMATICS
9. MENSURATION FORMULAS :

r : radius ; d = diameter ;
V = Volume S.A = surface area
(a) Circle
1 2
Perameter : 2r = d, Area : r2 = d
4
(b) Sphere
4 3 1
Surface area = 4r2 = d2 , Volume = r = d3
3 6
(c) Spherical Shell (Hollow sphere)
Surface area = 4r2 = d2
Volume of material used = (4r2)(dr), dr = thickness
(d) Cylinder
Lateral area = 2rh
V = r2h
Total area = 2rh + 2r2 = 2r (h + r)
(e) Cone
Lateral area = r r 2  h2 h = height

 2 2  1 2
Total area = r  r  h  r  V = r h
  3
(f) Ellipse

a2  b2
Circumference  2
2
b
a

Corporate Head Office : Motion Education Pvt. Ltd., 394 - Rajeev Gandhi Nagar, Kota-5 (Raj.)
UNITS AND DIMENSIONS Page # 17

area = ab
a = semi major axis
b = semi minor axis

(g) Parallelogram a
h
A = bh = ab sin  
a = side ; h = height ; b = base b
 = angle between sides a and b b

(h) Trapezoid
h
h
area = (a  b)
2 a
a and b parallel sides
(i) Triangle h = height

bh ab
area = = sin  = s(s  a)(s  b)(s  c ) 
2 2 c
b
a, b, c sides are opposite to angles , , 
b = base ; h = height  
a
1
s= (a  b  c )
2

(j) Rectangular container h


lateral area = 2(b  bh  h )
V =  bh b

side , b, h
Mathematics is the language of physics. It becomes easier to describe, understand and apply the
physical principles, if one has a good knowledge of mathematics.

10. LOGARITHMS :

(i) e  2.7183 (ii) If ex = y, then x = loge y = ln y (iii) If 10x = y, then x = log10y


(iv) log10y = 0.4343 loge y = 2.303 log10 y (v) log (ab) = log (a) + log (b)
a
(vi) log  = log (a) – log (b) (vii) log an = n log (a)
b
 

11. TRIGONOMETRIC PROPERTIES :

(i) Measurement of angle & relationship between degrees & radian


In navigation and astronomy, angles are measured in degrees, but in calculus it is best to use units
called radians because of they simplify later calculations.
Let ACB be a central angle in circle of radius r, as in figure.
Then the angle ACB or  is defined in radius as -

: 0744-2209671, 08003899588 | url : www.motioniitjee.com, : info@motioniitjee.com


B
Page # 18 UNITS AND DIMENSIONS

A
Arc length AB r
  = C
Radius r
If r = 1 then  = AB
The radian measure for a circle of unit radius of angle ABC is defined to be the length of the circular
arc AB. since the circumference of the cirlce is 2 and one complete revolution of a cicle is 360°, the
relation between radians and degrees is given by the following equation.
 radians = 180°

ANGLE CONVERSION FORMULAS

 
1 degree = ( 0.02) radian Degrees to radians : multiply by
180 180
180
1 radian  57 degrees Radians to degrees : multiply by

 
Ex.15 Covert 45° to radians : 45 • = rad
 4

  180
Convert rad to degrees : • = 30°
6 6 
Ex.16 Convert 30º to radians :

 
Sol. 30 º = rad
180º 6


Ex.17 Convert rad to degrees.
3

 180
Sol.  = 60
3 
Standard values
  
(1) 30° = rad (2) 45° = rad (3) 60° = rad
6 4 3

 2 3
(4) 90° = rad (5) 120° = rad (6) 135° = rad
2 3 4

5
(7) 150° = rad (8) 180° =  rad (9) 360° = 2 rad
6
(Check these values yourself to see that the satisfy the conversion formulaes)

(ii) Measurement of positive & Negative Angles :


y

x
Positive Negative
measure Measure
x

Corporate Head Office : Motion Education Pvt. Ltd., 394 - Rajeev Gandhi Nagar, Kota-5 (Raj.)
UNITS AND DIMENSIONS Page # 19

An angle in the xy-plane is said to be in standard position if its vertex lies at the origin and its initial ray
lies along the positive x-axis (Fig). Angles measured counterclockwise from the positive x-axis are
assigned positive measures; angles measured clockwise are assigned negative measures.

y y y y
5

2
x x x x
3 3
9 –
4
4

(iii) Six Basic Trigonometric Functions :


y

P(x,y)
se
nu

r
te

oppsite
po

y
hy

side

 x
O adjacent side
P(x,y)

The trigonometric fucntion of a general angle  are defined in tems of x, y and r.

opp y hyp r
Sine : sin    Cosecant : cos ec  
hyp r opp y

adj x hyp r
Cosine: cos    Secant : sec   
hyp r adj x

opp y adj x
Tangent: tan    Cotangent: cot   
adj x opp y

VALUES OF TRIGONOMETRIC FUNCTIONS

If the circle in (Fig. above) has radius r = 1, the equations defining sin  and cos  become
cos = x, sin = y
We can then calculate the values of the cosine and sine directly from the coordinates of P.

Ex.18 Find the six trigonometric ratios from given fig. (see above)
5
opp 4 adj 3 4
Sol. sin = hyp  5 cos = hyp  5

opp 4 adj 3
3
tan = adj  3 cot  = 
opp 4

hyp 5 hyp 5
sec  =  cosec  = 
opp 3 opp 4

: 0744-2209671, 08003899588 | url : www.motioniitjee.com, : info@motioniitjee.com


Page # 20 UNITS AND DIMENSIONS

Ex.19 Find the sine and cosine of angle  shown in the unit circle if coordinate of point p are as shown.
 
y
 – 1, 3 
 2 2 
 

3
1 
2

Sol. 1 x
2

1 3
cos  = x-coordinate of P = – sin  = y-coordinate of P =
2 2

12. Values of sin , cos  and tan  for some standard angles.

Degree 0 30 37 45 53 60 90 120 135 180

Radians 0 /6 37 / 180 / 4 53 / 180 / 3 /2 2 / 3 3 / 4 

sin  0 1/2 3/5 1/ 2 4/5 3/2 1 3 /2 1/ 2 0

cos  1 3 /2 4/5 1/ 2 3/5 1/2 0 –1/2 – 1/ 2 –1

tan  0 1/ 3 3/4 1 4/3 3  – 3 –1 0

A useful rule for remembering when the basic trigonometric funcions are positive and negative is the
CAST rule. If you are not very enthusiastic about CAST. You can remember it as ASTC (After school to
college)
y
nd nd
II Quadrant I Quadrant

S A
sin positive all positive

T C
Tan positive cos positive
nd nd
III Quadrant IV Quadrant

The CAST rule


RULES FOR FINDING TRIGONOMETRIC RATIO OF ANGLES GREATER THAN 90°.
Step 1  Identify the quadrant in which angle lies.
Step 2  (a) If angle = (n ± ) where n is an integer. Then

  
(b) If angle = ( 2n  1)   where n is in interger. Then
 2 

  
trigonometric function of ( 2n  1)   = complimentry trignometric function of  and
 2 
sign will be decided by CAST Rule.
Ex.20 Evaluate sin 120°

3
Sol. sin 120° = sin (90° + 30°) = cos 30° =
2

Corporate Head Office : Motion Education Pvt. Ltd., 394 - Rajeev Gandhi Nagar, Kota-5 (Raj.)
UNITS AND DIMENSIONS Page # 21

3
Aliter sin 120° = sin (180° – 60°) = sin 60° =
2
Ex.21 Evaluate cos 210°
3
Sol. cos 210° = cos (180° + 30°) = – cos 30° = –
2
1
Ex.22 tan 210° = tan (180° + 30°) = tan 30° = 
3

13. IMPORTANT FORMULAS

(i) sin2 + cos2 = 1 (ii) 1 + tan2 = sec2


(iii) 1 + cot2 = cosec2 (iv) sin2 = 2 sin  cos 
(v) cos 2 = 2 cos2 – 1 = 1 – 2 sin2 = cos2 – sin2
(vi) sin (A ± B) = sin A cos B ± cos A sin B (vii) cos (A ± B) = cos A cos B  sin A sin B

CD C –D C–D C D


(viii) sin C + sin D = 2 sin  cos  (ix) sinC – sin D = 2 sin  cos 
 2   2   2   2 

CD C–D D–C CD


(x) cos C + cos D = 2 cos cos (xi) cos C – cos D = 2 sin sin
2 2 2 2
2 tan  tan A  tan B
(xii) tan 2  2 (xiii) tan(A ± B) = 1  tan A tan B
1 – tan 
(xiv) sin(90° + ) = cos  (xv) cos (90° + ) = – sin 
(xvi) tan (90° + ) = – cot  (xvii) sin(90° – ) = cos 
(xviii) cos(90° – ) = sin (xix) cos (180° – ) = – cos 
(xx) sin(180° – ) = sin  (xxi) cos (180° + ) = – cos 
(xxii) tan (180° + ) = tan  (xxiii) sin(– ) = – sin 
(xxiv) cos (–) = cos  (xxv) tan (–) = – tan 

sin A sin B sin C


• Sine Rule   B
a b c
c a
• Cosine rule a = b + c – 2bc cos A
2 2 2

A C
b

4 3
90°
Ex.23 37° 53°
x
Find x :
sin 90  sin 53 
Sol. 
x 4
x=5

14. SMALL ANGLE APPROXIMATION

It is a useful simplification which is only approximately true for finite angles. It involves linerarization of
the trigonometric functions so that, when the angle  is measured in radians.
sin  ~ 

: 0744-2209671, 08003899588 | url : www.motioniitjee.com, : info@motioniitjee.com


Page # 22 UNITS AND DIMENSIONS

2
cos ~ 1 or cos  ~ 1 – for the second - order approximation
2
tan  ~ 

Geometric justification

tan Object
tan
arc arc
D
 
d
Small angle approximation. The value of the small angle X in radians is
approximately equal to its tangent.
• When one angle of a right triangle is small, is hypotenuse in approximately equal in length to the leg
adjacent
to the small angle, so the cosine is approximately 1.
• The short leg is approximately equal to the arc from the long leg to the hypotenuse, so the sine and
tangent are
both approximated by the value of the angle in radians.
15. BINOMIAL THEOREM :

n(n – 1)x 2
(1 ± x)n = 1 ± nx + .......... .
2!

n(n  1) 2
(1 ± x)–n = 1  nx + x .........
2!
If x << 1 ; then
(1 ± x)n = 1 ± nx (neglecting higher terms)
(1 ± x)–n = 1 ± (–n)x = 1  nx
(1 + x)2 = 1 + 2x+ x2
(1 + x)3 = 1 + 3x + x3 – 3x2
(1 + x)n = 1 + nx ..............
if x << 1

Note : (1) When n is a positive integer, then expansion will have (n + 1) terms
(2) When n is a negative integer, expansion will have infinite terms.
(3) When n is a fraction expansion will have infinite terms.

Ex. 24 Calculate (1001)1/3.

 1 
Sol. We can write 1001 as : 1001 = 1000 1   , so that we have
e
 1000 

1/ 3 1/ 3
  1   1 
(1001)1/ 3  10001    10 1  
  1000   1000 

1
 10(1  0.001)1/ 3  10(1   0.001)
3
= 10.003333

Corporate Head Office : Motion Education Pvt. Ltd., 394 - Rajeev Gandhi Nagar, Kota-5 (Raj.)
UNITS AND DIMENSIONS Page # 23

Ex.25 Expand (1+x)–3.

( 3)(3  1)x 2 ( 3)( 3  1)( 3  2) 3


Sol. (1  x ) 3  1  ( 3)x   x 
2! 3!

12 2 60 3
 1 3x  x  x  .......
2 3 2

 1  3 x  6x 2  10 x 3  .......

16. GRAPHS :
Following graphs and their corresponding equations are frequently used in Physics.
(i) y = mx, represents a straight line passing through origin. Here, m = tan  is also called the slope of
line, where  is the angle which the line makes with positive x-axis, when drawn in anticlockwise
direction from the positive x-axis towards the line.
y y

 
x x

(i) (ii)
The two possible cases are shown in figure 1.1 (i)  < 90°. Therefore, tan  or slope of line is positive.
In fig. 1.1 (ii), 90° <  < 180°. Therefore, tan  or slope of line is negative.
1
Note : That y = mx of y  x also means that value of y becomes 2 time if x is doubled. Or it becomes th if x
4
x
becomes , and c the intercept on y-axis.
4

y y

c=+ve c=+ve  
 x x
x
c=–ve

(i) (ii) (iii)

In figure (i) : slope and intercept both are positive.


In figure (ii) : slope is negative but intercept is positive and
In figure (iii) : slope is positive but intercept is negative.
Note : That in y = mx + c, y does not become two times if x is doubled

Ex.26 v = u + at

t
.
Ex.27 P = mv

: 0744-2209671, 08003899588 | url : www.motioniitjee.com, : info@motioniitjee.com


Page # 24 UNITS AND DIMENSIONS

O v

Ex.28 Draw the graph for the equation : 2y = 3x + 2


3 (0,1)
Sol. 2y = 3x + 2 y = x  1 3
2 tan 
3  2
m= > 0   < 90°
2
c = +1 > 0
 The line will pass through (0, 1)

Ex.29 Draw the graph for the equation : 2y + 4x + 2 = 0 tan = –2


Sol. 2y + 4x + 2 = 0  y = – 2x – 1 
m = – 2 < 0 i.e.,  > 90°
c = – 1 i.e.,
line will pass through (0, –1) (0,–1)

: (i) If c = 0 line will pass through origin.


(ii) y = c will be a line parallel to x axis.
(0,c)

(0,0)
(iii) x = c will be a line perpendicular to y axis

(c,0)
(0,0)

(ii) Parabola
A general quadratic equation represents a parabola.
y = ax2 + bx + c a0
if a > 0 ; It will be a opening upwards parabola.
if a < 0 ; It will be a opening downwards parabola.
if c = 0 ; It will pass through origin.

y  x2 or y = 2x2, etc. represents a parabola passing through origin as shown in figure shown.
y y

x  y2
y  x2
x x

k = 1/2mv2

(i) (ii)

Corporate Head Office : Motion Education Pvt. Ltd., 394 - Rajeev Gandhi Nagar, Kota-5 (Raj.)
UNITS AND DIMENSIONS Page # 25

1
e.g. y = 4 x2 + 3x e.g. k= mv2
2

k
2
y=4x +3x

v
2
y=–4x +3x

Note : That in the parabola y = 2x2 or y  x2, if x is doubled, y will beome four times.
Graph x  y2 or x = 4 y2 is again a parabola passing through origin as shown in figure shown. In this cae
if y is doubled, x will become four times.
y = x2 + 4 or x = y2 – 6 will represent a parabola but not passing through origin. In the first equation
(y = x2 + 4), if x doubled, y will not become four times.
17. SIMILAR TRIANGLE
Two given triangle are said to be similar if
(1) All respective angle are same
or
(2) All respective side ratio are same.
P
A

B C Q R
As example, ABC, PQR are two triangle as shown in figure.
If they are similar triangle then
(1) A=P
B=Q
C=R
OR

AB BC AC
(2)  =
PQ QR PR

A
P
5
3
Ex.30 O
B Q x
6

Find x :
Sol. By similar triangle concept

AB OB

PQ OQ

5 6 18
  x=
3 x 5
: 0744-2209671, 08003899588 | url : www.motioniitjee.com, : info@motioniitjee.com
Page # 26 UNITS AND DIMENSIONS

Exercise - I OBJECTIVE PROBLEMS (JEE MAIN)


SECTION A : UNITS Sol.
1. Which of the following is not the name of a physical
quantity ?
(A) kilogram (B) impulse
(C) energy (D) density
Sol.

6. Unit of pressure in S.I. system is-


(A) atmosphere (B) dynes per square cm
(C) pascal (D) bar
Sol.
2. Light year is the unit of
(A) speed (B) mass
(C) distance (D) time
Sol.

7. In SI unit the angular acceleration has unit of-


(A) Nmkg-1 (B) ms-2
(C) rad.s-2
(D) Nkg-1
3. PARSEC is a unit of Sol.
(A) Time (B) Angle
(C) Distance (D) Velocity
Sol.

8. The SI unit of the universal gravitational constant


G is
(A) Nm kg–2 (B) Nm2kg–2
(C) Nm2 kg–1 (D) Nmkg–1
Sol.
4. Which of the following system of units is NOT based
on the unit of mass, length and time alone
(A) FPS (B) SI
(C) CGS (D) MKS
Sol.

5. In the S.I. system the unit of energy is- 9. Surface tension has unit of-
(A) erg (B) calorie (A) Joule.m2 (B) Joule.m-2
(C) joule (D) electron volt (C) Joule.m (D) Joule.m3

Corporate Head Office : Motion Education Pvt. Ltd., 394 - Rajeev Gandhi Nagar, Kota-5 (Raj.)
UNITS AND DIMENSIONS Page # 27

Sol. 14. The unit of magnetic moment is-


(A) amp m2 (B) amp m-2
(C) amp m (D) amp m-1
Sol.

10. The unit of intensity of magnetisation is- 15. The SI unit of the universal gas constant R is :
(A) Amp m2 (B) Amp m-2 (A) erg K–1 mol–1 (B) watt K–1 mol–1
(C) Amp m (D) Amp m-1 (C) newton K mol
–1 –1
(D) joule K–1 mol–1
Sol.
Sol.

11. The M.K.S. units of coefficient of viscosity is-


(A) kg m-1s-1 (B) kg m s-2 16. A physical quantity is measured and the result is
(C) kg m s
2 -1
(D) kg-1 m-1 s2 expressed as nu where u is the unit used and n is the
Sol. numerical value. If the result is expressed in various
units then
(A) n  size of u (B) n  u2
(C) n  u (D) n  1/u
Sol.

12. The specific resistance has the unit of-


(A) ohm/m (B) ohm/m2
(C) ohm.m2
(D) ohm.m
Sol.

17. One watt-hour is equivalent to


(A) 6.3 × 103 Joule (B) 6.3 × 10–7 Joule
(C) 3.6 × 103 Joule (D) 3.6 × 10–3 Joule
Sol.

13. The mutual inductance has unit of-


(A) Gauss (B) Weber
(C) Farad (D) Henry
Sol.

: 0744-2209671, 08003899588 | url : www.motioniitjee.com, : info@motioniitjee.com


Page # 28 UNITS AND DIMENSIONS

18. Which of the following statement is wrong ? Sol.


(A) Unit of K.E. is Newton-metre
(B) Unit of viscosity is poise
(C) Work and energy have same dimensions
(D) Unit of surface tension is Newton metre
Sol.

23. The dimensions of universal gravitational constant


are
(A) M–1 L3 T–2 (B) M–1 L3 T–1
(C) M L T
–1 –1 –2
(D) M–2 L2 T–2
Sol.
SECTION : B DIMENSIONS
19. What are the dimensions of lenth in force ×
displacement/time
(A) –2 (B) 0 (C) 2 (D) none of these
Sol.

24. The SI unit of Stefan's constant is :


(A) Ws–1 m–2 K–4 (B) J s m–1 K–1
(C) J s m K
–1 –2 –1
(D) W m–2 K–4
Sol.
20. The angular frequency is measured in rad s–1. Its
dimension in length are :
(A) – 2 (B) –1 (C) 0 (D) 2
Sol.

25. What are the dimensions of Boltzmann's constant?


21 [M L T ] are the dimensions of-
-1 (A) MLT–2K–1 (B) ML2T–2K–1
(A) power (B) momentum (C) M LT
0 –2
(D) M0L2T–2K–1
(C) force (D) couple Sol.
Sol.

22. The dimensional formula for angular momentum


is- 26. Dimensions of magnetic flux density is -
(A) ML2T -2 (B) ML2T-1 (A) M1 L0 T-1A-1 (B) M1 L0 T-2A-1
(C) MLT -1
(D) M0L2T-2 (C) M L T A
1 1 -2 -1
(D) M1 L0 T-1A-2

Corporate Head Office : Motion Education Pvt. Ltd., 394 - Rajeev Gandhi Nagar, Kota-5 (Raj.)
UNITS AND DIMENSIONS Page # 29

Sol. L
30. The dimensions of the quantity are -
RCV
(A) M0 L0 T1A1 (B) M0 L0 T-1A-1
(C) M0 L0 T0A1 (D) M0 L0 T0A-1
Sol.

27. A pair of physical quantities having the same


dimensional formula is :
(A) angular momentum and torque 31. For 10(at+3) , the dimension of a is-
(B) torque and energy (A) M0 L0 T0 (B) M0 L0 T1
(C) force and power (C) M0 L0 T-1 (D) None of these
(D) power and angular momentum Sol.
Sol.

32. The pressure of 106 dyne/cm2 is equivalent to


(A) 105 N/m2 (B) 106 N/m2
(C) 10 N/m
7 2
(D) 108 N/m2
28. Dimensions of pressure are the same as that of Sol.
(A) force per unit volume
(B) energy per unit volume
(C) force
(D) energy
Sol.
33. The SI unit of length is the meter. Suppose we
adopt a new unit of length which equals to
x meters. The area 1m2 expressed in terms of the
new unit has a magnitude-
(A) x (B) x2
1 1
(C) (D)
x x2
Sol.

29. Which one of the following has the dimensions of


ML–1T–2 ?
(A) torque (B) surface tension 34.  = 2 g/cm3 convert it into MKS system -
(C) viscosity (D) stress kg kg
(A) 2 × 10–3 3 (B) 2 × 103 3
Sol. m m
kg kg
(C) 4 × 10 3
3 (D) 2 × 10 6
m m3
Sol.

: 0744-2209671, 08003899588 | url : www.motioniitjee.com, : info@motioniitjee.com


Page # 30 UNITS AND DIMENSIONS

35. Given that v is the speed, r is radius and g is 38. The density of mercury is 13600 kg m–3. Its value
acceleration due to gravity. Which of the following is of CGS system will be :
dimension less (A) 13.6 g cm–3 (B) 1360 g cm–3
v 2g v2 (C) 136 g cm–3 (D) 1.36 g cm–3
(A) (B) v2rg (C) vr2g (D) Sol.
r rg
Sol.

36. Choose the correct statement(s) :


(A) A dimensionally correct equation must be correct.
(B) A dimensionally correct equation may be correct. BASIC MATHEMATICS
(C) A dimensionally incorrect equation may be correct. 39. The radius of two circles are r and 4r what will be
(D) A dimensionally incorrect equation may be the ratio of their Area and perimeter.
incorrect. Sol.
Sol.

37. The value of G = 6.67 × 10–11 N m2 (kg)–2. Its


numerical value in CGS system will be :
(A) 6.67 × 10–8 (B) 6.67 × 10–6 40. Internal radius of a ball is 3 cm and external radius
(C) 6.67 (D) 6.67 × 10–5 is 4 cm. What will be the volume of the material used.
Sol. Sol.

Corporate Head Office : Motion Education Pvt. Ltd., 394 - Rajeev Gandhi Nagar, Kota-5 (Raj.)
UNITS AND DIMENSIONS Page # 31

Exercise - II (JEE ADVANCED)


1. Which of the following sets cannot enter into the 5. If the unit of length is micrometer and the unit of
list of fundamental quantities in any system of units ? time is microsecond, the unit of velcoity will be :
(A) length, mass and velocity (A) 100 m/s (B) 10 m/s
(B) length, time and velocity (C) micrometers (D) m/s
(C) mass, time and velocity Sol.
(D) length, time and mass
Sol.

2. Which of the following is not the unit of time 6. A dimensionless quantity :


(A) solar day (B) parallactic second (A) never has a unit (B) always has a unit
(C) leap year (D) lunar month (C) may have a unit (D) does not exit
Sol. Sol.

3. The unit of impulse is the same as that of : 7. A unitless quantity :


(A) moment force (A) never has a nonzero dimension
(B) linear momentum (B) always has a nonzero dimension
(C) rate of change of linear momentum (C) may have a nonzero dimension
(D) force (D) does not exit
Sol. Sol.

8. If a and b are two physical quantities having different


dimensions then which of the following can denote a
4. Which of the following is not the unit of energy? new physical quantity
(A) watt-hour (B) electron-volt (A) a + b (B) a – b
(C) N × m (D) kg × m/sec2 (C) a/b (D) ea/b (E) sin (a/b)
Sol. Sol.

: 0744-2209671, 08003899588 | url : www.motioniitjee.com, : info@motioniitjee.com


Page # 32 UNITS AND DIMENSIONS

9. Two physical quantities whose dimensions are not 12. The velocity 'v' (in cm/s) of a particle is given in
same, cannot be : terms of time 't' (in s) by the equation
(A) multiplied with each other
b
(B) divided v = at +
tc
(C) added or substracted in the same expression
(D) added together The dimensions of a, b and c are
Sol. a b c a b c
(A) L2 T LT2 (B) LT2 LT L
(C) LT–2 L T (D) L LT T2
Sol.

10. Choose the correct statement(s) :


(A) All quantities may be represented dimensionally in
terms of the base quantities.
(B) A base q uant i t y cannot b e re pres ente d
dimensionally in terms of the rest of the base
quantities.
(C) The dimension of a base quantity in other base
quantities is always zero.
(D) The dimension of a derived quantity is never zero 13. The position of a particle at time 't' is given by
in any base quantity. the relation
Sol.
V0
x(t) = [1 – e – t ]

where V0 is a constant and  > 0. The dimensions of
V0 and  are respectively.
(A) M0L1T0 and T–1 (B) M0L1T0 and T–2
(C) M L T and T
0 1 –1 –1
(D) M0L1T–1 and T–2
Sol.

11. Planck's constant has the dimensions of :


(A) force (B) energy
(C) linear momentum (D) angular momentum
Sol.

Corporate Head Office : Motion Education Pvt. Ltd., 394 - Rajeev Gandhi Nagar, Kota-5 (Raj.)
UNITS AND DIMENSIONS Page # 33

14. The time dependence of a physical quantity ? Sol.


P = P0exp(– t2)
where  is a constant and t is time
The constant 
(A) will be dimensionless
(B) will have dimensions of T–2
(C) will have dimensions as that of P
(D) will have dimensions equal to the dimension of P
multiplied by T–2
Sol.
17. In above question 16, the dimensions of b are the
same as those of
(A) P (B) V (C) PV (D) nRT
Sol.

15. Force F is given in terms of time t and distance x


18. In above question 16, the dimensions of nRT are
by the same as those of
F = A sin C t + B cos D x
(A) energy (B) force
A C (C) pressure (D) specific heat
Then the dimensions of and are given by
B D Sol.
(A) MLT , M L T
–2 0 0 –1
(B) MLT , M0L–1T0
–2

(C) M L T , M L T
0 0 0 0 1 –1
(D) M0L1T–1, M0L0T0
Sol.

19. In above question 16, the dimensional formula for


ab is
(A) ML2T–2 (B) ML4T–2 (C) ML6T–2 (D) ML8T–2
Sol.

16. The Van der Waal equation for 1 mole of a real gas
is
 a 
 P  2  ( V – b)  RT
 V  20. Which pair of following quantities has dimensions
where P is the pressure, V is the volume, T is the different from each other.
absolute temperature, R is the molar gas constant (A) Impulse and linear momentum
and a, b are Van dar Waal constants. The dimensions (B) Plank's constant and angular momentum
of a are the same as those of (C) Moment of inertia and moment of force
(A) PV (B) PV2 (C) P2V (D) P/V (D) Young's modulus and pressure

: 0744-2209671, 08003899588 | url : www.motioniitjee.com, : info@motioniitjee.com


Page # 34 UNITS AND DIMENSIONS

Sol. Sol.

b
24. In the above question dimensions of are the
c
21. If force (F) is given by F = Pt–1 +  t, where t is same as those of
time. The unit of P is same as that of (A) wave velocity (B) wavelength
(A) velocity (B) displacement (C) wave amplitude (D) wave frequency
(C) acceleration (D) momentum Sol.
Sol.

25. What is the physical quantity whose dimensions


are M L2 T–2 ?
(A) kinetic energy (B) pressure
22. The product of energy and time is called action. (C) momentum (D) power
The dimensional formula for action is same as that for Sol.
(A) power (B) angular energy
(C) force × velocity (D) impulse × distance
Sol.

26. If force, acceleration and time are taken as


fundamental quantities, then the dimensions of length
will be :
(A) FT2 (B) F–1 A2 T–1 (C) FA2T (D) AT2
Sol.

23. When a wave traverses a medium, the displacement


of a particle located at x at time t is given by
y = a sin (bt – cx)
where a, b and c are constants of the wave. The
dimensions of b are the same as those of
(A) wave velocity (B) amplitude
(C) wavelength (D) wave frequency

Corporate Head Office : Motion Education Pvt. Ltd., 394 - Rajeev Gandhi Nagar, Kota-5 (Raj.)
UNITS AND DIMENSIONS Page # 35

27. The dimensions ML–1T–2 can correspond to Sol.


(A) moment of a force or torque
(B) surface tension
(C) pressure
(D) co-efficient of viscosity

(useful relation are   r  F , S = F/l, F = 6 r v,,
where symbols have usual meaning)
Sol.

31. In a book, the answer for a particular question is

ma  2kl 
expressed as b   1  here m represents
28. Which of the following can be a set of fundamental k  ma 
quantities mass, a represents accelerations, l represents length.
(A) length, velocity, time The unit of b should be
(B) momentum, mass, velocity (A) m/s (B) m/s2 (C) meter (D) /sec
(C) force, mass, velocity Sol.
(D) momentum, time, frequency
Sol.

29. If area (A) velocity (v) and density () are base F
32.  = sin(t) (here V = velocity, F = force, t =
units, then the dimensional formula of force can be V2
represented as time) : Find the dimension of  and  -
(A) Av (B) Av2 (C) Av2 (D) A2v (A)  = [M1L1T0],  = [T–1]
Sol. (B)  = [M1L1T–1],  = [T1]
(C)  = [M1L1T–1],  = [T–1]
(D)  = [M1L–1T0],  = [T–1]
Sol.

30. In a certain system of units, 1 unit of time is 5


sec, 1 unit of mass is 20 kg and unit of length is 10m.
In this system, one unit of power will correspond to
(A) 16 watts (B) 1/16 watts
(C) 25 watts (D) none of these

: 0744-2209671, 08003899588 | url : www.motioniitjee.com, : info@motioniitjee.com


Page # 36 UNITS AND DIMENSIONS

33. If E, M, J and G denote energy, mass, angular 36. If the unit of force is 1 kilonewton, the length is 1
momentum and gravitational constant respectively, km and time is 100 second, what will be the unit of
mass :
EJ2 (A) 1000 kg (B) 10 kg
then has the dimensions of
M5 G2 (C) 10000 kg (D) 100 kg
(A) length (B) angle (C) mass (D) time Sol.
Sol.

34. The dimensions ML–1T–2 may correspond to


37. A body moving through air at a high speed 'v'
(A) work done by a force (B) linear momentum
experiences a retarding force 'F' given by F = K A d vx
(C) pressure (D) energy per unit volume
Sol. where 'A' is the surface area of the body, 'd' is the
density of air and 'K' is a numerical constant. The value
of 'x' is :
(A) 1 (B) 2 (C) 3 (D) 4
Sol.

35. The velocity of water waves may dpend on their


wavelength , the density of water  and the
acceleration due to gravity g. The method of
dimensions gives the relation between these quantities
as
(A) v2 = k–1 g–1 –1
(B) v2 = k g 
(C) v2 = k g  38. The velocity of a freely falling body changes as gp
(D) v2 = k 3 g–1 –1 where k i s a di mensi onl e ss hq where g is acceleration due to gravity and h is the
constant height. The values of p and q are :
Sol. 1 1 1
(A) 1, (B) ,
2 2 2
1
(C) ,1 (D) 1, 1
2
Sol.

Corporate Head Office : Motion Education Pvt. Ltd., 394 - Rajeev Gandhi Nagar, Kota-5 (Raj.)
UNITS AND DIMENSIONS Page # 37

41. The units of length, velocity and force are doubled.


Which of the following is the correct change in the
other units ?
(A) unit of time is doubled
(B) unit of mass is doubled
(C) unit of momentum is doubled
(D) unit of energy is doubled
Sol.

39. If the acceleration due to gravity is 10 ms–2 and


the units of length and time are changed to kilometre
and hour, respectively, the numerical value of the
acceleration is :
(A) 360000 (B) 72000
(C) 36000 (D) 129600
Sol.

42. If the units of force and that of length are doubled,


the unit of energy will be :
(A) 1/4 times (B) 1/2 times
(C) 2 times (D) 4 times
Sol.

40. If 'c' the velocity of light 'g' the acceleration due


to gravity and 'P" the atmospheric pressure are
fundamental units, then the dimensions of length will
be
(A) c/g (B) P × c × g (C) c/P (D) c2/g
Sol.

43. If the units of M, L are doubled then the unit of


kinetic energy will become
(A) 2 times (B) 4 times
(C) 8 times (D) 16 times
Sol.

: 0744-2209671, 08003899588 | url : www.motioniitjee.com, : info@motioniitjee.com


Page # 38 UNITS AND DIMENSIONS

44. Binomial
(a) (99)1/2 (b) (120)1/2 (c) (126)1/3
Sol.

4
x 2
45. A normal human eye can see an object making an
47. Find x and y :
angle of 1.8° at the eye. What is the approximate y
height of object which can be seen by an eye placed
2
at a distance of 1 m from the eye.
3
Sol.
h
1.8°

1m
Sol.

48. Which of the following is not the unit length :


(A) micron (B) light year
(C) angstrom (D) radian
Sol.

46. Draw graph for following equations :


(i) v = v0 – at (ii) x = 4t – 3
(iii) x = 4 at2 (iv) v = – gt
Sol.

Corporate Head Office : Motion Education Pvt. Ltd., 394 - Rajeev Gandhi Nagar, Kota-5 (Raj.)
UNITS AND DIMENSIONS Page # 39

49. A particle is in a uni-directional potential field Sol.


where the potential energy (U) of a particle depends
on the x-coordinate given by Ux = k(1 – cos ax) & k
and ‘a’ are constants. Find the physical dimensions of
‘a’ & k.
Sol.

52. The distance moved by a particle in time t from


centre of a ring under the influence of its gravity is
given by x = a sint where a &  are constants. If  is
found to depend on the radius of the ring (r), its mass
(m) and universal gravitational constant (G), find using
dimensional analysis an expression for  in terms of r, m
50. The time period (T) of a spring mass system and G.
depends upon mass (m) & spring constant (k) & length Sol.
Force
of the spring (l) [k  ] . Find the relation among,
length
(T), (m), (l) & (k) using dimensional method.
Sol.

53. If the velocity of light c, Gravitational constant G


& Plank’s constant h be chosen as fundamental units,
find the dimension of mass, length & time in the new
system.
Sol.

51. The equation of state for a real gas at high


nRT a
temperature is given by P   1/ 2 where
V  b T V( V  b)
n, P V & T are number of moles, pressure, volume &
temperature respectively & R is the universal gas
constant. Find the dimensions of constant ‘a’ in the
above equation.

: 0744-2209671, 08003899588 | url : www.motioniitjee.com, : info@motioniitjee.com


Page # 40 UNITS AND DIMENSIONS

54. A satellite is orbiting around a planet. Its orbital


velocity (v0) is found to depend upon
(a) Radius of orbit (R)
(b) Mass of planet (M)
(c) Universal gravitation constant (G)
Using dimensional analysis find an expression relating
orbital velocity (v0) to the above physical quantities.
Sol. 56. Use the approximation (1 + x)n  1 + nx, |x| << 1,
to find approximate value for

1
(a) 99 (b)
1.01
Sol.

57. Use the small angle approximations to find


approximate values for
(a) sin 8° and (b) tan 5°
Sol.

55. The angle subtended by the moon's diameter at a


point on the earth is about 0.50°. Use this and the
act that the moon is about 384000 km away to find
the approximate diameter of the moon.

 D
rm

(A) 192000 km (B) 3350 km


(C) 1600 km (D) 1920 km
Sol.

Corporate Head Office : Motion Education Pvt. Ltd., 394 - Rajeev Gandhi Nagar, Kota-5 (Raj.)
UNITS AND DIMENSIONS Page # 41

Exercise - III PREVIOUS YEAR QUESTIONS

LEVEL - I JEE MAIN

1. Identify the pair whose dimensions are equal. 4. Which one of the following represents the correct
[AIEEE 2002] dimensions of the coefficient of viscosity ?
(A) Torque and work (B) Stress and energy [AIEEE 2004]
(C) Force and stress (D) Force and work (A) [ML T ]
–1 –2
(B) [MLT ] –1

Sol. (C) [ML T ]


–1 –1
(D) [ML–2T–2]
Sol.

2. The physical quantities not having same dimen-


sions are [AIEEE 2003]
(A) torque and work
(B) momentum and Planck's constant
(C) stress and Young's modulus

(D) speed and ( 0 0 ) 1/ 2


5. Out of the following pairs, which one does not
Sol. have identical dimensions ? [AIEEE 2005]
(A) Angular momentum and Planck's constant
(B) Impulse and momentum
(C) Moment of inertia and moment of a force
(D) Work and torque
1 Sol.
3. Dimensions of ,
 0 0 where symbols have their
usual meaning, are [AIEEE 2003]
(A) [L–1T] (B) [L2 T2]
(C) [L2T–2] (D) [LT–1]
Sol.

6. The 'rad' is the correct unit used to report the


measurement of [AIEEE 2006]
(A) the ability of a beam of gamma ray photons to
produce ions in a target
(B) the energy delivered by radiation to a target
(C) the biological effect of radiation
(D) the rate fo decay of a radioactive source

: 0744-2209671, 08003899588 | url : www.motioniitjee.com, : info@motioniitjee.com


Page # 42 UNITS AND DIMENSIONS

Sol. Sol.

7. Which of the following units denotes the dimen- 10.In an experiment the angles are required to be
sions [ML /Q ], where Q denotes the electric charges?
2 2
measured using an instrument. 29 divisions of the main
[AIEEE 2006] scale exactly coincide with the 30 divisions of the
(A) Wb/m 2
(B) henry (H) vernier scale. If the smallest division of the main scale
(C) H/m 2
(D) weber (Wb) is half-a-degree (= 0.5°), then the least count of the
Sol. instrument is [AIEEE 2009]
(A) one minute (B) half minute
(C) one degree (D) half degree

8. A body of mass m = 3.513 kg is moving along the


x-axis with a speed of 5.00 ms–1. The magnitude of its
11.The respective number of significant figures for
momentum is recorded as [AIEEE 2008]
(A) 17.6 kg ms–1 (B) 17.565 kg ms–1 the numbers 23.023, 0.0003 and 21 × 10 –3 are
[AIEEE 2010]
(C) 17.56 kg ms–1 (D) 17.57 kg ms–1
(A) 5, 1, 2 (B) 5, 1, 5
Sol.
(C) 5, 5, 2 (D) 4, 4, 2

9. The dimensions of magnetic field in M, L, T and C


(coulomb) is given as [AIEEE 2008]
(A) [MLT C ]
– –1
(B) [MT C–2]
2

(C) [MT–1C–1] (D) [MT2C–1]

Corporate Head Office : Motion Education Pvt. Ltd., 394 - Rajeev Gandhi Nagar, Kota-5 (Raj.)
UNITS AND DIMENSIONS Page # 43

LEVEL - II JEE ADVANCED


Note (*) means not for speed batch 3*. Which of the following pairs have same dimensions :
1*. The pairs of physical quantities that have the (A) Torque and work
same dimensions are : (B) Angular momentum and work
(A) Raynolds number and coefficient of friction (C) Energy and young's modulus
(B) Latent heat and gravitational potential (D) Light year and wavelength [JEE-1996' 2/100]
(C) curie and frequency of light wave Sol.
(D) Planck's constant and torque
[JEE - 1995'2/100]
Sol.

4*. The SI unit of inductance, the henry can be written


as : [JEE-1998' 2/200]
2*. In the formula X = 3YZ2, X and Z have dimensions (A) weber/ampere (B) volt-second/ampere
of capacitance and magnetic induction respectively. (C) joule/(ampere)2 (D) ohm-second
What are the dimensions of Y in MKSQ system ? Sol.
[JEE-1995,2/100]
(A) [M–3 L–1 T3 Q4] (B) [M–3 L–2 T4 Q4]
(C) [M–2 L–2 T4 Q4] (D) [Mß L–2 T4 Q1]
Sol.

: 0744-2209671, 08003899588 | url : www.motioniitjee.com, : info@motioniitjee.com


Page # 44 UNITS AND DIMENSIONS

5*. Let [0] denote the dimensional formula of the Sol.


permittivity of the vaccum, and [0 ] that of the
permeability of the vacuum. If M = mass, L = length,
T = time and I = electric current :[JEE-1998' 2/200]
(A) [0] = M–1 L–3 T2 I (B) [0] = M–1 L–3 T4 I2
(C) [0] = M LT–2 I–2 (D) [0] = M L2 T–1 I
Sol.

  z 
8*. Pressure depends on distance as, P = exp –  ,
  k 
where ,  are constants, z is distance, k is Boltzmann's
constant and  is temperature. The dimension of  are
[JEE-2004s '3/84]
(A) M0 L0 T0 (B) M–1 L–1 T–1
(C) M0 L2 T0 (D) M–1 L1 T2
Sol.

 1
6*. The dimensions of  2  0 E2 (0 : permittivity of

free space, E : electric field) is :


[JEE Sc 2000' 2/200]
(A) MLT–1 (B) ML2T–2 (C) ML–1T–2 (D) ML2T–1
Sol.

9*. Which of the following set have different dimensions ?


[JEE-2005s ; 3/60]
(A) Pressure, Young's modulus, Stress
(B) Emf, Potential difference, Electric potential
(C) Heat, Work done, Energy
(D) Dipole moment, Electric flux, Electric field
Sol.

V
7*. A quantity X is given by 0L . where 0 is the
t
permittivity of free space, L is length, V is potential
difference and t is time interval. The dimensional
formula for X is the same as that of
[JEE Sc.2000'3/105]
(A) resistance (B) charge
(C) voltage (D) current

Corporate Head Office : Motion Education Pvt. Ltd., 394 - Rajeev Gandhi Nagar, Kota-5 (Raj.)
UNITS AND DIMENSIONS Page # 45

Exercise - I OBJECTIVE PROBLEMS (JEE MAIN)


1. A 2. C 3. C 4. B 5. C 6. C 7. C 8. B
9. B 10. D 11. A 12. D 13. D 14. A 15. D 16. D
17. C 18. D 19. C 20. C 21. B 22. B 23. A 24. D
25. B 26. B 27. B 28. B 29. D 30. D 31. C 32. A
1 1 148 
33. D 34. B 35. D 36. ABD 37. A 38. A 39. , 40.
16 4 3

Exercise - II (JEE ADVANCED)

1. B 2. B 3. B 4. D 5. D 6. C 7. A 8. C
9. C 10. ABC 11. D 12. C 13. C 14. B 15. C 16. B
17. B 18. A 19. D 20. C 21. D 22. D 23. D 24. A
25. A 26. D 27. C 28. C 29. B 30. A 31. C 32. D
33. B 34. CD 35. B 36. C 37. B 38. B 39. D 40. D
41. C 42. D 43. C 44. (a) 9.9498 (b) 10.954 (c) 5.0132 45.  cm
v v
v0 x x

46. (i) t (ii) t (iii) (iv) t


t

–3
8 3 m Gm
47. x= ,y= 48. D 49. L–1, ML2T–2 50. T = a 51. ML5T–2K1/2 52.  = K
3 2 k r3

Gm
53. [M] = [h1/2.c1/2.G–1/2] ; [L] = [h1/2.c–3/2.G1/2] ; [T] = [h1/2.c–5/2.G1/2] 54. v0 = k
R
55. B 56. (a) 9.95, (b) 0.99 57. 0.14, 0.09

Exercise - III PREVIOUS YEAR QUESTIONS

LEVEL - I JEE MAIN


1. A 2. B 3. C 4. C 5. C 6. C 7. B
8. A 9. C 10. A 11. A

LEVEL - II JEE ADVANCED

1. A,B,C 2. B 3. A,D 4. A,B,C,D 5. B,C


6. C 7. D 8. C 9. D

: 0744-2209671, 08003899588 | url : www.motioniitjee.com, : info@motioniitjee.com


Page # 46 VECTOR & CALCULUS

VECTOR & CALCULUS


1. SCALAR :
In physics we deal with two type of physical quantity one is scalar and other is vector. Each scalar
quantity has a magnitude and a unit.
For example mass = 4kg
Magnitude of mass =4
and unit of mass = kg
Example of scalar quantities : mass, speed, distance etc.
Scalar quantities can be added, subtracted and multiplied by simple laws of algebra.

2. VECTOR :

Vector are the physical quantites having magnitude as well as specified direction.
For example :
Speed = 4 m/s (is a scalar)
Velocity = 4 m/s toward north (is a vector)
If someone wants to reach some location then it is not sufficient to provide information about the
distance of that location it is also essential to tell him about the proper direction from the initial
location to the destination.
 
The magnitude of a vector ( A ) is the absolute value of a vector and is indicated by | A | or A.
Example of vector quantity : Displacement, velocity, acceleration, force etc.

Knowledge of direction
N

W E

S
3. GENERAL POINTS REGARDING VECTORS :

3.1 Representation of vector :


Geometrically, the vector is represented by a line with an arrow indicating the direction of vector as
Tail Head
Length
(magnitude)
Y 
 A
Mathematically, vector is represented by A .
Sometimes it is represented by bold letter A.


Thus, the arrow in abow figure represents a vector A
X
in xy-plane making an angle  with x-axis.

A representation of vector will be complete if it gives us direction and magnitude.

Corporate Head Office : Motion Education Pvt. Ltd., 394 - Rajeev Gandhi Nagar, Kota-5 (Raj.)
VECTOR & CALCULUS Page # 47

   
Symbolic form : v, a,F, s used to separate a vector quantity from scalar quantities (u, i, m)

Graphical form : A vector is represented by a directed straight line,


N
having the magnitude and direction of the quantity represented by it. B
5cm
e.g. if we want to represent a force of 5 N acting 45° N of E Head
(i) We choose direction co-ordinates. 45°
W E
(ii) We choose a convenient scale like 1 cm  1 N A
(iii) We draw a line of length equal in magnitude and in the direction
tail
of vector to the chosen quantity. S
(iv) We put arrow in the direction of vector. 1cm  1N
AB
Magnitude of vector :

| A B | 5 N

3.2 Angle between two Vectors ()


Angle between two vectors means smaller of the two angles between the vectors when they are
placed tail to tail by displacing either of the vectors parallel to itself (i.e 0).

 
B B

B
  
   
A A A

       
Ex.1 Three vectors A,B,C are shown in the figure. Find angle between (i) A and B , (ii) B and C , (iii) A

and C .

x x
 30º 45º
A  
B C
30º
x

Sol. To find the angle between two vectors we connect the tails of the two vectors. We can shift B &

C such that
   
tails of A,B and C are connected as shown in figure. A
  30º x
Now we can easily observe that angle between A and B is 60º, 30º
45º
     
B and C is 15º and between A and C is 75º. C B

3.3 Negative of Vector


It implies vector of same magnitude but opposite in direction.

 
A –A

: 0744-2209671, 08003899588 | url : www.motioniitjee.com, : info@motioniitjee.com


Page # 48 VECTOR & CALCULUS

3.4 Equality of Vectors.


Vectors having equal magnitude and same direction are called equal vectors

C


A 
B
  
if | A| |B| | C|
and A   B  C 
  
then A  B  C

3.5 Collinear vectors :


Any two vectors are co-linear then one can be express in the term of other.
 
a = b (where  is a constant)
3.6 Co-initial vector : If two or more vector start from same point then they called co-initial vector.
A
 
A B
B
C

e.g. C

O  D
D
here A, B, C, D are co-initial.

3.7 Coplanar vectors :


Three (or more) vectors are called coplanar vectors if they lie in the same plane or are parallel to the
same plane. Two (free) vectors are always coplanar.

Important points
 If the frame of reference is translated or rotated the vector does not change (though its components
may change).

or 

ect
B v A S'

O'
S
O
Two vectors are called equal if their magnitudes and directions are same, and they represent values of
same physical quantity.

3.8 Multiplication and division of a vector by a scalar


  
Multiplying a vector A with a positive number  gives a vector (B   A ) whose magnitude become 
 
times but the direction is the same as that of A . Multiplying a vector A by a negative number  gives
  
a vector B whose direction is opposite to the direction of A and whose magnitude is – times | A | .

  1
The division of vector A by a non-zero scalar m is defined as multiplication of A by .
m
 
At here A and B are co-linear vector

Corporate Head Office : Motion Education Pvt. Ltd., 394 - Rajeev Gandhi Nagar, Kota-5 (Raj.)
VECTOR & CALCULUS Page # 49

  
Ex.2 A physical quantity (m = 3kg) is multiplied by a vector a such that F = ma . Find the magnitude

and direction of F if

(i) a = 3m/s2 East wards

(ii) a = –4 m/s2 North wards
 
Sol. (i) F  ma  3  3 ms 2 East wards
= 9 N East wards
 
(ii) F  ma  3  ( 4) N North wards
= –12 N North wards
= 12 N South wards

4. LAWS OF ADDITION AND SUBTRACTION OF VECTORS :


4.1 Triangle rule of addition : Steps for additing two vector representing same physical quantity by
triangle law.
(i) Keep vectors s.t. tail of one vector coincides with head of other.
(ii) Join tail of first to head of the other by a line with arrow at head of the second.
(iii) This new vector is the sum of two vectors. (also called reultant)

D
D

CD AD
C
(i) (ii) (iii) AB  CD  AD
A B A C
AB B

Take example here.


Q. A boy moves 4 m south and then 5 m in direction 37° E of N. Find resultant displacement.

4.2 Polygon Law of addition :


This law is used for adding more than two vectors. This is extension of triangle law of addition. We
keep on arranging vectors s.t. tail of next vector lies on head of former.
When we connect the tail of first vector to head of last we get resultant of all the vectors.


d 
 
a  c d

d
b 
b

c
P  a 

 
c  c
 b
a 


b  b
a

a

        
   
Note : P  (a  b)  c  d  (c  a  d)  d [Associative Law]

: 0744-2209671, 08003899588 | url : www.motioniitjee.com, : info@motioniitjee.com


Page # 50 VECTOR & CALCULUS

4.3 Parallelogram law of addition :


Steps :
(i) Keep two vectors such that there tails coincide.
(ii) Draw parallel vectors to both of them considering both of them as sides of a parallelogram.
(iii) Then the diagonal drawn from the point where tails coincide represents the sum of two vectors,
with its tail at point of coincidence of the two vectors.


b

a

D a
   C
b b 
(i) (ii) (iii)
b  
  b AC  a  b
a a A 
a B

       
Note : AC  a  b and AC  b  a thus a  b  b  a [Cummutative Law]
Note : Angle between 2 vectors is the angle between their positive directions.
 
Suppose angle between these two vectors is , and | a | a, | b | b
(AD)2 = (AE)2 + (DE)2 
C a
  D
= (AB + BE)2 + (DE)2  b
b a
= (a + b cos )2 + (b sin )2   
A 
= a + b cos  + 2ab cos  + b sin 
2 2 2 2 2 a B E
= a2 + b2 + 2ab cos 

Thus,AD = a 2  b 2  2ab cos 


 
or | a  b | a 2  b 2  2ab cos 
angle  with vector a is

DE b sin 
tan  = =
AE (a  b cos )

Important points :
 To a vector, only a vector of same type can be added that represents the same physical quantity and
the resultant
is also a vector of the same type.

 As R = [A2 + B2 + 2AB cos]1/2 so R will be maximum when, cos  = max = 1,


i.e.,  = 0º, i.e. vectors are like or parallel and Rmax = A + B.
 
 | A || B | and angle between them  then R = 2A cos  / 2
 
 | A || B | and angle between them  – then R = 2A sin  / 2

 The resultant will be minimum if, cos  = min = – 1, i.e.,  = 180º, i.e. vectors are antiparallel and Rmin = A – B.

 If the vectors A and B are orthogonal, i.e.,  = 90º, R  A 2  B 2

Corporate Head Office : Motion Education Pvt. Ltd., 394 - Rajeev Gandhi Nagar, Kota-5 (Raj.)
VECTOR & CALCULUS Page # 51

 As previously mentioned that the resultant of two vectors can have any value from (A – B) to (A + B)
depending on the angle between them and the magnitude of resultant decreases as  increases 0º to
180º.

 Minimum number of unequal coplanar vectors whose sum can be zero is three.

 The resultant of three non-coplanar vectors can never be zero, or minimum number of non coplanar
vectors whose sum can be zero is four.

5. SUBTRACTION OF VECTOR :
 
Negative of a vector say – A is a vector of the same magnitude as vector A but pointing in a

direction opposite to that of A .
       
Thus, A – B can be written as A  (– B ) or A – B is really the vector addition of A and – B .

A

–A

   
Suppose angle between two vectors A and B is . Then angle between A and – B will be 180° –  as
shown in figure.

B 180   
 A

 
A –B   
S  A– B
(a)
(b)
  
Magnitude of S  A – B will be thus given by
 
S = | A– B | = A 2  B 2  2AB cos(180 – )

or S= A 2  B 2 – 2AB cos  ...(i)


For direction of S we will either calculate angle  or , where,

B sin(180 – ) B sin 
tan = = ...(ii)
A  B cos(180 – ) A – B cos 

A sin(180 – ) A sin 
or tan  = = ...(iii)
B  A cos(180 – ) B – A cos 
Ex.3 Two vectors of 10 units & 5 units make an angle of 120° with each other. Find the magnitude &
angle of resultant with vector of 10 unit magnitude.
 
Sol. | a  b | a 2  b 2  2ab cos   100  25  2  10  5(–1/ 2) = 5 3

5 sin 120 5 3 5 3 1
tan        = 30°
10  5 cos120 20 – 5 5  3 3
[Here shows what is angle between both vectors = 120° and not 60°]

: 0744-2209671, 08003899588 | url : www.motioniitjee.com, : info@motioniitjee.com


Page # 52 VECTOR & CALCULUS

   
Note : A – B or B – A can also be found by making triangles as shown in figure. (a) and (b)
   
B B B –A
 
A –B
Or

A 
(a) (b) A

Ex.4 Two vectors of equal magnitude 2 are at an angle of 60° to each other find magnitude of their
sum & difference.
 
Sol. | a  b | 22  22  2  2  2 cos 60  4  4  4  2 3


b

60°

a

 
| a  b | 2 2  2 2  2  2  2 cos 120  4  4  4  2


b
60° 
a
 120°
–b

   
Ex.5 Find A  B and A– B in the diagram shown in figure. Given A = 4 units and B = 3 units.

B

  60

A
Sol. Addition :

R= A 2  B 2  2AB cos 

= 16  9  2  4  3 cos 60 = 37 units


B sin  3 sin 60
tan  = = = 0.472
A  B cos  4  3 cos 60
  = tan–1(0.472) = 25.3°
  
Thus, resultant of A and B is 37 units at angle 25.3° from A in the direction shown in figure.

Corporate Head Office : Motion Education Pvt. Ltd., 394 - Rajeev Gandhi Nagar, Kota-5 (Raj.)
VECTOR & CALCULUS Page # 53

Subtraction : S = A 2  B 2 – 2AB cos   


 A
= 16  9 – 2  4  3 cos 60 = 13 units 
–B
B sin 
and tan  =
A – B cos 
  
3 sin 60 S  A– B
= = 1.04
4 – 3 cos 60
  = tan–1 (1.04) = 46.1°
  
Thus, A – B is 13 units at 46.1° from A in the direction shown in figure.

6. UNIT VECTOR AND ZERO VECTOR



Unit vector is a vector which has a unit magnitude and points in a particular direction. Any vector ( A )

can be written as the product of unit vector ( Â ) in that direction and magnitude of the given vector..

 A
A  A  or  
A
A unit vector has no dimensions and unit. Unit vectors along the positive x-, y-and z-axes of a
rectangular coordinate system are denoted by î, ĵ and k̂ respectively such that | î | = | ĵ | = | k̂ | = 1.
y

ˆj
î x

z
A vector of zero magnitude is called a zero or a null vector. Its direction is arbitrary.

Ex.6 A unit vector along East is defined as î . A force of 105 dynes acts west wards. Represent the
force in terms of î .

Sol. F  10 5 î dynes
7. RESOLUTION OF VECTORS
  
If a and b be any two non-zero vectors in a plane with different directions and A be another vector
 
in the same plane. A can be expressed as a sum of two vectors-one obtained by multiplying a by a

real number and the other obtained by multiplying b by another real number..
  
A  a  b (where  and  are real numbers)  
 b A
We say that A has been resolved into two component vectors namely 
  b

A  a  b (where  and  are real number)  
 a a
We say that A has been resolved into two component vectors namely
 
a and b
   
 a and  b along a and b respectively. Hence one can resolve a given vector into two component
vectors along a set of two vectors – all the three lie in the same plane.

: 0744-2209671, 08003899588 | url : www.motioniitjee.com, : info@motioniitjee.com


Page # 54 VECTOR & CALCULUS

7.1 Resolution along rectangular component : y


It is convenient to resolve a general vector along axes of a
rectangular coordinate system using vectors of unit ĵ
magnitude, which we call as unit vectors. î, ĵ, k̂ are unit
O x
along x, y and z-axis as shown in figure below :


z
7.2 Resolution in two Dimension

Consider a vector A that lies in xy plane as shown in figure,
   y
A  A1  A 2
  

A 1  A x î, A 2  A y ĵ A  A x î  A y ĵ
A 2  A y ĵ 
 A
Ay = A sin 
The quantities Ax and Ay are called x-and y-components


of the vector A .  x
A1  A x î
Ax is itself not a vector but A x î is a vector and so it A y ĵ . A x  A cos 

Ax = A cos  and Ay = A sin 


It's clear from above equation that a component of a vector can be positive, negative or zero

depending on the value of . A vector A can be specified in a plane by two ways :

(a) its magnitude A and the direction  it makes with the x-axis; or

–1
Ay
(b) its components Ax and Ay A= A 2x  A 2y ,  = tan
Ax

Note : If A = Ax  Ay = 0 and if A = Ay  Ax = 0 i.e.,


y Bx
components of a vector perpendicular to itself is always Ax By
zero. The rectangular components of each vector and those 
    B
of the sum C  A  B are shown in figure. We saw that Ay A  Cy
C

   Cx
C  A  B is equivalent to both
Cx = Ax + Bx x
(a)
and Cy = Ay + By
Refer figure (b)

y
Vector R has been resolved in two axes x and y not
perpendicular to each other. Applying sine law in the triangle Ry
Ry R
shown, we have

R R Ry 
 x  
sin[180 – (   )] sin  sin  x
O Rx
(b)
R sin  R sin 
or Rx = and Ry =
sin(   ) sin(   )
If  +  = 90°, Rx = R sin  and Ry = R sin 

Corporate Head Office : Motion Education Pvt. Ltd., 394 - Rajeev Gandhi Nagar, Kota-5 (Raj.)
VECTOR & CALCULUS Page # 55

Ex.7 Resolve the vector A  A x î  A y ĵ along an perpendicular to the line which make angle 60° with x-
axis.
y
Ay Aycos30°

line y (Aycos30° + Axcos60°)

Axcos60°
Aysin30° 30°
60° x
Sol. Ax x 
(Axsin60° – Aysin30°)

Axsin60°
so the component along line = |Aycos30° + Axcos60°|
and perpendicular to line = |Axsin60° – Aysin30°|

Ex.8 Resolve a weight of 10 N in two directions which are parallel and perpendicular to a slope in-
clined at 30° to the horizontal
Sol. Component perpendicular to the plane
W  W cos 30
W||
3 30° W
= (10) = 5 3 N Ans.
2
and component parallel to the plane W=10 N
30°

 1
W|| =W sin 30° = (10)   = 5 N
2

Ex.9 Resolve horizontally and vertically a force F = 8 N which makes an angle of 45° with the hori-
zontal.

Sol. Horizontal component of 


F is Fv
 1 
FH = F cos 45° = (8)   = 4 2 N
 2 
F

and vertical component of F is
45°
FH
 1 
Fv = F sin 45° = (8)   = 4 2 N Ans.
 2
8. PROCEDURE TO SOLVE THE VECTOR EQUATION
  
A  B  C ...(1)
(a) There are 6 variables in this equation which are following :

(1) Magnitude of A and its direction

(2) Magnitude of B and its direction

(3) Magnitude of C and its direction.

: 0744-2209671, 08003899588 | url : www.motioniitjee.com, : info@motioniitjee.com


Page # 56 VECTOR & CALCULUS

(b) We can solve this equation if we know the value of 4 variables [Note : two of them must be directions]
(c) If we know the two direction of any two vectors then we will put them on the same side and other on
the different side.
For example
  
If we know the directions of A and B and C' s direction is unknown then we make equation as
follows:-
  
C  A –B
(d) Then we make vector diagram according to the equation and resolve the vectors to know the
unknown values.

Ex.10 Find the net displacement of a particle from its starting point if it undergoes two sucessive
 
displacement given by S1  20m , 37° North of West, S2  50m , 53° North of East

N S
S1 N y
50
S1 20 W x' x
 =
Sol. 37° 53°  E
W E
S y'
S

  
S  S1  S2
Sx = S1 x + S2 x
Sy = S1 y + S2 y

= – 20 cos 37° + 50 cos 53° = 20 sin 37° + 50 sin 53°


= 14 = 52

S  S2x  S 2y = (14) 2  (52) 2  53.85


Angle from west - east axis (x- axis)
S y 52 26
tan = S  14  7
x

 26 
 = tan–1  
7
   
Ex.11 Find magnitude of B and direction of A . If B makes angle 37° and C makes 53° with x axis and
    
A has magnitude equal to 10 and C has 5. (given A  B  C  0 )

   C
Sol. –A  C  B y'
 
A  A x i  A y j 53° B
37°
 x

 –A  –A i  –A j  x'
x y

Ax = –(Bcos 37° + Ccos 53°)


y
Ay = –(B sin 37° + C cos 53°)

| A|2  A x 2  A y 2
2 2
 4 3  3 4
A2 =  B   C     B   C  
 5 5  5 5

Corporate Head Office : Motion Education Pvt. Ltd., 394 - Rajeev Gandhi Nagar, Kota-5 (Raj.)
VECTOR & CALCULUS Page # 57

2 2
 4B   3B 
10 = 
2  3    4
 5   5 

16 2 9 2  3  4 4  3
 100 = 25 B  25 B  25  2 5  5  B

2 48
 B  B – 75  0
5
B = 5 (magnitude can not be negative)
& Angle made by A

20
 Ax = –( + 3) = –12
5

15
Ay = –( + 4) = –7
5

Ay –7
tan  = A  –12
x

 = 180° + 25° = 205°

Ex.12 Find the magnitude of F1 and F2. If F1, F2 make angle 30° and 45° with F3 and magnitude of F3 is
  
10 N. (given F1  F2 = F3 )

Sol. |F3 |  F1 cos 30F2 cos 45 F3
& F2 sin 45° = F1 sin 30°
F1
F2
3F1 F2 F2 F1
 10 =  ,  45° 30°
2 2 2 2

20 20 2
 F1 = & F2 =
3 1 3 1

9. SHORT - METHOD

A   
C  A B


Asin  
B
Bsin

   
If their are two vectors A and B and their resultent make an anlge  with A and  with B .
then A sin  =  sin 
 
Means component of A perpendicular to resultant is equal in magnitude to the component of B
perpendicular to resultant.

: 0744-2209671, 08003899588 | url : www.motioniitjee.com, : info@motioniitjee.com


Page # 58 VECTOR & CALCULUS

  
Ex.13 If two vectors A and B make angle 30° and 45° with their B

resultant and B has magnitude equal to 10, then find

magnitude of A . 60°
30° 
So B sin 60° = A sin 30° Bsin60° A
 10 sin 60° = A sin 30°
A sin 30
A = 10 3

  
Ex.14 If A and B have angle between them equals to 60° and their resultant make, angle 45° with A
 
and A have magnitude equal to 10. Then Find magnitude of B .
Sol. here  = 45° and  = 60° – 45° = 15°    
B C  A B
so A sin  = B sin 
10 sin 45° = B sin 45°

10
So B = sin15 60°
2
45° 
A
10 1 – cos(2  15) 5
=  2– 3
2 2 2

10. ADDITION AND SUBTRACTION IN COMPONENT FORM :


Suppose there are two vectors in component form. Then the addition and subtraction between these
two are

A  A x î  A y ĵ  A zk̂

B  B x î  B y ĵ  B zk̂

A  B  ( A x  B x ) î  ( A y  B y ) ĵ  ( A z  Bz ) k̂

Also if we are having a third vector present in component form and this vector is added or subtracted
from the addition or subtraction of above two vectors then

C  C x î  C y ĵ  C z k̂

A  B  C  ( A x  B x  C x ) î  ( A y  B y  C y ) ĵ  ( A z  B z  C z ) k̂
Note : Modulus of vector A is given by

| A | A 2x  A 2y  A 2z

 
Ex.15 Obtain the magnitude of 2 A– 3 B if

 
A  î  ĵ – 2k̂ and B  2î – ĵ  k̂

 
Sol. 2 A – 3 B  2( î  ĵ – 2k̂ ) – 3(2 î – ˆj  k̂ )

 
 Magnitude of 2 A – 3 B  (–4)2  (5)2  (–7)2

= 16  25  49  90 Ans.

Corporate Head Office : Motion Education Pvt. Ltd., 394 - Rajeev Gandhi Nagar, Kota-5 (Raj.)
VECTOR & CALCULUS Page # 59

     
Ex.16 Find A  B and A – B if A make angle 37° with positive x-axis and B make angle 53° with negative

x-axis as shown and magnitude of A is 5 and of B is 10.
 
B A

Sol. 53° 37°

 
for A  B

3 = A sin 37° 8 + 3 = 11
8 = B sin 53°
53°
+ =
–B cos 53° –(6–4) = –2
A cos 37° = 4 = –6

so the magnitude of resultant will be = 112  (–2) 2 = 5 5

 11
and have angle  = tan –1  from negative x - axis towards up
 2
 
for A – B

Bcos53°=6 (6+4) = 10
Asin37°=3 + =
53°
–8= –(8–3)
Acos37=4 –Bsin53°
 = –5
–B

So the magnitude of resultant will be

= 10 2  (–5)2  5 5

5
and have angle   tan –1   from positive x-axis towards down.
 10 

11. MULTIPLICATION OF VECTORS (The Scalar and vector products) :

11.1 Scalar Product


  
The scalar product or dot product of any two vector A and B , B
   
denoted as A . B (read A dot B ) is defined as the product of 
their magnitude with cosine of angle between them. Thus,

  A
A.B  AB cos  (here  is the angle between the vectos)

: 0744-2209671, 08003899588 | url : www.motioniitjee.com, : info@motioniitjee.com


Page # 60 VECTOR & CALCULUS

Properties :
• It is always a scalar which is positive if angle between the vectors is acute (i.e.< 90°) and negative if
angle between them is obtuse (i.e., 90° < q  180°)
   
• It is commutative i.e. A.B  B.A
      
• It is distributive, i.e. A.(B  C)  A.B  A.C
 
  
–1 A.B

• As by definition A . B = AB cos . The angle between the vectors  = cos  
 AB 
 
• A.B  A(B cos )  B( A cos )
 
Geometrically, B cos  is the projection of B onto A and vice versa

 
B B


  B
 
A B cos  A
 
  A.B   
Component of B along A = B cos  = = Â .B (Projection of B on A )
A


os
 

Ac
  A.B   
Component of A along B = A cos  = = A.B̂ (Projection of A on B )
B 

A

• Scalar product of two vectors will be maximum when cos  = max = 1, i.e.,  = 0°,
 
i.e., vectors are parallel  ( A.B )max  AB
• If the scalar product of two non-zero vectors vanishes then the vectors are perpendicular.
• The scalar product of a vector by itself is termed as self dot product and is given by
    
( A )2  A.A = AA cos  = A2  A  A.A
• In case of unit vector n̂ ,

n̂.n̂ = 1 × 1 × cos 0° = 1  n̂.n̂  î.î  ˆj. ĵ  k̂.k̂  1

In case of orthogonal unit vectors, î , ĵ and k̂ ; î.ˆj  ĵ.k̂  k̂.î  0


 
A.B  ( î A x  ˆjA y  k̂A z ) . ( î B x  ĵ B y  k̂Bz ) = [AxBx + AyBy + AzBz]

 
Ex.17 If the vectors P  a î  aĵ  3k̂ and Q  a î – 2 ĵ – k̂ are perpendicular to each other. Find the value of
a?
 
Sol. If vectors P and Q are perpendicular
 
 P.Q  0  (a î  a ĵ  3k̂ ).(a î – 2 ĵ – k̂ )  0
 a2 – 2a – 3 = 0  a2 – 3a + a – 3 = 0
 a(a – 3) + 1 (a – 3 )  a = –1, 3

Corporate Head Office : Motion Education Pvt. Ltd., 394 - Rajeev Gandhi Nagar, Kota-5 (Raj.)
VECTOR & CALCULUS Page # 61

Ex.18 Find the component of 3 î  4 ĵ along î  ĵ ?


 
  A.B
Sol. Component of A along B is given by hence required component
B
(3 î  4ˆj).( î  ĵ) 7
= 
2 2
 
Ex.19 Find angle between A  3î  4 ĵ and B  12î  5 ĵ ?
 
A.B (3 î  4 ĵ ).(12 î  5 ĵ )
Sol. We have cos  = AB 
3 2  4 2 12 2  5 2
36  20 56  56 
cos  = =  = cos–1  65 
5  13 65
 
Ex.20 (i) For what value of m the vector A  2 î  3 ĵ – 6k̂ is perpendicular to B  3î – mĵ  6k̂

(ii) Find the component of vector A  2î  3 ĵ along the direction of î  ĵ ?
5
Sol. (i) m = – 10 (ii)
2
Important Note :
Components of b along and perpendicular to a.

Let OA . OB represent two (non-zero) given vectors a, b respectively. Draw BM perpendicular to OA

From OMB, OB = OM + MB

 b = OM + MB B

Thus OM and MB are components of b along a


and perpendicular to a. b
Now OM = ( OM ) â = (OB cos ) â

= |b| cos â = |b| . a . b / |a| |b| . â  A


O
a M
= a . b / |a| . a/|a| = (a . b) a / |a|2
= (a . b) a / a2

MB = b – OM = b – (a . b / |a| ) . a
2

Hence, components of b along a perpendicular to a are.


(a . b/ |a|2) a and b – (a . b / |a|2) a respectively.

Ex.21 The velocity of a particle is given by v  3 î  2 ĵ  3k̂ . Find the vector component of its velocity parallel

to the line l  î  ĵ  k̂ .
 
Sol. Component of v along l
 
ˆ v.l ˆ v. l 
 v cos l  v l  2 l
vl l

(3 î  2 ĵ  3k̂ ) ( î  ĵ  k̂ ) 4
 2
 ( î  ĵ  k̂ )
| î  ĵ  k̂ | 3

: 0744-2209671, 08003899588 | url : www.motioniitjee.com, : info@motioniitjee.com


Page # 62 VECTOR & CALCULUS

11.2 Vector product


 
The vector product or cross product of any two vectors A and B , denoted as
   
A  B (read A cross B ) is defined as :
 
A  B  AB sin n̂
Here  is the angle between the vectors and the direction n̂ is given by the right - hand - thumb rule.

Right - Hand - Thumb Rule :   

  V  A B
To find the direction of n̂ , draw the two vectors A and B with both the 

tails coinciding. Now place your stretched right palm perpendicular to the n B
  
plane of A and B in such a way that the fingers are along the vector A
 
and when the fingers are closed they go towards B . The direction of the

thumb gives the direction of n̂ . A
Properties :
• Vector product of two vectors is always a vector perpendicular to the plane containing the two
vectors i.e. orthogonal to both the vectors 
  
A and B , though the vectors A and B may or may not be
orthogonal.

Vector product of two vectors is not commutative i.e. 


  
• A B  B A
   
But | A B || B A | AB sin 
• The vector product is distributive when the order of the vectors is strictly maintained i.e.
      
A  (B  C)  A  B  A  C
• The magnitude of vector product of two vectors will be maximum when sin  = max = 1. i.e.  = 90°
 
| A  B |max  AB
• The magnitude of vector product of two non-zero vectors will be minimum when |sin| = minimum = 0,
 
i.e.,  = 0° or 180° and | A  B |min  0 i.e., if the vector product of two non-zero vectors vanishes, the
vectors are collinear.
• The self cross product i.e. product of a vector by itself vanishes i.e. is a null vector.
  
A  A  AA sin 0n̂  0
 
• In case of unit vector n̂ , n̂  n̂  0  î  î  ĵ  ĵ  k̂  k̂  0

• In case of orthogonal unit vectors î, ĵ and k̂ in accordance with right-hand-thumb-rule,

î  ĵ  k̂ ĵ  k̂  î k̂  î  ĵ

j j

k
i
i

k
(A) (B)

Corporate Head Office : Motion Education Pvt. Ltd., 394 - Rajeev Gandhi Nagar, Kota-5 (Raj.)
VECTOR & CALCULUS Page # 63

î ĵ k̂
 
• In terms of components, A  B  A x A y A z
Bx By Bz
 
A  B  î ( A yB z – A zB y )  ĵ( A zB x – A xB z )  k̂( A xB y – A yB x )
   
Ex.22 A is East wards and B is downwards. Find the direction of A × B ?
 
Sol. Applying right hand thumb rule we find that A  B is along North.
     
Ex.23 If A.B | A  B | , find angle between A and B
   
Sol. A.B | A  B | AB cos  = AB sin  tan  = 1   = 45°

 
  AB  
Ex.24 A  B  AB sinn̂  n̂  here n̂ is perpendicular to both A and B
AB sin
   
Ex.25 Find A  B if A  î – 2 ĵ  4k̂ and B  2î – ĵ  2k̂

î ĵ k̂
 
Sol. A  B  1 – 2 4 = î (–4 – (–4)) – ĵ(2 – 12)  k̂(–1 – (–6)) = 10 ĵ  5k̂
3 –1 2

   
Ex.26 (i) A is North-East and B is down wards, find the direction of A  B
   
(ii) Find B × A if A  3î – 2 ĵ  6k̂ and B  î – ĵ  k̂
Ans. (i) North - West. (ii) – 4 î – 3 ĵ  k̂

12. POSITION VECTOR :


Positin vector for a point is vector for which tail is origin & head is the given point itself.
Position vector of a point defines the position of the point w.r.t. the origin.
y

P( x , y )

r

x
O

OP  r

r  x î  yĵ
13. DISPLACEMENT VECTOR :
Change in position vector of particle is Q(x2, y2)
known as displacement vector. 
 r2
OP  r1  x1 î  y1 ĵ
  P( x1, y1 )
OQ  r 2  x 2 î  y 2 ˆj r1
 
PQ  r 2  r1  ( x 2  x1 ) î  ( y 2 – y1 ) ĵ
T h us w e c an re p re s e nt a v e c t o r i n s p ac e s t art i ng fro m ( x 1 , y 1 ) & e nd i n g at
(x 2, y 2) as (x 2 – x 1) î + (y2 – y1) ĵ

: 0744-2209671, 08003899588 | url : www.motioniitjee.com, : info@motioniitjee.com


Page # 64 VECTOR & CALCULUS

CALCULUS
14. CONSTANTS : They are fixed real number which value does not change
Ex. 3, e, a, – 1, etc.

15. VARIABLE :
Somthing that is likly to vary, somthing that is subject to variation.
or
A quantity that can assume any of a set of value.
Types of variables.
(i) Independent variables : Indepedent variables is typically the variable being manipulated or change
(ii) dependent variables : The dependent variables is the object result of the independent variable
being manipulated.

Ex. y = x2
here y is dependent variable and x is independent variable

16. FUNCTION :
Function is a rule of relationship between two variables in which one is assumed to be dependent and
the other independent variable.
The temperatures at which water boils depends on the elevation above sea level (the boiling point
drops as you ascend). Here elevation above sea level is the independent & temperature is the dependent
variable.
The interest paid on a cash investment depends on the length of time the investment is held. Here time
is the independent and interest is the dependent variable.
In each case, the value of one variable quantity (dependent variable), which we might call y, depends
on the value of another variable quantity (independent variable), which we might call x. Since the
value of y is completely determined by the value of x, we say that y is a function of x and represent it
mathematically as y = f(x).
x f(x)
Input f Output
(Domain) (Range)
all possible values of independent variables (x) are called domain of function.
all possible values of dependent variable (y) are called Range of fucntion.
Think of function f as a kind machine that produces an output value f(x) in its range whenever we feed
it an input value x from its domain (figure).
When we study circles, we usualy call the area A and the radius r. Since area depends on radius, we
say that A is a function of r, A = f(r). The eauation A = r2 is a rule that tells how to calculate a unique
(single) output value of A for each possible input value of the radius r.
A = f(x) = r2. (Here the rule of relationship which describes the function may be described as square
& multiply by )
if r=1 A=
if r=2 A = 4
if r=3 A = 9
The set of all possible input values for the radius is called the domain of the function. The set of all
output values of the area is the range of the function.

Corporate Head Office : Motion Education Pvt. Ltd., 394 - Rajeev Gandhi Nagar, Kota-5 (Raj.)
VECTOR & CALCULUS Page # 65

We usually denote functions in one of the two ways :


1. By giving a formula such as y = x2 that uses a dependent variable y to denote the value of the
fucntion.
2. By giving a formula such as f(x) =x2 that defines a functions symbols f to name the function.
Strictly speaking, we should call the function f and not f(x).
y = sinx. Here the function is y since, x is the independent variable.
4
Ex.27 The volume V of ball (solid sphere) of radius r is given by the function V(r) = (r )3
3
The volume of a ball of radius 3m is ?
4
Sol. V(3) = (3)3 = 36 m3.
3

Ex.28 Suppose that the function F is defined for all real numbers r by the formula.
F(r) = 2 (r – 1) + 3.
Evaluate F at the input values 0, 2 x + 2, and F(2).
Sol. In each case we substitute the given input value for r into the formula for F:
F(0) = 2(0 – 1) + 3 = – 2 + 3 = 1
F(2) = 2(2 – 1) + 3 = 2 + 3 =5
F(x + 2) = 2 (x + 2 – 1) + 3 = 2x + 5
F(F(2)) = F(5) = 2(5 – 1) + 3 = 11

Ex.29 function f(x) is defined as


f(x) = x2 + 3, Find
f(0), f(1), f(x2), f(x + 1) and f(f(1))
Sol. f(0) = 02 + 3 =3
f(1) = 12 + 3 =4
f(x2) = (x2)2 + 3 = x4 + 4
f(x + 1) = (x + 1)2 + 3 = x2 + 2x + 4
f(f(1)) = f(4) = 42 + 3 = 19

17. DIFFERENTIATION
Finite difference :
The finite difference between two values of a physical is represented by  notation.
For example :
Difference in two values of y is written as y as given in the table below.

y2 100 100 100

y1 50 99 99.5

y  y 2 – y 1 50 1 0.5

Infinitey small difference :


The infinitely small difference means very-very small difference. And this difference is represented by
'd' notation insted of ''.
For example infinitely small difference in the values of y is written as 'dy'
if y2 = 100 and y1 = 99.9999999999999.....
then dy = 0.00000000000000..........00001

: 0744-2209671, 08003899588 | url : www.motioniitjee.com, : info@motioniitjee.com


Page # 66 VECTOR & CALCULUS

Definition of differentiation
Another name of differentiation is derivative. Suppose y is a function of x or y = f(x)
Differentiation of y with respect to x is denoted by sumbols f (x)

dy
where f (x) = ; dx is very small change in x and dy is corresponding very small change in y..
dx
Notation : There are many ways to denote the derivative of function y = f(x), the most common
notations are these :
Nice and brief and does not name the
y "y prime"
independent variable
dy
" dy by dx" Names the variables and uses d for derivative
dx
df
dx
" df by dx" Emphasizes the function's name

d
f( x )
Emphasizes the idea that differentiation is an
dx
" d by dx of f "
operation performed on f.
Dx f " dx of f " A common operator notation
 One of Newton's notations, now common for time
y
" y dot"
derivative i.e. dy/dt

Average rates of change :


Given an arbitrary function y = f(x) we calculate the average rate of change of y with respect to x
over the interval (x, x+ x) by dividing the change in value of y, i.e., y = f(x+ x) – f(x), by length of
interval x over which the change occurred.
The average rate of change of y with respect to x over the interval [x, x + x]

y f ( x  x ) – f ( x )
 
x x
Geometrically
Q
y  y
y QR
 = tan  = Slope of the line PQ
x PR
y
y
In triangle QPR tan  = P
x y 
R
x
therefore we can say that average rate of change of y with
x x  x
respect to x is equal to slope of the line joining P & Q.

The derivative of a fucntion

We know that Average rate of change of y w.r.t x is -

y f ( x  x ) – f ( x )

x x
If the limit of this ratio exists as x  0, then it is called the derivative of given function f(x) and is
denoted as

dy f ( x  x ) – f ( x )
f ' ( x)   lim
dx x 0 x

Corporate Head Office : Motion Education Pvt. Ltd., 394 - Rajeev Gandhi Nagar, Kota-5 (Raj.)
VECTOR & CALCULUS Page # 67

18. GEOMETRICAL MEANING OF DIFFERENTIATION :

The geometrical meaning of differentiation is very much useful in the analysis of graphs in physics. To
understand the geometrical meaning of derivatives we should have knowledge of secant and tangent
to a curve.

Secant and Tangent to a Curve

Secant : - A secant to a curve is a straight line, which intersects the curve at any two points.
y
q
Secant

x
Tangent :
A tangent is straight line, which touches the curve a particular point. Tangent is limiting case of secant
which intersects the curve at two overlapping point.

Q
In the figure - 1 shown, if value of x is gradually reduced y  y
then the point Q will move nearer to the point P. If the
process is continuously repeated (Figure-2) value of x
will be infinitely small and secant PQ to the given curve will y
become a tangent at point P.
y P  R
Therefore x

 y  dy x x  x
   tan 
x  0 x  dx
Figure-1

 dy 
we can say that differentiation of y with respect to x, i.e.  dx  is Q
y  y
equal to slope of the tangent at point P (x,y) Q

dy Q y
or tan =
dx y P  R
(From fig-1 the average rate change of y from x to x + x x

is identical with the slope of secant PQ) x x  x

Figure-2

Rule No. 1 Derivative Of A Constant


The first rule of differentiation is that the derivative of every constant function is zero.
d
If c is constant, then c0
dx
d  1
Ex.30
d
dx
(8)  0 ,    0 ,
dx  2 
d
dx
 3 0

: 0744-2209671, 08003899588 | url : www.motioniitjee.com, : info@motioniitjee.com


Page # 68 VECTOR & CALCULUS

Rule No.2 Power Rule


d n
If n is a real number, then x  nx n1
dx
To apply the power Rule, we subtract 1 from the original exponent (n) and multiply the result by n.

2 3 4
f x x x x …..
Ex.31 2 3
f' 1 2x 3x 4x …..

d  1  d 1 1 d  4  d 3 12
Ex.32 (i)   ( x )  ( 1)x 2   2 (ii)  34 ( x )  4 ( 3)x  4   4
dx  x  dx x dx  x  dx x
d 1/ 2 1 1
Ex.33 (a) ( x )  x 1/ 2 =
dx 2 2 x
Function defined for x  0 derivative defined only for x > 0
d 1/ 5 1 4 / 5
(b) (x ) = x
dx 5
Function defined for x  0 derivative not defined at x = 0

Rule No.3 The Constant Multiple Rule


d du
If u is a differentiable function of x, and c is a constant, then (cu)  c
dx dx

d
In particular, if n is a positive integer, then (cx n )  cn x n1
dx

Ex.34 The derivative formula


d
(3x 2 )  3 (2x)  6x
dx
says that if we rescale the graph of y = x2 by multiplying each y-coordinate by 3, then we
multiply the slope at each point by 3.

Ex.35 A useful special case


The derivative of the negative of a differentiable function is the negative of the function’s derivative.
Rule 3 with c = – 1 gives.
d d d d
( u)  ( 1.u)  1 . (u)   (u)
dx dx dx dx

Rule No.4 The Sum Rule


The derivative of the sum of two differentiable functions is the sum of their derivatives.
If u and v are differentiable functions of x, then their sum u + v is differentiable at every point where
u and v are both differentiable functions in their derivatives.

d d du dv du dv
(u  v )  [u  ( 1) v ]   ( 1)  
dx dx dx dx dx dx
The sum Rule also extends to sums of more than two functions, as long as there are only finite
functions in the sum. If u1, u2, ........ un are differentiable at x, then so if u1 + u2 + ....... + un, then
d du du du
(u1  u 2  ......  un )  1  2  ........  n
dx dx dx dx

Corporate Head Office : Motion Education Pvt. Ltd., 394 - Rajeev Gandhi Nagar, Kota-5 (Raj.)
VECTOR & CALCULUS Page # 69

4 2
Ex.36 (a) y = x4 + 12x (b) y = x3 + x – 5x + 1
3

dy d 4 d dy d 3 d 4 2 d d
 (x )  (12x )  x   x  (5 x )  (1)
dx dx dx dx dx dx  3  dx dx

4
= 4x3 + 12 = 3x2 + . 2x – 5 + 0
3
8
= 3x2 +
x5
3
Notice that we can differentiate any polynomial term by term, the way we differentiated the polynomials
in above example.

Rule No. 5 The Product Rule


d dv du
If u and v are differentiable at x, then if their product uv is considered, then (uv )  u v .
dx dx dx
The derivative of the product uv is u times the derivative of v plus v times the derivative of u. In prime
notation
(uv)’ = uv’ + vu’.
While the derivative of the sum of two functions is the sum of their derivatives, the derivative of the
product of two functions is not the product of their derivatives. For instance,

d d 2
(x . x)  ( x )  2x, while d ( x ). d ( x )  1.1  1 , which is wrong
dx dx dx dx

Ex.37 Find the derivatives of y = (x2 + 1) (x3 + 3)


Sol. Using the product Rule with u = x2 + 1 and v = x3 + 3, we find

d
[( x 2  1)( x 3  3)] = (x2 + 1) (3x2) + (x3 + 3) (2x)
dx
= 3x4 + 3x2 + 2x4 + 6x = 5x4 + 3x2 + 6x
Example can be done as well (perhaps better) by multiplying out the original expression for y and
differentiating the resulting polynomial. We now check :
y = (x2 + 1) (x3 + 3) = x5 + x3 + 3x2 + 3

dy
= 5x4 + 3x2 + 6x
dx
This is in agreement with our first calculation.
There are times, however, when the product Rule must be used. In the following examples. We have
only numerical values to work with.

Ex.38 Let y = uv be the product of the functions u and v. Find y’(2) if u(2) = 3, u’(2) = – 4, v(2) = 1, and
v’(2) = 2.
Sol. From the Product Rule, in the form
y’ = (uv)’ = uv’ + vu’,
we have y’(2) = u(2) v’(2) + v(2) u’(2)
= (3) (2) + (1) (–4) = 6 – 4 = 2

: 0744-2209671, 08003899588 | url : www.motioniitjee.com, : info@motioniitjee.com


Page # 70 VECTOR & CALCULUS

Rule No.6 The Quotient Rule


If u and v are differentiable at x, and v(x)  0, then the quotient u/v is differentiable at x,

du dv
v u
d u dx dx
and  
dx  v  v2

Just as the derivative of the product of two differentiable functions is not the product of their
derivatives, the derivative of the quotient of two functions is not the quotient of their derivatives.

t2  1
Ex.39 Find the derivative of y 
t2  1
Sol. We apply the Quotient Rule with u = t2 – 1 and v = t2 + 1

dy ( t 2  1) 2t  ( t 2  1). 2t  d  u  v(du / dt )  u(dv / dt ) 


  As    
dt ( t 2  1)2  dt v v2 

2t 3  2t  2t 3  2t 4t
 
2
( t  1) 2
(t  1)2
2

Rule No. 7 Derivative Of Sine Function


d
(sin x )  cos x
dx

dy d
Ex.40 (a) y = x2 – sin x :  2x  (sin x ) = 2x – cos x Difference Rule
dx dx

dy d
(b) y = x2 sin x :  x2 (sin x )  2x sin x Product Rule
dx dx

= x2cosx + 2x sinx
d
x. (sin x)  sin x .1
sin x dy dx
(c) y  :  Quotient Rule
x dx x2
x cos x  sin x

x2

Rules No.8 Derivative Of Cosine Function


d
(cos x )   sin x
dx
Ex.41 (a) y = 5x + cos x Sum Rule
dy d d
 (5 x )  (cos x ) = 5 – sin x
dx dx dx
(b) y = sin x cos x

dy d d
 sin x (cos x )  cos x (sin x ) Product Rule
dx dx dx
= sin x(– sin x) + cos x (cos x)
= cos2 x – sin2 x = cos 2x

Corporate Head Office : Motion Education Pvt. Ltd., 394 - Rajeev Gandhi Nagar, Kota-5 (Raj.)
VECTOR & CALCULUS Page # 71

Rule No. 9 Derivatives Of Other Trigonometric Functions


Because sin x and cos x are differentiable functions of x, the related functions
sin x 1
tan x  ; sec x 
cos x cos x
cos x 1
cot x  ; cos ec x 
sin x sin x
are differentiable at every value of x at which they are defined. There derivatives, Calculated from the
Quotient Rule, are given by the following formulas.
d d
(tan x )  sec 2 x ; (sec x )  sec x tan x
dx dx

d d
(cot x )   cos ec 2 x ; (cos ec x )   cos ec x cot x
dx dx
Ex.42 Find dy / dx if y = tan x.
d d
cos x (sin x )  sin x (cos x)
d d  sin x  dx dx
Sol. (tan x)   
dx dx  cos x  cos 2 x

cos x cos x  sin x(  sin x ) cos 2 x  sin 2 x 1


 2
   sec 2 x
cos x cos 2 x cos 2 x

d d
Ex.43 (a) (3x + cot x) = 3 + (cot x) = 3 – cosec2 x
dx dx

d  2  d d
(b) dx  sin x   dx (2 cosec x )  2 dx (cosec x )
 
= 2(– cosec x cot x) = – 2 cosec x cot x

Rule No. 10 Derivative Of Logrithm And Exponential Functions


d 1 d x
(log e x )  , (e )  e x
dx x dx
Ex.44 y = ex . loge (x)

dy d x d dy ex
 (e ). log( x )  [loge ( x )] e x   e x . loge ( x ) 
dx dx dx dx x

Rule No. 11 Chain Rule Or ‘Outside Inside’ Rule


dy dy du
 .
dx du dx
It sometime helps to think about the Chain Rule the following way. If y = f (g(x)),
dy
= f’[g(x)] . g’(x)
dx
In words : To find dy/dx, differentiate the “outside” function f and leave the “inside” g(x) alone; then
multiply by the derivative of the inside.
We now know how to differntiate sin x and x 2 – 4, but how do we differentiate a composite like
sin(x2 – 4)?
The answer is, with the Chain Rule, which says that the derivative of the composite of two differentiable
functions is the product of their derivatives evaluated at appropriate points. The Chain Rule is probably
the most widely used differentiation rule in mathematics. This section describes the rule and how to
use it. We begin with examples.

: 0744-2209671, 08003899588 | url : www.motioniitjee.com, : info@motioniitjee.com


Page # 72 VECTOR & CALCULUS

Ex.45 The function y = 6x – 10 = 2(3x – 5) is the composite of the functions y = 2u and u = 3x – 5. How
are the derivatives of these three functions related ?

dy dy du
Sol. We have  6,  2, 3
dx du dx
dy dy du
Since 6 = 2 × 3  .
dx du dx
dy dy du
Is it an accident that  . ?
dx du dx
If we think of the derivative as a rate of change, our intution allows us to see that this relationship is
reasonable. For y = f(u) and u = g(x), if y changes twice as fast as u and u changes three times as fast
as x, then we expect y to change six times as fast as x.

Ex.46 Let us try this again on another function.


y = 9x4 + 6x2 + 1 = (3x2 + 1)2
is the composite y = u2 and u = 3x2 + 1. Calculating derivatives. We see that

dy du
.  2u.6x = 2 (3x2 + 1). 6x = 36x3 + 12 x
du dx

dy d
and  (9 x 4  6x 2  1) = 36 x3 + 12 x
dx dx

dy du dy
Once again, . 
du dx dx
The derivative of the composite function f(g(x)) at x is the derivative of f at g(x) times the derivative of
g at x.

Ex.47 Find the derivation of y  x 2  1

Sol. Here y = f(g(x)), where f(u) = u and u = g(x) = x2 + 1. Since the derivatives of f and g are

1
f  (u) = and g(x) = 2x,
2 u
the Chain Rule gives

dy d 1 1 x
 f (g( x )) = f (g(x)).g(x) = .g(x) = . (2x) =
dx dx 2 g( x ) 2 x2  1 2
x 1
derivative of
outside the outside

d
sin( x 2  x )  cos( x 2  x).( 2x  1)
Ex.48 dx
Inside Inside derivative
left along of the inside
d 1
Ex.49 (a) (1 – x 2 )1/ 4  (1 – x 2 ) – 3 / 4 (–2x ) u = 1 – x2 and n = 1/4
dx 4
(Function defined) on [–1, 1]
–x
= (derivative defined only on (–1, 1))
2(1 – x 2 )3 / 4

Corporate Head Office : Motion Education Pvt. Ltd., 394 - Rajeev Gandhi Nagar, Kota-5 (Raj.)
VECTOR & CALCULUS Page # 73

d d
(b) sin 2x = cos 2x 2x = cos 2x .2 = 2 cos 2x
dx dx

d d
(c) ( A sin( t  )) = A cos (t + ) (t +  ) = A cos (t + ).  = A  cos (t + )
dt dt

Rull No. 12 Power Chain Rule


d n du
* If u  nun –1
dx dx
d  1  d d
Ex.50   = (3 x – 2) –1 = – 1 (3x – 2)–2 ( 3 x – 2)
dx  3x – 2  dx dx

3
= – 1 (3x – 2)–2 (3) = –
(3 x – 2) 2
In part (d) we could also have found the derivation with the Quotient Rule.

d
Ex.51 (a) ( Ax  B)n
dx
du
Sol. Here u = Ax + B, A
dx
d
 ( Ax  B)n  n( Ax  B)n–1.A
dx
d d 1
(b) sin( Ax  B)  cos( Ax  B ).A (c) log(Ax + B) = .A
dx dx Ax  B
d d ( Ax B )
(d) tan (Ax+B) = sec2 (Ax + B).A (e) e  e( Ax B ) .A
dx dx
Note : These results are important

19. DOUBLE DIFFERENTIATION

If f is differentiable function, then its derivative f' is also a function, so f' may have a derivative of its
own, denoted by ( f ' )'  f ' ' . This new function f'' is called the second derivative of because it is the
derivative of the derivative of f. Using Leibniz notation, we write the second derivative of y = f(x) as

d  dy  d2 y
 
dx  dx  dx 2

Another notation is f''(x) = D2 f (x).

Ex.52 If f(x) = x cos x, find f" (x)


d d
Sol. Using the Product Rule, we have f '(x) = x (cos x) + cos x ( x ) = – x sin x + cos x
dx dx
To find f" (x) we differentiate f'(x) :
d d d d
f"(x) = (– x sin x  cos x ) = – x (sin x ) + sin x (–x) + (cos x)
dx dx dx dx
= – x cos x – sinx – sinx = – x cos x – 2 sin x

: 0744-2209671, 08003899588 | url : www.motioniitjee.com, : info@motioniitjee.com


Page # 74 VECTOR & CALCULUS

20. APPLICATION OF DERIVATIVE DIFFERENTIATION AS A RATE OF CHANGE


dy
is rate of change of 'y' with respect to 'x' :
dx
For examples :
dx
(i) v = this means velocity 'v' is rate of change of displacement 'x' with respect to time 't'
dt

dv
(ii) a = this means acceleration 'a' is rate of change of velocity 'v' with respect to time 't'.
dt

dp
(iii) F  this means force 'F' is rate of change of monentum 'p' with respect to time 't'.
dt

dL
(iv)  = this means torque '' is rate of change of angular momentum 'L' with respect to time 't'
dt

dW
(v) Power = this means power 'P' is rate of change of work 'W' with respect to time 't'
dt

 2
Ex.53 The area A of a circle is related to its diameter by the equation A  D .
4
How fast is the area changing with respect to the diameter when the diameter is 10 m ?
Sol. The (instantaneous) rate of change of the area with respect to the diameter is
dA  D
 2D 
dD 4 2
When D =10m, the area is changing at rate (/2) = 5 m2/m. This mean that a small change D m in the
diameter would result in a changed of about 5p D m2 in the area of the circle.

Physical Example :
Ex.54 Boyle's Law state that when a sample of gas is compressed at a constant temperature, the
product of the pressure and the volume remains constant : PV = C. Find the rate of change of
volume with respect to pressure.

dV C
Sol. – 2
dP P

Ex.55 (a) Find the average rate of change of the area of a circle with respect to its radius r as r
changed from
(i) 2 to 3 (ii) 2 to 2.5 (iii) 2 to 2.1
(b) Find the instantaneous rate of change when r = 2.

(c) Show that thre rate of change of the area of a circle with respect to its radius (at any r) is
equal to the circumference of the circle. Try to explain geometrically when this is true by drawing
a circle whose radius is increased by an amount r. How can you approximate the resulting
change in area A if r is small ?
Sol. (a) (i) 5 (ii) 4.5  (iii) 4.1 
(b) 4 
(c) A  2  r r

Corporate Head Office : Motion Education Pvt. Ltd., 394 - Rajeev Gandhi Nagar, Kota-5 (Raj.)
VECTOR & CALCULUS Page # 75

21. MAXIMA & MINIMA


y
Suppose a quantity y depends on another quantity x in a manner shown
in figure. It becomes maximum at x1 and minimum at x2. At these points
the tangent to the curve is parallel to the x-axis and hence its slope is
tan  = 0. Thus, at a maxima or a minima slope
dy x
 0 x1 x2
dx

Maxima
Just before the maximum the slope is positive, at the maximum it
dy y
is zero and just after the maximum it is negative. Thus, decrease
dx
dy 3
at a maximum and hence the rate of change of is negative at 2 2 4 4
dx
1
1 5 5
d  dy  d  dy 
a maximum i.e.,    0 at maximum. The quantity   is slope  m1  tan 1
dx  dx  dx  dx 
m1  m2 (m3  0) > m 4  m5
the rate of change of the slope. It is written x
O For maxima, as x increases
d2 y dy d2 y
as 0 0 the slope decreases
2 . Conditions for maxima are : (a) (b)
dx dx dx 2

Minima
Similarly, at a minimum the slope changes from negative to
positive, y slope  m1  tan 1
m1  m2 (m3  0)  m4  m5
Hence with the increases of x. The slope is increasing 1
that means the rate of change of slope with respect to x is 2 5 5
positive. 1 4
2
d  dy  3 4 3  0
Hence  0
dx  dx 

Conditions for minima are :


O x
For minima, as x increases
dy d2 y slope increases
(a) 0 (b) 0
dx dx2
Quite often it is known from the physical situation whether the quantity is a maximum or a minimum.
d2 y
The test on may then be omitted.
dx 2
Ex.56 Find maximum or minimum values of the functions :
(A) y = 25x2 + 5 – 10x (B) y = 9 – (x – 3)2
dy
Sol. (A) For maximum and minimum value, we can put 0
dx
dy 1
or  50 x – 10  0  x =
dx 5

d2 y
Further,  50
dx 2

: 0744-2209671, 08003899588 | url : www.motioniitjee.com, : info@motioniitjee.com


Page # 76 VECTOR & CALCULUS

d2 y 1 1
or 2 has positive value at x = . Therefore, y has minimum value at x = . Therefore, y has
dx 5 5

1 1
minimum value at x = . Substituting x = in given equation, we get
5 5
2
 1  1
ymin = 25   5 – 10   4
5 5

(B) y = 9 – (x – 3)2 = 9 – x2 – 9 + 6x
or y = 6x – x2
dy
  6 – 2x
dx
dy
For minimum or maximum value of y we will substitute 0
dx
or 6 – 2x = 0
x=3
d2 y
To check whether value of y is maximum or minimum at x = 3 we will have to check whether is
dx 2
positive or negative.
d2 y
 –2
dx 2
d2 y
or is negative at x = 3. Hence, value of y is maximum. This maximum value of y is,
dx 2
ymax = 9 – (3 – 3)2 = 9

22. INTEGRATION
Definitions :
A function F(x) is a antiderivative of a function f(x) if
F'(x) = f(x)
for all x in the domain of f. The set of all antiderivatives of f is the indefinite integral of f with respect
to x, denoted by

 f (x) dx
The symbol  is an integral sign. The function f is the integrand of the integral and x is the variable of
integration.
For example f(x) = x3 then f'(x) = 3x2
So the integral of 3x2 is x3
Similarly if f(x) = x3 + 4
there for general integral of 3x2 is x3 + c where c is a constant
One antiderivative F of a function f, the other antiderivatives of f differ from F by a constant. We
indicate this in integral notation in the following way :

 f( x) dx  F( x)  C .....(i)

The constant C is the constant of integration or arbitrary constant, Equation (1) is read, "The indefinite
integral of f with respect to x is F(x) + C." When we find F(x) + C, we say that we have integrated f
and evaluated the integral.

Corporate Head Office : Motion Education Pvt. Ltd., 394 - Rajeev Gandhi Nagar, Kota-5 (Raj.)
VECTOR & CALCULUS Page # 77

Ex.57 Evaluate  2x dx
an antiderivative of 2x

Sol. 2
 2x dx  x C
the arbitrary constant
The formula x + C generatres all the antiderivatives of the function 2x. The function x2 + 1, x2 – , and
2

x2 + 2 are all antiderivatives of the function 2x, as you can check by differentiation.
Many of the indefinite integrals needed in scientific work are found by reversing derivative formulas.

Integral Formulas
Indefinite Integral Reversed derivated formula

n x n 1 d  x n 1 
1.  x dx 
n 1
 C , n  – 1, n rational
dx  n  1  = x
n

d
 dx  1dx  x  C (special case)
dx
(x)  1

cos kx d  cos kx 
–   sin kx
2.  sin kx dx  – k
C
dx  k 

sin kx d  sin kx 
   cos kx
3.  cos kxdx  k
C
dx  k 

d
2
tan x  sec 2 x
4.  sec xdx  tan x  C
dx

d
2
(– cot x )  csc 2 x
5.  cosec xdx  – cotx  C dx
d
6.  sec x tan xdx  sec x  C dx
sec x = sec x tan x

d
7.  cos ec x cot x dx = – cosec x + C dx
(– csc x )  csc x cot x

Ex.58 Examples based on above formulas :

(a)  dx  x  c
x6
(b)  x 5 dx  C Formula 1 with n = 5
6

1 1
dx  x –1 / 2 dx  2x1 / 2  C  2 x  C
(c)  x  Formula 1 with n = –
2

– cos 2x
(d)  sin 2x dx  2
C Formula 2 with k = 2

x 1 sin(1/ 2)x x 1
(e)  cos 2 dx   cos 2 xdx = 1/ 2
C =  2 sin 2  C Formula 3 with k =
2

: 0744-2209671, 08003899588 | url : www.motioniitjee.com, : info@motioniitjee.com


Page # 78 VECTOR & CALCULUS

Ex.59 Right :  x cos x dx = x sin x + cos x + C


Reason : The derivative of the right-hand side is the integrand :

d
Check : ( x sin x  cos x  C) = x cos x + sin x – sin x + 0 = x cos x.
dx

Wrong :  x cos x dx = x sin x + C


Reason : The derivative of the right-hand side is not the integrand :

d
Check : ( x sin x  C) = x cos x + sin x + 0  x cos x
dx

Rule No. 1 Constant Multiple Rule


• A function is an antiderivative of a constant multiple k of a function f if and only if it is k times an
antiderivative of f.

 kf (x)dx  k  f (x) dx
5( x )n 1
Ex.60  5x n dx = 5 x n dx = c
n 1

Rule No.2 Sum And Difference Rule


A function is an antiderivative of a sum or difference f ± g if and only if it is the sum or difference of an
antiderivative of f an antiderivative of g.

 [f (x)  g( x)dx]   f (x)dx  g(x)dx


Ex.61 Term-by-term integration

2
Evaluate :  (x  2x  5) dx

Sol. If we recognize that (x3/3) – x2 + 5x is an antiderivative of x2 – 2x + 5, we can evaluate the integral as

antideriva
tive 
 arbitrary constant
3
x
 ( x 2  2x  5 ) dx   x2  5x  C
3
If we do not recognize the antiderivative right away, we can generate it term by term with the sum
and difference Rule :

2
 (x  2x  5) dx  x 2 dx  2xdx  5dx
  
x3
  C1  x 2  C 2  5 x  C 3
3
This formula is more complicated than it needs to be. If we combine C1, C2 and C3 into a single constant
C = C1 + C2 + C3, the formula simplifies to

x3
 x 2  5x  C
3

Corporate Head Office : Motion Education Pvt. Ltd., 394 - Rajeev Gandhi Nagar, Kota-5 (Raj.)
VECTOR & CALCULUS Page # 79

and still gives all the antiderivatives there are. For this reason we recommend that you go right to the
final form even if you elect to integrate term by term. Write

x3
 ( x 2  2x  5)dx  x 2 dx  2xdx  5dx 
    x 2  5x  C
3
Find the simplest antiderivative you can for each part add the constant at the end.

Ex.62 We can sometimes use trigonometric identities to transform integrals we do not know how to
evaluate into integrals. The inetgral formulas for sin2 x and cos2 x arise frequently in applications.

2 1  cos 2x 1  cos 2x
(a)  sin x dx =  dx sin 2 x 
2 2

1 1 1
=
2 
(1  cos 2x )dx  
2
dx 
2 
cos 2x dx

x  1  sin 2x x sin 2x
   C   C
2  2 2 2 4

2 1  cos 2x 1  cos 2x
(b)  cos x dx =  dx cos 2 x 
2 2

x sin 2x
   C As in part (a), but with a sign change
2 4

23. SOME INDEFINITE INTEGRALS (AN ARBITRARY CONSTANT SHOULD BE


ADDED TO EACH OF THESE INTEGRALS.

(ax  b)n1 1
(ax  b)n dx 
(a)  a(n  1)
(provided n  –1) + C (b)  x dn  ln x  C
dx 1 ax 1 ax
(c)  a  bx  b ln(a  bx )  C (d) e dx 
a
e C

–1 1
(e)  sin(ax  b)  a
cos( ax  b)  C 
(f) cos(ax  b) 
a
sin(ax  b)  C

(3x  2)4 (3 x  2) 4 2dx


Ex.63 (a)  (3x  2)3 dx  C C (b)   2lnx  C
43 12 x

dx 1 dx 1
(c)  5  2x  2 ln(5  2x )  C (d)  3  5 x   5 ln(3  5x )  C
3x 1
e dx  e 3 x  C –x/2
dx  –2 e – x / 2  C
(e)
3
(f) e
1 1
(g)  sin(3 x  5)dx  – 3 cos(3 x  5)  C (h)  cos(2x  5)dx  2 sin( 2x  5)  C

: 0744-2209671, 08003899588 | url : www.motioniitjee.com, : info@motioniitjee.com


Page # 80 VECTOR & CALCULUS

24. DEFINITE INTEGRATION OR INTEGRTION WITH LIMITS


Upper limit of integration The function is the integrand

x is the variable of integration


b
Integral sign

a 
f ( x ) dx

Lower limit of integration



Integral of f from a to b

b
b
 f( x) dx  [g( x)]
a
a  g(b)  g(a)

4 4
Ex.64  3dx  3 dx  3[ x ] 4–1 = 3 [4 – (–1)] = (3) (5) = 15

–1 –1

/2
/2 
 sin x dx  [– cos x] 0 = – cos  + cos (0) = – 0 + 1 = 1
0 2

a a 5 5 b b
 x3  a3  x2  52  32  x5 / 2  2 5/2
x 2 dx     3/ 2
Ex.65 (1)
0
  3 0 3 3

(2) xdx   2  
 3 2
8
0

(3) x dx   5 / 2   5 b
 0

25. APPLICATION OF DEFINITE INTERGRAL


Calculation Of Area Of A Curve.
f(x)

dx

a x b x
From graph shown in figure if we divide whole area in infinitely small strips of dx width.

We take a strip at x position of dx width.

Small area of this strip dA = f(x) dx


b

So, the total area between the curve and x-axis = sum of area of all strips =  f( x)dx
a

Let f(x)  0 be continuous on [a,b]. The area of the region between the graph of f and the x-axis is


A  f ( x )dx
a

Corporate Head Office : Motion Education Pvt. Ltd., 394 - Rajeev Gandhi Nagar, Kota-5 (Raj.)
VECTOR & CALCULUS Page # 81

Ex.66 Using an area to evaluate a definite integral

Evaluate  xdx
a
0 < a < b. y

b
Sol. We sketch the region under the curve y = x, a  x  b (figure) and y=x
see that it is a trapezoid with height (b – a) and bases a and b.
The value of the integral is the area of this trapezoid : a b
b
ab b2 a2
Thus 
xdx  (b – a) 
a
2 =
2

2
a
0 a b x
5 2 2 a-b
( 5) (1) The region in Example
 xdx 
1
2

2
2

and so on.

Notice that x 2/2 is an antiderivative of x, further evidence of a connection between antiderivatives


and summation.

(i) To find impulse

dp
dF 
dt
so imples =  F.dt
Ex.67 If F = kt then find impulse at t = 3 sec.
f
so impulse will be area under f - t curve

3 3
 t2 

I  kt dt = K  
0  2 0
t
9k
 I
2

2. To calculate work done by force :

 
w  f .dx
So area under f - x curve will give the value of work done.

: 0744-2209671, 08003899588 | url : www.motioniitjee.com, : info@motioniitjee.com


Page # 82 VECTOR & CALCULUS

Exercise - I OBJECTIVE PROBLEMS (JEE MAIN)


SECTION - A : FUNCTION 5. s  5t 3  3t 5
1. f(x) = cos x + sin x Find f(/2)
Sol.
Sol.

6. y  5 sin x
2. f(x) = 4x + 3 Find f(f(2)) Sol.
Sol.

3. f(x) = log x3 and g(x) = log x 7. y  x 2  sin x


Which of the following statement is / are true- Sol.
(a) f(x) = g(x) (b) 3f(x) = g(x)
(c) f(x) = 3g(x) (d) f(x) = (g(x))3
Sol.

8. y  tan x  cot x
Sol.

SECTION - B : DIFFERENTIATION OF ELEMENTRY


FUNCTIONS
Fi nd the de ri vati ve of gi ve n func ti on w.r.t . Find the first derivative & second derivative of given
corresponding independent variable. functions w.r.t. corresponding independent variable.
4. y  x 2  x  8 9. y  6x 2  10x  5 x 2
Sol. Sol.

Corporate Head Office : Motion Education Pvt. Ltd., 394 - Rajeev Gandhi Nagar, Kota-5 (Raj.)
VECTOR & CALCULUS Page # 83

12 4 1 15. y  e x  nx
10. r   
 3  4 Sol.
Sol.

16. y  ( x  1) ( x 2  x  1)
7 3 2
11.   3 z  7 z  21z Sol.
Sol.

12. y  sin x  cos x


Sol. 2  1
17. y  ( x  1)  x  5  
 x
Sol.

13. y  nx  e x
Sol.

18. y  sin x cos x


Sol.

SECTION - C : DIFFERENTIATION BY PRODUCT RULE


Fi nd deri vat i ve of gi ven functi ons w.r.t. the
independent variable x.
14. x sin x
Sol. SECTION - D : DIFFERENTIATION BY QUOTIENT
RULE
Find derivative of given function w.r.t. the independent
variable.
sin x
19. y  cos x

: 0744-2209671, 08003899588 | url : www.motioniitjee.com, : info@motioniitjee.com


Page # 84 VECTOR & CALCULUS

Sol.
24. y  x 2 cot x
Sol.

2x  5
20. y 
3x  2
Sol.
SECTION - E : DIFFERENTIATION BY CHAIN RULE
dy
Find as a function of x
dx
25. y  (2x  1)5
Sol.

nx
21. y 
x
Sol.

26. y  ( 4  3x )9
Sol.

t2  1
22. f ( t )  , find f(t)
t2  t  2
Sol.

7
 x
27. y  1  
 7
Sol.

2x  1
23. z 
x2  1
Sol.

Corporate Head Office : Motion Education Pvt. Ltd., 394 - Rajeev Gandhi Nagar, Kota-5 (Raj.)
VECTOR & CALCULUS Page # 85

10 Sol.
x 
28. y    1
2 
Sol.

33. Suppose that the radius r and surface area S =


4r2 of a sphere are differentiable functions of t. Write
ds dr
29. y  sin 5x an equation that relates to .
dt dt
Sol. Sol.

30. y  sin( x)  ln(x 2 )  e 2 x


SECTION - H : MAXIMA & MINIMA
Sol. 34. Particle’s position as a function of time is given by
x  t 2  4t  4 find the maximum value of position
coordinate of particle.
Sol.

31. y  2 sin( x  ) where  and  constants


Sol.
35. Find the maximum and minimum values of function
2x 3  15x 2  36 x  11
Sol.

SECTION - I
SECTION - G : DIFFERENTIATION AS A RATE
dy
MEASUREMENT Given y = f(u) and u = g(x) Find
32. Suppose that the radius r and area A = r2 of a dx
circle are differentiable functions of t. Write an equation 36. y  2u3 , u  8 x  1
that relates dA / dt to dr / dt.

: 0744-2209671, 08003899588 | url : www.motioniitjee.com, : info@motioniitjee.com


Page # 86 VECTOR & CALCULUS

Sol. (b) x2
Sol.

37. y  sin u, u  3x  1
(c) x2 – 2x + 1
Sol.
Sol.

38. y  6u  9, u  (1/ 2) x 4
2. (a) –3x–4
Sol.
Sol.

x
39. y  cos u, u   (b) x–4
3
Sol. Sol.

PART - II : INTEGRATION

Find integrals of given functions (c) x–4 + 2x + 3


1. (a) 2x Sol.
Sol.

Corporate Head Office : Motion Education Pvt. Ltd., 394 - Rajeev Gandhi Nagar, Kota-5 (Raj.)
VECTOR & CALCULUS Page # 87

Sol.
1
3. (a)
x2
Sol.

1
(c) x
x
5
(b) Sol.
x2
Sol.

43
5. (a) x
3
Sol.
5
(c) 2 
x2
Sol.

1
(b) 3
3 x
Sol.

3
4. (a) x
2
Sol.

(c) 3 1
x
3
x
Sol.

3
(b)
2 x

: 0744-2209671, 08003899588 | url : www.motioniitjee.com, : info@motioniitjee.com


Page # 88 VECTOR & CALCULUS

1 1/ 2 1
6. (a) x 9.
2 3x
Sol. Sol.

Integrate by using the substitution suggested in


bracket.
1 3 / 2
(b)  x
2 10.  sin 3x dx , (use, u = 3x)
Sol. Sol.

11.  sec 2t tan 2t dt , (use, u = 2t)


3 5 / 2
(c)  x Sol.
2
Sol.

7. (1  x 2  3 x 5 ) 12.  5 dx
2
Sol. Sol.

1
8. 3 sin x 
13.
 2 d
Sol. 4

Sol.

Corporate Head Office : Motion Education Pvt. Ltd., 394 - Rajeev Gandhi Nagar, Kota-5 (Raj.)
VECTOR & CALCULUS Page # 89

4 x
x  19. y  1
14.   2  3  dx
2
2
Sol.
Sol.

5 2

15.  r dr Use a definite integral to find the area of the region


2
between the given curve and the x-axis on the interval
Sol. [0, ]
20. y = sin x
Sol.

2

16.  sin  d
0
Sol.

PART - III VECTOR


SECTION - A : DEFINITION OF VECTOR & ANGLE
BETWEEN VECTORS
  
1. Vectors A,B and C are shown in figure. Find angle
between
y
1 
x B 
17. e
0
dx 45º A
30º
x
Sol. 60º

C

     
(i) A and B (ii) A and C (iii) B and C .
Sol.
Use a definite integral to find the area of the region
between the given curve and the x-axis on the interval
[0, b]
18. y = 2x
Sol.

: 0744-2209671, 08003899588 | url : www.motioniitjee.com, : info@motioniitjee.com


Page # 90 VECTOR & CALCULUS

2. The forces, each numerically equal to 5 N, are acting SECTION - B : ADDITION OF VECTORS
as shown in the Figure. Find the angle between forces? 5. A man walks 40 m North, then 30 m East and then 40
m South. Find the displacement from the starting point?
Sol.
5N

60º
5N

Sol.

 
6. Two forces F1 and F2 are acting at right angles to
each other, find their resultant ?
Sol.

3. Rain is falling vertically down wards with a speed 5


m/s. If unit vector along upward is defined as ĵ ,
represent velocity of rain in vector form.
Sol.

7. A vector of magnitude 30 and direction eastwards


is added with another vector of magnitude 40 and
direction Northwards. Find the magnitude and direction
of resultant with the east.
Sol.

4. The vector joining the points A(1, 1, –1) and B(2, –


3, 4) & pointing from A to B is

(a)  î  4 ĵ  5k̂ (b) î  4 ĵ  5k̂

(c) î  4 ĵ  5k̂ (d)  î  4 ĵ  5k̂


 
Sol. 8. Two force of F1  500 N due east and F2  250 N
 
due north. Find F2  F1 ?
Sol.

Corporate Head Office : Motion Education Pvt. Ltd., 394 - Rajeev Gandhi Nagar, Kota-5 (Raj.)
VECTOR & CALCULUS Page # 91

  12. A set of vectors taken in a given order gives a


9. Two vectors a and b inclined at an angle  w.r.t.
 closed polygon. Then the resultant of these vectors
each other have a resultant c which makes an angle is a
   (A) scalar quantity (B) pseudo vector
 with a . If the directions of a and b are interchanged,
then the resultant will have the same (C) unit vector (D) null vector
(A) magnitude Sol.
(B) direction
(C) magnitude as well as direction
(D) neither magnitude nor direction.
Sol.

13. The vector sum of two force P and Q is minimum


when the angle  between their positive directions, is
  
(A) (B) (C) (D) 
4 3 2
Sol.

 
10. Two vectors A and B lie in a plane. Another

vector C lies outside this plane. The resultant
  
A  B  C of these three vectors
(A) can be zero
(B) cannot be zero
 
(C) lies in the plane of A  B
 
(D) lies in the plane of A  B
Sol.  
14. The vector sum of two vectors A and B is
maximum, then the angle  between two vectors is
(A) 0º (B) 30º (C) 45º (D) 60º
Sol.

11. The vector sum of the forces of 10 N and 6 N can


be
(A) 2N (B) 8N (C) 18N (D) 20N
Sol.

    
15. Given : C  A  B . Also, the magnitude of A, B and

C are 12, 5 and 13 units respectively. The angle
 
between A and B is
 
(A) 0º (B) (C) (D) 
4 2

: 0744-2209671, 08003899588 | url : www.motioniitjee.com, : info@motioniitjee.com


Page # 92 VECTOR & CALCULUS

Sol. Sol.


19. If A  3 î  4 ĵ then find Â
Sol.

    
16. If P  Q  P  Q and  is the angle between P and

Q , then
(A)  = 0º (B)  = 90º (C) P = 0 (D) Q = 0
Sol.
20. What are the x and the y components of a 25 m
displacement at an angle of 210º with the x-axis
(clockwise) ?
Sol.

21. One of the rectangular components of a velocity


17. The sum and difference of two perpendicular
of 60 km h–1 is 30 km h–1 . Find other rectangular
vectors of equal lengths are
component ?
(A) of equal lengths and have an acute angle between
them Sol.
(B) of equal lengths and have an obtuse angle between
them
(C) also perpendicular to each other and are of different
lengths
(D) also perpendicular to each other and are of equal
lengths
Sol.

22. If 0.5 î  0.8 ĵ  C k̂ is a unit vector. Find the value


of C
Sol.

SECTION - C : RESOLUTION OF VECTORS


18. Find the magnitude of 3 î  2 ĵ  k̂ ?

Corporate Head Office : Motion Education Pvt. Ltd., 394 - Rajeev Gandhi Nagar, Kota-5 (Raj.)
VECTOR & CALCULUS Page # 93

23. The rectangular components of a vector are Sol.


(2, 2). The corresponding rectangular components of
 
another vector are 1, 3 . Find the angle between the
two vectors.
Sol.

  
27. Three non-zero vectors A ,B & C satisfy the
    
relation A .B  0 & A . C  0 . Then A can be parallel
to:
     
(A) B (B) C (C) B . C (D) B  C
24. The x and y components of a force are 2N and –
Sol.
3N. The force is

(A) 2 î  3 ĵ (B) 2 î  3 ĵ (C)  2 î  3 ĵ (D) 3 î  2 ĵ


Sol.

28. The magnitude of scalar product of two vectors


is 8 and that of vector product is 8 3 . The angle
SECTION-D : PRODUCT OF VECTORS between them is
 
25. If A  î  ĵ  k̂ and B  2 î  ĵ find (A) 30º (B) 60º (C) 120º (D) 150º
    Sol.
(a) A .B (b) A  B
Sol.

 
26. If | A |  4 , | B |  3 and  = 60º in the figure. Find
   
(a) A .B (b) | A  B |

B

 
A

: 0744-2209671, 08003899588 | url : www.motioniitjee.com, : info@motioniitjee.com


Page # 94 VECTOR & CALCULUS

Exercise - II
SECTION - A : FUNCTION
4. y  7 x  tan x
x 1 Sol.
1. If f ( x )  then find f{f(x)}
x 1
Sol.

SECTION - C : DIFFERENTIATION BY PRODUCT RULE


Fi nd deri vat i ve of gi ven functi ons w.r.t. the
corresponding independent variable.

5. y  e x tan x
 x  2, x  2
2. If f ( x )   Evalute f(2), f(1), and f(3) Sol.
2x  1, x  2

Sol.

6. y  x 2 sin 4 x  x cos2 x
Sol.

SECTION - B : DIFFERENTIATION OF ELEMENTRY


FUNCTIONS
Find the first derivative and second derivative of given
functions w.r.t. the independent variable x.

3. y  nx 2  sin x
 1  1 
Sol. 7. y   x    x   1
 x  x 
Sol.

Corporate Head Office : Motion Education Pvt. Ltd., 394 - Rajeev Gandhi Nagar, Kota-5 (Raj.)
VECTOR & CALCULUS Page # 95

SECTION - D : DIFFERENTIATION BY QUOTIENT


8. y  x 2 sin x  2x cos x  2 sin x
RULE
Sol. Find derivative of given functions w.r.t. the respective
independent variable.

sin x  cos x
11. y 
cos x
Sol.

9. y  x 2 cos x  2x sin x  2 cos x


Sol.

cot x
12. y 
1  cot x
Sol.

10. r  (1  sec ) sin 


Sol.

cos x x
13. y  
x cos x

Sol.

: 0744-2209671, 08003899588 | url : www.motioniitjee.com, : info@motioniitjee.com


Page # 96 VECTOR & CALCULUS

tan q 17. y  x( x 2  1)1/ 2


14. p  1  tan q
Sol.
Sol.

dq
18. q  2r  r 2 , find
SECTION - E : DIFFERENTIATION BY CHAIN RULE dr
Sol.
dy
Find as a function of x
dx

15. y  sin3 x  sin 3 x


Sol.

4
 x2 1
16. sin 2 ( x 2  1) 19. y   x 
 8 x 
Sol. Sol.

Corporate Head Office : Motion Education Pvt. Ltd., 394 - Rajeev Gandhi Nagar, Kota-5 (Raj.)
VECTOR & CALCULUS Page # 97

SECTION - F : DIFFERENTIATION AS A RATE 22. A sheet of area 40 m2 in used to make an open


MEASUREMENT tank with a square base, then find the dimensions of
the base such that volume of this tank is maximum.
20. The radius r and height h of a circular cylinder are
related to the cylinder’s volume V by the formula Sol.
V  r 2h.
(a) If height is increasing at a rate of 5 m/s while
radius is constant, Find rate of increase of volume of
cylinder.
(b) If radius is increasing at a rate of 5 m/s while
height is constant, Find rate of increase of volume of
cylinder.
(c) If height is increasing at a rate of 5 m/s and radius
is increasing at a rate of 5 m/s. Find rate of increase
of volume of cylinder.
Sol.
SECTION - H : MISCELLANEOUS
23. Find y if
(a) y  cos x
Sol.

(b) y  sec x
Sol.

SECTION - G : MAXIMA & MINIMA


21. Find two positive numbers x & y such that x + y =
60 and xy is maximum.
Sol.

24. y  cos u, u  sin x


Sol.

: 0744-2209671, 08003899588 | url : www.motioniitjee.com, : info@motioniitjee.com


Page # 98 VECTOR & CALCULUS

25. y  sin u, u  x  cos x 3


4. x ( x  1) dx
Sol. Sol.

PART - II : INTEGRATION t t t
Find integrals of given functions
5.  t2
dt

3 Sol.

1. (2x  5x  7) dx
Sol.

4 t
6.  t3
dt
1 2 
2.   5  x 3
 2x  dx Sol.

Sol.


7. cos  (tan   sec ) d
3.   x  x dx
3
Sol.
Sol.

Corporate Head Office : Motion Education Pvt. Ltd., 394 - Rajeev Gandhi Nagar, Kota-5 (Raj.)
VECTOR & CALCULUS Page # 99

1
2 dx
8.   d 11.  3x  2
0

Sol. Sol.

3
7 Use a definite integral to find the area of the region
x 2 dx between the given curve and the x-axis on the interval
9. 
0 [0, b]
Sol. 12. y = 3x2
Sol.

10.  cos x dx
0 13. y  b 2  x 2
Sol. Sol.

: 0744-2209671, 08003899588 | url : www.motioniitjee.com, : info@motioniitjee.com


Page # 100 VECTOR & CALCULUS

PART - III : VECTOR 3. A vector is not changed if

SECTION - A : DEFINITION OF VECTOR & ANGLE (A) it is displaced parallel to itself


BETWEEN VECTORS (B) it is rotated through an arbitrary angle
 (C) it is cross-multiplied by a unit vector
1. Vector A points N–E and its magnitude is 3 kg ms–
1
it is multiplied by the scalar  such that  = –4 (D) it is multiplied by an arbitrary scalar
second. Find the direction and magnitude of the new Sol.
vector quantity. Does it represent the same physical
quantity or not ?
Sol.

4. Which of the arrangement of axes in fig. can be


labelled “right handed coordinate system” ? As usual,
each axis lable indicates the positive side of the axis.

2. A hall has the dimensions 10 m × 12 m × 14 m. A z


fly starting at one corner ends up at a diametrically
opposite corner. The magnitude of its displacement is
nearly (i) x (ii) x
(A) 16 m (B) 17 m
y z
(C) 18 m (D) 21 m y
Sol.
x
x

(iii) y (iv) z

z
y

x z

(v) y (vi) y

z x

(A) (i), (ii) (B) (iii) (iv)


(C) (vi) (D) (v)

Corporate Head Office : Motion Education Pvt. Ltd., 394 - Rajeev Gandhi Nagar, Kota-5 (Raj.)
VECTOR & CALCULUS Page # 101

Sol.   
6. Find the resultant of three vectors OA, OB and OC
each of magnitude r as shown in figure ?
A

B
r
r
45º
45º
O C
r
Sol.

SECTION : B ADDITION OF VECTOR


 
5. The angle  between directions of forces A and B
is 90º where A = 8 dyne and B = 6 dyne. If the
 
resultant R makes an angle  with A then find the
value of ‘  ’ ?
Sol.

7. If the angle between two forces increases, the


magnitude of their resultant
(A) decreases
(B) increases
(C) remains unchanged
(D) first decreases and then increases
Sol.

: 0744-2209671, 08003899588 | url : www.motioniitjee.com, : info@motioniitjee.com


Page # 102 VECTOR & CALCULUS

8. A car is moving on a straight road due north with a  


10. When two vector a and b are added, the
uniform speed of 50 km h–1 when it turns left through
magnitude of the resultant vector is always
90°. If the speed reamins unchanged after turning,
(A) greater than (a + b)
the change in the velcoity of the car in the turning
process is : (B) less than or equal to (a + b)
(C) less than (a + b)
(A) zero (D) equal to (a + b)
(B) 50 2kmh –1S  W direction Sol.
(C) 50 2kmh –1N  W direction

(D) 50 kmh–1 due West


Sol.

 
11. Given : A  2 î  3 ĵ and B  5 î  6 ĵ . The magnitude
9. Which of the following sets of displacements might  
of A  B is
be capable of bringing a car to its returning point ?
(A) 4 units (B) 10 units
(A) 5, 10, 30 and 50 km
(B) 5, 9, 9 and 16 km (C) 58 units (D) 61 units
(C) 40, 40, 90 and 200 km
Sol.
(D) 10, 20, 40 and 90 km
Sol.

Corporate Head Office : Motion Education Pvt. Ltd., 394 - Rajeev Gandhi Nagar, Kota-5 (Raj.)
VECTOR & CALCULUS Page # 103

      
12. Given : A  2 î  ĵ  2k̂ and B   î  ĵ  k̂ . The unit 14. Given : a  b  c  0 . Cut of the three vectors a, b a
  
vector of A  B is nd c two are equal in magnitude. The magnitude of

3 î  k̂ 3 î k̂  3 î  k̂ the third vector is 2 times that of either of the two


(A) (B) (C) (D) having equal magnitude. The angles between the
10 10 10 10
vectors are
Sol. (A) 90º, 135º, 135º (B) 30º, 60º, 90º
(C) 45º, 45º, 90º (D) 45º, 60º, 90º
Sol.

    
13. If | A  B |  | A |  | B | , then the angle between A

and B is 15. Which of the following is a true statement ?
(A) 0 (B) 60º (C) 90º (D) 120º (A) A vector cannot be divided by another vector
Sol. (B) Angular displacement can either be a scalar or a
vector
(C) Since addition of vectors is commutative therefore
vector subtraction is also commutative
(D) The resultant of two equal forces of magnitude F
acting at a point is F if the angle between the two
forces is 120º.
Sol.

: 0744-2209671, 08003899588 | url : www.motioniitjee.com, : info@motioniitjee.com


Page # 104 VECTOR & CALCULUS

SECTION - C : RESOLUTION OF VECTORS 18. Six forces, 9.81 N each, acting at a point are
  coplanar. If the angles between neighbouring forces
16. If A  3 î  4 ĵ and B  î  ĵ  2k̂ then find out unit
are equal, then the resultant is
 
vector along A  B . (A) 0 N (B) 9.81 N
(C) 2 × 9.81 N (D) 3 × 9.81 N
Sol.
Sol.


17. Vector A is of length 2 cm and is 60º above the SECTION : D PRODUCT OF VECTORS
  
x-axis in the first quadrant. Vector B is of length 2 19. If a  x1 î  y1ˆj & b  x 2 î  y 2 ĵ . The condition that
cm and 60º below the x-axis in the fourth quadrant.  
  would make a & b parallel to each other is ______.
The sum A  B is a vector of magnitude.
(A) 2 along + y axis (B) 2 along + x-axis Sol.
(C) 1 along – x-axis (D) 2 along – x-axis
Sol.

 
20. A vector A points vertically downward & B points
 
towards east, then the vector product A  B is
(A) along west (B) along east
(C) zero (D) none of above
Sol.

Corporate Head Office : Motion Education Pvt. Ltd., 394 - Rajeev Gandhi Nagar, Kota-5 (Raj.)
VECTOR & CALCULUS Page # 105

Exercise - III (JEE ADVANCED)

1. Match the statements given in column-I with 2. Position of particle is given by S = t3 – 2t2 + 5t + 4
statements given in column-II
(a) Find the position of particle at t = 1 sec
Column - I Column - II
(b) Find the first derivative of S at t = 1 sec
    
(A) If | A |  | B | and | A  B |  | A | then (p) 90º (c) Find the second derivative of S t = 1 sec
  Sol.
angle between A and B is

(B) Magnitude of resultant of two (q) 120º


 
forces | F1 |  8N and | F2 |  4 N may be
 
(C) Angle between A  2 î  2 ĵ & B  3 k̂ is (r) 12 N

(D) Magnitude of resultant of vectors (s) 14


 
A  2 î  ĵ & B  3 k̂ is
Sol.

 
3.Two forces F1  2 î  2 ĵ N amd F2  3 î  4 k̂ N are acting
on a particle
(a) Find the resultant force acting on particle
 
(b) Find the angle between F1 & F2
 
(c) Find the componant of force F1 along force F2

Sol.

: 0744-2209671, 08003899588 | url : www.motioniitjee.com, : info@motioniitjee.com


Page # 106 VECTOR & CALCULUS

4. Statement-1 : A vector is a quantity that has   


6. Statement-1 : If three vectors A,B and C satisfy
both magnitude and direction and obeys the triangle
    
law of addition. the relation A .B  0 & A . C  0 then the vector A is
Statement-2 : The magnitude of the resultant vector  
parallel to B  C .
of two given vectors can never be less than the
magnitude on any the given vector.    
Sta te me nt-2 : A  B and A  C he nce A i s
(A) Statement-1 is True, Statement-2 is True;  
State me nt -2 i s a correc t ex pl anat i on for perpendicular to plane formed by B and C
Statement-1 (A) Statement-1 is True, Statement-2 is True;
(B) Statement-1 is True, Statement-2 is True; State me nt -2 i s a correc t ex pl anat i on for
Statement-2 is NOT a correct explanati on for Statement-1
Statement-1 (B) Statement-1 is True, Statement-2 is True;
Statement-2 is NOT a correct explanati on for
(C) Statement-1 is True, Statement-2 is False
Statement-1
(D) Statement-1 is False, Statement-2 is True (C) Statement-1 is True, Statement-2 is False
Sol. (D) Statement-1 is False, Statement-2 is True
Sol.

5. Statement-1 : If the rectangular components of a


force are 8 N and 6 N, then the magnitude of the force 7. Statement-1 : The minimum number of vectors of
is 10 N. unequal magnitude required to produce zero resultant
      is three.
Statement-2 : If | A |  | B |  1 then | A  B |2  | A .B |2  1 . Statement-2 : Three vectors of unequal magnitude
(A) Statement-1 is True, Statement-2 is True; which can be represeted by the three sides of a triangle
State me nt -2 i s a correc t ex pl anat i on for taken in order, produce zero resultant.
Statement-1 (A) Statement-1 is True, Statement-2 is True;
(B) Statement-1 is True, Statement-2 is True; State me nt -2 i s a correc t ex pl anat i on for
Stateent-2 i s N OT a correct expl anat i on for Statement-1
Statement-1 (B) Statement-1 is True, Statement-2 is True;
(C) Statement-1 is True, Statement-2 is False Statement-2 is NOT a correct explanati on for
Statement-1
(D) Statement-1 is False, Statement-2 is True
(C) Statement-1 is True, Statement-2 is False
Sol.
(D) Statement-1 is False, Statement-2 is True
Sol.

Corporate Head Office : Motion Education Pvt. Ltd., 394 - Rajeev Gandhi Nagar, Kota-5 (Raj.)
VECTOR & CALCULUS Page # 107

8. Statement-1 : The angle between the two vectors 10. State true or false
     
 (i) If A & B are two force vectors A .B  B . A
Î  Ĵ and k̂  is 2
radian.
Sol.
 
Statement-2 : Angle between two vectors A and B
 
 A .B 
is given by   cos 1 .
 AB 
 
(A) Statement-1 is True, Statement-2 is True;
State me nt -2 i s a correc t ex pl anat i on for
Statement-1
(B) Statement-1 is True, Statement-2 is True;
Statement-2 is NOT a correct explanation for
Statement-1      
(C) Statement-1 is True, Statement-2 is False (ii) If A & B are two force vectors then A  B  B  A
(D) Statement-1 is False, Statement-2 is True Sol.
Sol.

(iii) If the vector product of two non-zero vectors


vanishes, the vectors are collinear.
Sol.

9. Statement-1 : Distance is scalar quantity.


Statement-2 : Distance is the length of path
transversed.
(A) Statement-1 is True, Statement-2 is True;
State me nt -2 i s a correc t ex pl anat i on for
Statement-1
(B) Statement-1 is True, Statement-2 is True;
Statement-2 is NOT a correct explanation for (iv) If a function has maximum value at point P theh
Statement-1 slope of tangent drawn on function at point P is zero.
(C) Statement-1 is True, Statement-2 is False Sol.
(D) Statement-1 is False, Statement-2 is True
Sol.

: 0744-2209671, 08003899588 | url : www.motioniitjee.com, : info@motioniitjee.com


Page # 108 VECTOR & CALCULUS

11. Fill in the blanks (iv) The magnitude of area of the parallelogram formed
 
(i) The scalar product of vector A  2 î  5 k̂ and by the adjacent sides of vectors A  3 î  2 ĵ a nd
 
B  3 ĵ  5 k̂ is .......... B  2 î  2 k̂ is ......................
Sol. Sol.

 
(ii) If A  3 î  4 ĵ and B  7 î  24 ĵ , then the vector
 
having the same magnitude as B and parallel to A is
............ (v) A force i s represented by 2 î  3 ĵ  6 k̂ . The
Sol. magnitude of the force is .................
Sol.

    (vi) T he uni t ve ct or al ong v ec tor î  ĵ  k̂ is


(iii) If A || B then A  B  ..............
..................
Sol.
Sol.

Corporate Head Office : Motion Education Pvt. Ltd., 394 - Rajeev Gandhi Nagar, Kota-5 (Raj.)
VECTOR & CALCULUS Page # 109

   
(vii) If A is ........................ to B , then A .B  0
Sol.


(viii) The vector A  î  ĵ , where î and ĵ are unit
vectors along x-axis and y-axis respectively, makes
an angle of ..................... degree with x-axis.
Sol.

      
(ix) If A  B  C  0 , then A .(B  C)  ....................
Sol.

: 0744-2209671, 08003899588 | url : www.motioniitjee.com, : info@motioniitjee.com


Page # 110 VECTOR & CALCULUS

1. If the resultant of two forces of magnitudes P and Q acting 3. A man moves towards 3m north then 4m towards east and
finally 5m towards 37º south of west. His displacement from
at a point at an angle of 60º is 7 Q , then P/Q is
origin is
(A) 1 (B) 3/2 (C) 2 (D) 4
(A) 5 2 m (B) 0 m (C) 1 m (D) 12 m
Sol.
Sol.

4. Three forces P, Q & R are acting at a point in the plane. The


2. The resultant of two forces F1 and F2 is P. If F2 is reversed,
angle between P & Q and Q & R are 150º & 120º respectively,
then resultant is Q. Then the value of (P2 + Q2) in terms of F1
then for equilibrium, forces P, Q & R are in the ratio
and F2 is
(A) 2(F12 + F22) (B) F12 + F22 (A) 1 : 2 : 3 (B) 1 : 2 : 3 (C) 3 : 2 : 1 (D) 3 :2:1
(C) (F1 + F2)2 (D) none of these Sol.
Sol.

Corporate Head Office : Motion Education Pvt. Ltd., 394 - Rajeev Gandhi Nagar, Kota-5 (Raj.)
VECTOR & CALCULUS Page # 111

5. A man rows a boat with a speed of 18 km/hr in northwest 7. The resultant of two forces, one double the other in magni-
direction. The shoreline makes an angle of 15º south of west. tude is perpendicular to the smaller of the two forces. The angle
Obtain the component of the velocity of the boat along the between the two forces is
shoreline. (A) 150º (B) 90º (C) 60º (D) 120º
Sol.
3
(A) 9 km/hr (B) 18 km / hr
2
(C) 18 cos 15º km/hr (D) 18 cos 75º km/hr
Sol.

6. A brid moves from point (1, – 2, 3) to (4, 2, 3). If the speed of


the bird is 10m/sec, then the velocity vector of the bird is 8. If the angle between the unit vectors a and b is 60º, then

(A) 5( i  2j  3k ) (B) 5( 4 i  2j  3k ) | a – b | is


(A) 0 (B) 1 (C) 2 (D) 4
(C) 0.6 i  0.8 j (D) 6 i  8 j Sol.
Sol.

: 0744-2209671, 08003899588 | url : www.motioniitjee.com, : info@motioniitjee.com


Page # 112 VECTOR & CALCULUS

9. For a particle moving in a straight line, the position of the      


11. Two vectors A and B are such that | A  B | = | A – B |. The
particle at time (t) is given by x = t3 – 6t2 + 3t + 7 what is the  
velocity of the particle when it’s acceleration is zero ? angle between the vectors A and B is -

(A) – 9ms–1 (B) –12ms–1 (C) 3ms–1 (D) 42ms–1 (A) 0 (B) /3 (C) /2 (D) 
Sol.
Sol.

10. Two forces each numerically equal to 10 dynes are acting 12. A particle moves through angular displacement  on a cir-
as shown in the following figure, then their resultant is - cular path of radius r. The linear displacement will be -
(A) 2r sin (/2) (B) 2r cos (/2)
(C) 2r tan (/2) (D) 2r cot (/2)
10 dynes Sol.

60° 10 dynes

(A) 10 dynes (B) 20 dynes


(C) 10 3 dynes (D) 5 dynes
Sol.

Corporate Head Office : Motion Education Pvt. Ltd., 394 - Rajeev Gandhi Nagar, Kota-5 (Raj.)
VECTOR & CALCULUS Page # 113

  
13. The vector P makes 120° with the x-axis and vector Q 15. The angle that the vector A  2i  3 j makes with y-axis is-
makes 30° with the y-axis. What is their resultant ? (A) tan–1(3/2) (B) tan–1(2/3)
(C) sin–1(2/3) (D) cos–1(3/2)
(A) P + Q (B) P – Q (C) P 2  Q 2 (D) P 2 – Q 2
Sol.
Sol.

14. A man travels 1 mile due east, then 5 miles due south, then 16. A man moves towards 3m north then 4m towards east and
2 miles due east and finally 9 miles due north, how far is he finally 5 m towards 37° south of west. His
from the starting point - displacement from origin is -
(A) 3 miles (B) 5 miles (A) 5 2 m (B) 0 m (C) 12 m (D) 5 m
(C) 4 miles (D) between 5 and 9 miles
Sol.
Sol.

: 0744-2209671, 08003899588 | url : www.motioniitjee.com, : info@motioniitjee.com


Page # 114 VECTOR & CALCULUS


17. If 3 i  2j  8k and 2i  xj  k are at right angles that x= 19. If a is a vector and x is a non-zero scalar, then -
 
(A) 7 (B) –7 (C) 5 (D) –4 (A) x a is a vector in the direction of a
Sol.  
(B) x a is a vector collinear to a
 
(C) x a and a have independent directions
(D) none of these
Sol.

18. a1i  a2 j is a unit vector perpendicular to 4 i – 3 j if -


(A) a1 = .6, a2 = .8 (B) a1 = 3, a2 = 4
(C) a1 = .8, a2 = .6 (D) a1 = 4, a2 = 3
Sol.

Corporate Head Office : Motion Education Pvt. Ltd., 394 - Rajeev Gandhi Nagar, Kota-5 (Raj.)
VECTOR & CALCULUS Page # 115

20. Two vectors have magnitudes 3 unit and 4 unit 21. When two forces of magnitude P and Q are per-
respectively. What should be the angle between them pendicular to each other, their resultant is of magni-
if the magnitude of the resultant is tude R. When they are at an angle of 180º to each
(a) 1unit, R
Sol. other their resultant is of magnitude . Find the
2
ratio of P and Q.
Sol.

(b) 5 unit and


Sol.

22. A body acted upon by 3 given forces is under


(c) 7 unit. equilibrium.
Sol. y

37°

F3
  
(a) If |F1|  10 Nt.,|F2 |  6 Nt. Find the values of |F3 | &
angle ().

: 0744-2209671, 08003899588 | url : www.motioniitjee.com, : info@motioniitjee.com


Page # 116 VECTOR & CALCULUS

Sol.

 24. A part i c l e i s ac te d upon by t he force s


(b) Express F2 in unit vector form
  
Sol. F1  2i  aj  3k , F2  5 i  cj  bk , F3  bi  5 j  7k ,

F4  c i  6 j  ak . Find the values of the constants a,
b, c in order that the particle will be in equilibrium.
Sol.

23. If the four forces as shown are in equilibrium Ex-


 
press F1 & F2 in unit vector form.

15 N F2
°
30 10 N
37° 30°

F1

Sol.

Corporate Head Office : Motion Education Pvt. Ltd., 394 - Rajeev Gandhi Nagar, Kota-5 (Raj.)
VECTOR & CALCULUS Page # 117

25. A plane body has perpendicular axes OX and OY


27. (a) Calculate r  a    where 
marked on it and is acted on by following forces bc a  5 i  4 j  6k ,
 
5P in the direction OY b  2i  2j  3k and c  4 i  3 j  2k .
4P in the direction OX Sol.
10P in the direction OA where A is the point (3a, 4a)
15P in the direction AB where B is the point (–a, a)
Express each force in the unit vector from & calculate
the magnitude & direction of sum of the vector of
these forces.
Sol.

(b) Calculate the angle between r and the z-axis.


Sol.

 
 (c) Find the angle between a and b
26. A vector A of length 10 units makes an angle of
 Sol.
60º with the vector B of length 6 units. Find the
 
magnitude of the vector difference A – B & the angle

it makes with vector A .
Sol.

: 0744-2209671, 08003899588 | url : www.motioniitjee.com, : info@motioniitjee.com


Page # 118 VECTOR & CALCULUS

Exercise - I OBJECTIVE PROBLEMS (JEE MAIN)

PART - I
SECTION - A
1. 1 2. 47 3. (c)
SECTION - B
dy ds dy dy
4.  2x  1 5.  15 t 2  15 t 4 6.  5 cos x 7.  2x  cos x 8. sec 2 x  cos ec 2 x
dx dt dx dx

dy d2 y
9.  12x  10  10x 3 ,  12  30 x  4
dx dx 2

dr d2r
10.  12  2  12   4  4 5 ,  24  3  48  5  20  6
d d 2

d d2 
11.  21z 6  21z 2  42 z , 2
 126 z 5  42 z  42
dz dz

dy d2 y dy 1 d2 y 1
12.  cos x  sin x,   sin x  cos x 13.   ex,   2  ex
dx dx 2 dx x dx 2
x

SECTION - C
x dy 1
e
14. sin x  x cos x 15. e x nx  16.  3x 2 2
17. y  3 x  10 x  2 
x dx x2
18. cos 2 x  sin 2 x
SECTION - D

19 1 nx t 2  2t  1
19. sec2 x 20. y  2 21.  22. f ( t ) 
(3 x  2) x 2
x 2
( t 2  t  2)2

dz  2x 2  2x  2 dy
23.  24.   x 2 csc 2 x  2x cot x
dx ( x 2  1)2 dx

SECTION - E

25. With u  ( 2x  1)

dy dy
y  u5 :   5u 4 . 2  10( 2x  1) 4
dx du

dy
26.  27(4  3 x )8
dx
8
x dy dy du  1  x
27. With u  (1   ) y  u 7 :   7u 8 .     1  
7
  dx du dx  7   7 
11
dy x  2
28.  5  1 29. 5 cos 5x 30. cos( x )   2e 2 x 31. 2 cos( x  )
dx 2  x

Corporate Head Office : Motion Education Pvt. Ltd., 394 - Rajeev Gandhi Nagar, Kota-5 (Raj.)
VECTOR & CALCULUS Page # 119

SECTION - G
dA dr ds dr
32.  2r 33.  8r
dt dt dt dt
SECTION - H
34. 8 35. ymax = 39, ymin = 38
SECTION - I
dy dy 1 x
36.  48 (8 x  1)2 37. 3 cos( 3x  1) 38. 12x3 39.   sin
dx dx 3 3

PART - II
x3 x3 3 1 3 1
1. (a) x 2 (b) (c )  x2  x 2. (a)x (b)  x (c )  x 3  x 2  3 x
3 3 3 3

1 5 5 2 x3
3. (a)  (b)  (c ) 2x  4. (a) x 3 (b) 3 x (c ) 2 x
x x x 3
2 4 2
3 6
5. 4 /3 x3 3x 3 3 x 3 6. (a) x1/ 2 (b) x 1/ 2 (c ) x 3 / 2 7. x  x  x  C
(a) x (b) (c )  3 2
2 4 2

1 1
8. 3 cos x 9. nx 10.  cos 3x  C
3 3

1 3
11. sec 2t  C 12. 15 13. 14. Area = 21 15. 24 16. 0
2 2
b
b 2
17. e – 1 18. Using n subintervals of length x  and right-
n
endpoint values : Area =  2x dx  b
0

b2 b(4  b)
19. b  20. 2
4 4
PART - III
SECTION - A

1. (i) 105º, (ii) 150º, (iii) 105º 2. 120º 3. VR  5 ĵ 4. (C)

SECTION - B

5. 30 m East 6. F12  F22 7. 50, 53º with East 8. 250 5 N, tan 1 (2) W of N
9. (A) 10. (B) 11. (B) 12. (D) 13. (D) 14. (A)
15. (C) 16. (B) 17. (D)

SECTION - C
3 î  4 ĵ
18. 14 19. 20. – 25 cos 30º and + 25 sin 30º 21. 30 3 km h 1
5
22. 0.11 23. 15º 24. (A)

SECTION - D
25. (a) 3 (b)  î  2ˆj  k̂ 26. (a) 6 (b) 6 3 27. (D) 28. (B)

: 0744-2209671, 08003899588 | url : www.motioniitjee.com, : info@motioniitjee.com


Page # 120 VECTOR & CALCULUS

Exercise - II

PART-I
SECTION-A
1
1. – 2. f(2) = 3, f(1) = 3, f(3) = 5
x
SECTION-B
6
– –13
dy 2 d2 y –2 7 d2 y –6
3.   cos x ,  – sin x 4. dy  x  sec 2 x    7 2 tan x sec 2 x
dx x dx2 x2 dx 7 dx2 49

SECTION-C
dy 2 1
5. ex(tanx + sec2x) 6. 2xsin4x + 4x2 sin3x cosx + cos–2x + 2xcos–3xsinx 7.  1  2x  3 – 2
dx x x
dy dr
8. x2 cosx 9.  – x 2 sin x 10.  cos   sec 2 
dx d

SECTION-D
2
dy – csc x dy – x sin x – cos x x sin x  cos x sec 2 q
11.  sec 2 x 12. 13.  + 14.
dx (1  cot x)2 dx x2 cos 2 x (1  tan q) 2

SECTION-E
1 1– r
15. 3sin2x cosx + 3cos3x 16. 4x sin (x2 + 1) cos (x2 + 1) 17. 2 3/ 2 18.
( x  1) 2r – r 2
3
 x2   1 dy dy du 3 x 1  x2 1  x 1
19. With u =  8  + x –  x  , y = u4 : dx  du dx  4u .  4  1  2   4  4  x –    1 2 
  x  8 x   4 x 

SECTION-F
dV dV dr dV dh dr
20. (a) = r2 dh  5 r 2 (b)  2hr  10rh (c)  r 2  2hr = 5r2 + 10 rh
dt dt dt dt dt dt dt
SECTION - G
40
21. x = 30 & y = 30 22. x= m
3
SECTION-H
23. (a) –cosx, (b) 2 sec3 x – sec x
dy
Given y = f(u) and u = g(x), find
dx
dy
24. – sin (sinx) cos x. 25. = cos (x – cosx) (1 + sinx)
dx
PART - II
x4 5 x2 x 1 2 3/2 3 4/3 1 1 2
1. –  7x  C 2.   x2  C 3. x  x C 4. – –  C 5. 2 t – C
2 2 5 x2 3 4 x 2x 2
t
2
2 –3/ 2 3 7 1 5
6. –2t–2 – t +C 7. – cos +  + C 8. 9. 10. 0 11. n
3 2 3 3 2

Corporate Head Office : Motion Education Pvt. Ltd., 394 - Rajeev Gandhi Nagar, Kota-5 (Raj.)
VECTOR & CALCULUS Page # 121

b
b 2 3 b2
12. Using n subintervals of length x =
n
and right-end point values : Area =  3x dx  b
0
13.
4
PART - III
SECTION - A
 
1. B  A  –4  3 N  E = 12 S-W
No it does not represent the same physical quantity.
2. (D) 3. (A) 4. (A), (B), (C)
SECTION-B
5. 37° 6. r(1  2 ) 7. (A) 8. (B) 9. (B) 10. (B) 11. (C)
12. (A) 13. (D) 14. (A) 15. (A), (B), (D)
SECTION-C
4 i  5 j  2k
16. 17. B 18. (A)
45
SECTION- D
x1 y1
19.  20. (D)
x2 y 2

Exercise - III (JEE ADVANCED)

1. (A)  Q, (B)  R, (C)  P, (D)  S


   
F1.F2  3  F1.F2 6
2. (a) 8, (b) 4, (c) 2 (b) cos       = cos–1   (c) F1 cos  =  =
|F1||F2 |  5 2 |F2 | 5
  
3. (a) FR  F1  F2  2i  5 j  4k

4. (C) 5. (B) 6. (A) 7. (A) 8. (A) 9. (B)

10. (i) True (ii) False (iii) True(iv) True


1  1  1 
11. (i) 25 Units. (ii) 15 i  20 j (iii) Null vector (iv) 224 units (v) 7 units (vi) i j k
3 3 3
(vii) Perpendicular (viii) 45 (ix) zero.

1. C 2. A 3. B 4. D 5. A 6. D 7. D
8. B 9. A 10 A 11. C 12. A 13. A 14. B
15. B 16. B 17. B 18. A 19. B
 
20. (a) 180°, (b) 90°, (c) 0 21. 2 ± 3 22. (a) |F3 | = 8 N,  = 90° (b) F2 = –6 i
 
23. F1 = –(12 3 – 1) j & F2 = (12 – 5 3 ) i + (12 3 – 15) j 24. a = – 7, b = – 3, c = – 4

7
25. 5P ĵ,4P î,6Pî  8P ĵ,–12P î – 9P ĵ , 20P, tan–1[–2] with the +ve x axis. 26. 2 19 ; cos–1
2 19

–1 –7  –1  –20 
27. (a) 11i  5 j – 7k , (b) cos   , (c) cos  
 195   1309 

: 0744-2209671, 08003899588 | url : www.motioniitjee.com, : info@motioniitjee.com


Page # 122 KINEMATICS

KINEMATICS
1. REST AND MOTION :
* An object is said to be in motion wrt a frame of reference S1, when its location is changing with
time in same frame of reference S1.
* Rest and motion are relative terms.
* Absolute rest and absolute motion have no meaning.
Motion is broadly classified into 3 categories.
1. Rectilinear and translatory motion.
2. Circular and rotatory motion.
3. Oscillatory and vibratory motion.

1.1 Rectilinear or 1-D Motion


When a particle is moving along a straight line, then its motion is a rectilinear motion.
Parameters of rectilinear motion or translatory motion or plane motion :
(A) Time :
* It is a scalar quantity and its SI unit is second(s).
* At a particular instant of time, a physical object can be present at one location only.
* Time can never decrease.
y
(B) Position or location - It is defined with respect to A C
some reference point (origin) of given frame of reference.
 r1 B
Consider a particle which moves from location r (at time t )
1 1
 r2
to location r2 (at time t2) as shown in the figure below,
following path ACB. x

(C) Distance :
The length of the actual path traversed by the particle is termed as its distance.
Distance = length of path ACB.
* Its SI unit is metre and it is a scalar quantity.
* It can never decrease with time.
(D) Displacement :
The change in position vector of the particle for a given time interval is known as its displacement.
   
AB  r  r2  r1
* Displacement is a vector quantity and its SI unit is metre.
* It can decrease with time.
For a moving particle in a given interval of time
* Displacement can be +ve, –ve or 0, but distance would be always +ve.
* Distance  Magnitude of displacement.
* Distance is always equal to displacement only and only if particle is moving along a straight line
without any change in direction.
(E) Average speed and average velocity :
Average speed and average velocity are always defined for a time interval.
Total dis tan ce travelled s
Average speed(vav )  
Time int erval t
  
 Displacement r r  r
Average velocity (vav )    2 1
Time int erval t t2  t1
* Average speed is a scalar quantity, while average velocity is a vector quantity. Both have the same
SI units, i.e., m/s.
For a moving particle in a given interval of time
* Average speed can be a many valued function but average velocity would be always a single-
valued function.
* Average velocity can be positive, negative or 0 but average speed would be always positive.

Corporate Head Office : Motion Education Pvt. Ltd., 394 - Rajeev Gandhi Nagar, Kota-5 (Raj.)
KINEMATICS Page # 123

(F) Instantaneous speed and instantaneous velocity


Instantaneous speed is also defined exactly like average speed i.e. it is equal to the ratio of total
distance and time interval, but with one qualification that time interval is extremely (infinitesimally)
small. The instantaneous speed is the speed at a particular instant of time and may have entirly
different value than that of average speed. Mathematically.
s ds S
v  lim  ...(4)
s 0 t dt
When s is the distance travelled in time t. B

Distance
As t tends to zero, the ratio defining speed becomes
finite and equals to the first derivative of the distance.
The speed at the moment 't' is is called the instantaneous D S
speed at time 't'. 
On the distance - time plot, the speed is equal to the slope A t C
of the tangent to the curve at the time instant 't'. Let A t
and B point on the plot corresponds to the time t and t + O t t  t time
t during the motion. As t approaches zero, the chord AB
becomes the tangent AC at A. The slope of the tangent Instantaneous speed is equal to the slope
of the tangent at given instant.
equal ds/dt, which is equal to the intantaneous speed at
't'.
DC ds
v = tan = =
AC dt

(G) Instantaneous velocity :


Instantaneous velocity is defined exactly like speed. It is
equal to the ratio of total displacement and time interval,
but with one qualification that time interval is extremely
(infinitesimally) small. Thus, instantaneous velocity can S
be termed as the average velocity at a particular instant
Position/displacement

of time when  t tend to zero and may have entirely


B
different value that of average velocity : Mathematically.
B'
r dr D S
v  lim  
t 0 t dt
A t C
As  t tends to zero, the ratio defining velocity becomes t
O
finite and equals to the first derivative of the position t t  t time
vector. The velocity at the moment 't' is called the Instantaneous velocity is equal to the slope
instantaneous velocity or simply velocity at time 't'. of the tangent at given instant.

The magnitude of average velocity |vavg| and average speed vavg may not be equal, but magnitude of
instantaneous velocity |v| is always equal to instantaneous speed v.

Ex.1 In 1.0 sec a particle goes from point A to point B moving in a semicircle of radius 1.0 m. The
magnitude of average velocity is
(A) 3.14 m/sec (B) 2.0 m/sec (C) 1.0 m/sec (D) zero

Total displacement d A
Sol. Average velocity   1m
Total time t o
D = AO + OB 1m
= 1 + 1 = 2m B
t = 1 sec (given)
2
 mg of v of = 2m/sec
1

: 0744-2209671, 08003899588 | url : www.motioniitjee.com, : info@motioniitjee.com


Page # 124 KINEMATICS

Ex.2 A particle moves along a semicircular path of radius R in time t with


constant speed. For the particle calculate
(i) distance travelled, R
A B
(ii) displacement,
(iii) average speed,
(iv) average velocity,
Sol. (i) Distance = length of path of particle = AB = R
(ii) Displacement = minimum distance between initial and final point
= AB = 2R
total dis tan ce R
(iii) Average speed, v = =
time t
2R
(iv) Average velocity =
t

Ex.3 A body travels the first half of the total distance with velocity v1 and the second half with
velocity v2. Calculate the average velocity :
Sol. Let total distance = 2x. Then
x x  v1  v 2  2x 2v1v 2
total time taken = v  v = x  v v   Average speed = = v v
1 2  1 2   v1  v 2  1 2
x 
 v 1v 2 
(G-1) When velocity is given as a function of t :
Ex.4 Velocity-time equation of a particle moving in a straight line is,
v = (10 + 2t + 3t2)
Find :
(a) displacement of particle from the origin of time t = 1 s, if it is given that displacement is 20 m at
time t = 0
(b) acceleration-time equation.
Sol. (a) The given equation can be written as,
ds
v=  (10  2t  3t 2 )
dt
ds = (10 + 2t + 3t2) dt
s t

or  ds  (10  2t  3t 2 )dt
 or s – 20 = [10t + t2 + t3]01
20 0
or s = 20 + 12 = 32 m
(b) Acceleration-time equation can be obtained by differentiating the given equation w.r.t. time.
Thus,
dv d
a= = (10  2t  3 t 2 ) or a = 2 + 6t
dt dt
SPECIMEN PROBLEM :
(A) WHEN EQUATION OF DISPLACEMENT IS GIVEN AND SPEED TO BE FIND OUT

Ex.5 If displacement is depend on time such that


x = 2t –2 then find out average speed upt to 4 sec.
Total distance
Sol. Average speed =
Total time
for Total distance
at t = 0 it is at x = – 2
at t = 1 it is at 0 m
at t = 4 it is at 6 m.
Total distance = |– 2| + 0 + 6 = 8 m
Average speed = 8/4 = 2m/sec

Corporate Head Office : Motion Education Pvt. Ltd., 394 - Rajeev Gandhi Nagar, Kota-5 (Raj.)
KINEMATICS Page # 125

(B) WHEN VELOCITY IS GIVEN AS A FUNCTION OF TIME AND DISTANCE TO BE FIND OUT
In this type of question first find out at what instant the velocity is zero. If this instant is come in our
time limit then distance can be calculated by breaking the integration in two part with modulas

Ex.6 If velocity is depend on time such that v = 4 – 2t. Find out distance travelled by particle from 1
to 3 sec.
Sol. Velocity is zero (4 – 2t = 0) at t = 2 sec
2 3
dx
So for distance
dt
= 4 – 2t  dx =  (4 – 2t)dt +  (4 – 2t)dt
2
1

dx = 1 + 1 = 2m

(G-2) When velocity is given as a function of x


dx dx
v = f(x) 
dt
 f(x)   f(x)   dt
Ex.7 If velocity is given by following function V = x2. Then find out relation between x & t (assume x =
1 m at t = 0)
Sol. Relation between v & x is
v = x2
dx dx
we know that v =   x2
dt dt
x t x
dx dx  1
x2
 dt  x
1
2
  dt
0
 –   t
 x 1

1 1
 – +1=tx=
x 1–t

(E) Average and instantaneous acceleration.


When the velocity of a moving object/particle changes with time, we can say that it is accelerated.
Average acceleration,
  
 v2  v1 v Change in velocity
(aav )   
t2  t1 t Time int erval
Instantaneous acceleration,

  dv
(a)  lim aav  = Rate of change of velocity
t  0 dt
Acceleration is a vector quantity whose direction is same as that of change in velocity vector. Its SI
unit is m/s2.
* When direction of acceleration and velocity are opposite to each other, then acceleration is termed
as retardation.
  
 dv d2r  dv
* a  dt   v 
d t2 dr

(E-1) When acceleration is given as a function of x

Ex.8 if a = 2x ; initially particle is at x = 2m and is moving with 3 ms–1. Then find out v at x = 5 m.
Sol. Given a = 2x
v 5
vdv

dx
 2x  
3
vdv   2xdx
2
2
v 9
 – = 25 – 4  v2 – 9 = 21 × 2  v = 51 ms–1
2 2

: 0744-2209671, 08003899588 | url : www.motioniitjee.com, : info@motioniitjee.com


Page # 126 KINEMATICS

(E-2) When acceleration is given as function of velocity

Ex.9 If a is depend on v in a following way.


a = v and at t = 0 x = 1m, v = 1m/s.
(a) Find out its velocity at t = 2 sec.
(b) Find out its velocity at x = 3 m
Sol. Given a = v
v 2
dv dv

dt
v   v
 dt
1 0
 ln v = 2  v = e ms–1 2

(b) Given a = v
v 3
vdv

dx
v  
1

dv  dx
1
 v–1=2  v = 3 ms–1

(E-3) When acceleration is given as a function of t.

Ex.10 The acceleration of a particle which is depend on time is given by following function
a = 2t + 1
and at time t = 0, x = 1m and u i = 2m/s.
Then find out displacement of the particle at t = 3 sec.
dv dv
Sol.  We know that a =  = 2t + 1  dv = (2t + 1) dt
dt dt
vf t

 dv =  (2t  1)dt
2 0

vf – 2 = t2 + t  vf = t2 + t + 2
dx
Now v=
dt
xf t
dx dx  ( t 2  t  2)

dt
= t2 + t + 2  
1

0

t3 t 2
xf =   2t  1
3 2
So, xf at t = 3 sec is
( 3)3 (3)2
=   2( 3)  1  20.5 m
3 2
So, after t = 3 sec the position of the particle is 20.5m but the displacement of the particle is
= 20.5 – 1 = 19.5 m

(E-4) Constant Acceleration Format

Deduce the following equations for unifromly accelerated motion by using intergration technique.
1
(A) v = u + at (B) s = ut + at2
2
a
(C) v2 – u2 = 2as (D) snth = u + (2n – 1)
2

Corporate Head Office : Motion Education Pvt. Ltd., 394 - Rajeev Gandhi Nagar, Kota-5 (Raj.)
KINEMATICS Page # 127

First equation of motion. Acceleration is defined as


dv
a
dt
or dv = adt ...(1)
When time = 0, velocity = u (say)
When time = t, velocity = v (say)
Integrating equation (1) within the above limits of time and velocity, we get
v t
t
 
dv  a dt or [ v ]uv  a  dt  at
t
0
u 0
0

or v – u = a(t – 0)
or v = u + at ...(2)
Second equation of motion. Velocity is defined as
ds
v
dt
or ds = v dt = (u + at) dt ...(iii)
When time = 0, displacement travelled = 0
When time = t, displacement travelled = s (say).
Integrating equation (3) within the above limits of time and distance, we get
s t t t t
 t2 
 
ds  (u  at) dt  u  dt  a  t dt or [s]0s  u[t]0t  a 
0 0 0 0  2 0
 t2 
or s – 0 = u (t – 0) + a  2  0
 
1 2
or at
s = ut + ...(4)
2
Third equation of motion. By the definitions of acceleration and velocity,
dv dv ds dv
a
   v
dt ds dt ds
or ads = vdv ...(5)
When time = 0, velocity = u, displacement travelled = 0
When time = t, velocity = v, displacement travelled = s
(say)
Integrating equation (5) within the above limits of velocity and displacement, we get
s v s v v
 v2 
 a ds   v dv or a ds    v dv or a[s]s0   
0 u 0 u  2 u

v2 u2
or a[s  0] 
 or 2as = v2 – u2
2 2
or v2 – u2 = 2as ...(6)
Fourth equation of motion. By definition of velocity,
ds
v
dt
or ds = vdt = (u + at) dt ...(7)
When time = (n – 1) second, displacement travelled
= sn – 1 (say).
When time = n second, displacement travelled = sn
(say)

: 0744-2209671, 08003899588 | url : www.motioniitjee.com, : info@motioniitjee.com


Page # 128 KINEMATICS

Integrating equation (7) within the above limits of time and distance, we get
sn n n n n
 t2 
 ds   (u  at)dt or [s]ssn  u t dt or sn  sn 1  u[t]nn 1  a  
sn 1 n 1
n 1 
n 1
dt  a
n 1
  2 n 1
a a
= u[n –(n – 1)] + [n2 – (n – 1)2] = u + [n2 – (n2 – 2n + 1)]
2 2
a
snth  u 
(2n  1) ...(8)
2
where snth = sn – sn –1 = displacement in nth second.
Ex.11 A car starts from rest and accelerates uniformly for 20 seconds to a velocity of 72 km h–1. It
then runs at constant velocity and finally brought to rest in 200 m with a constant retardation.
The total distance covered is 600 m. Find the acceleration, retardation and the total time taken.
Sol. (i) Motion with uniform acceleration
5
Here, u = 0 ; t1 = 20 sec ; v = 72 × = 20 ms–1
18
 v = u + at1
20 = 0 + a × 20 or a = 1 m s–2
Distance travelled by car in this time (20 sec),
1 2 1
S1 = ut + at = 0 + × 1 × (20)2 = 200 m
2 2
(ii) Motion with uniform velocity.
As given, total distance = 600 m
we have calculated S1 = 200 m (with uniform acc.)
and S2 = 200 m (with retardation)
 Net distance for which body moves with uniform velocity,
S = 600 – S1 – S2
= 600 – 200 – 200 = 200 m
dis tan ce 200
 Time taken, t    10 sec .
uniform velocity 20
 Total time of journey, t = (20 + 10 + 20) sec
t = 50 sec
Total displacement 600
Average velocity =  = 12 m/s .
Total Time 50
(iii) Motion with uniform retardation.
For this motion, initial velocity, u = 20 m s–1 and final velocity v = 0 ; S2 = 200 m
Acceleration a' = ?
Using, v2 – u2 = 2 a' S2
(0)2 – (20)2 = 2(a) × 200
a = – 1 ms–2
Let t = time for which the body comes to rest.
 v = u + a t
0 = 20 – 1t
 t = 20 sec.
C. SPECIMEN PROBLEM 2
a=2m/s
Ex.12 Find out distance travelled by the block u=10 m/s
in 10 sec. for a given situation.
Sol. First find out it what instant velocity of block becomes zero.
v = u + at
given : u = 10 m/s, a = – 2m/s2
 0 = 10 – 2t  t = 5 sec

Corporate Head Office : Motion Education Pvt. Ltd., 394 - Rajeev Gandhi Nagar, Kota-5 (Raj.)
KINEMATICS Page # 129

So we calculate distance for two time intervals.


For first 5 sec.
1 1
S1 = ut – at2  S1 = (10)(5) – (2) (5)2 = 25 m
2 2
for Next 5 sec (blockis travelling towards the starting point)
1
S2 = ut + at2
2
u =0
1
S2 = × 2 × 25 = 25 m
2
S = S1 + S2 = 50 m

(D) Reaction time :


When a particular situation demands our immediate action, it takes some time before we really respond.
Reaction time is the time a person takes to observe, think and act. For example, if a person is driving
and suddenly a boy appears on the road, then the time elapse before he applies the breaks of the car
is the reaction time. Reaction time depends on complexity of the situation and on an individiual.
One can measure one's reaction time by a simple experiment. Take a rule and ask your friend to drop it
vertically through the gap between your thumb and forefinger. As soon as it is dropped, note the time
elapsed ts before you catch it and the distance d travelled by the ruler. (In a particular case, y was
found to be 21.0 cm. Estimate reaction time).
Sol. As the ruler drops under free fall so u = 0, and g = 9.8 ms–2. The distance travelled d and the reaction
time tr are related by
1 2 d
d gtr  4.9tr2 Or, tr 
2 4.9
Here d = 21.0 cm = 0.21 m
0.21
 tr   0.2 s
4.9
Note : a v=0
Definition : Time taken by a driver to react for a situation u
Reaction Time of the driver
is t = t1 – t0
Total distance covered by the car before stopping t1
t=t0 t2
= distance covered in uniform motion during to to t1 + distance (brakes applied)

u2
cover in deaccelerated motion during t1 to t2 = Total distance = u(t) +
2a
Ex.13 Assume that a car is able to stop with a retardation of 8 ms–2 and that a driver can react to an
emergency in 0.5 sec. Calculate the overall stopping distance of the car for a speed of 60 km–1
of the car.
5 50
Sol. Here, u = 60 km h–1  60   ms1
18 3
50
Since the application of brakes takes 0.5 s, before this the car was moving with uniform speed of ms 1 .
3
 Distance covered in 0.5 sec, with a uniform speed is
50 25
S1  u  t   0.5  m  8.33 m
3 3
Now car begins to move with a retardation of 8ms–2
 Distance covered before coming to rest,
2a S2 = v2 – u2
2
 50 
0  2500
or v2  u2  3   50  50   17.36 m
S2   144
2a 2  8 928
 Total (overall) distance = S1 + S2 = 8.33 + 17.36  S = 25.69 m

: 0744-2209671, 08003899588 | url : www.motioniitjee.com, : info@motioniitjee.com


Page # 130 KINEMATICS

Ex.14 Two buses A and B are at positions 50 m and 100 m from the origin at time t = 0. They start
moving in the same direction simultaneously with uniform velocity of 10 ms–1 and 5 ms–1.
Determine the time and postion at which A overtakes B.
Sol. Here we use equation of motion for constant velocity in Cartesian form.
Given x1 (0) = 50 m, x2 (0) = 100 m,
v1 = 10 ms–1, v2 = 5 ms–1
The positions of the two buses at any instant t are
x1 (t) = x1 (0) + v1t = 50 + 10 t
x2 (t) = x2 (0) + v2t = 100 + 10 t
When A overtakes B,
x1 (t) = x2 (t)
50 + 10t = 100 + 5t or 5t = 50
t = 10 s
x1 (10) = x2 (10) = 150 m
Thus A overtakes B at a position of 150 m from the origin at time t = 10 s.

Ex.15 A bus starts from rest with constant acceleration of 5 ms–2. At the same time a car travelling
with a constant velocity of 50 ms–1 overtakes and passes the bus. (i) Find at what distance will
the bus overtake the car ? (ii) How fast will the bus be travelling then ?
Sol. (i) Suppose the bus overtakes the car after covering distance s.
When the two meet, time taken t is same.
1 2 1 2
For bus, s  ut  at  0   5 t
2 2
For car, s = 50 t
5 2
 t  50 t or t = 20 s
2
Hence s = 50 t = 50 × 20 = 1000 m.
(ii) v2 = u2 + 2as = 0 + 2 × 5 × 1000 = 10,000 or v = 100 ms–1
SPECIMEN PROBLEM
(E) Maximum Separation :
u=0
a=4m/s2
Ex.16 40 m/s

What is the maximum separation between car and scooter ?


Sol. Initially seperation between car & scooter increases and then decreases.
Separation between them will be maximum at an instant at which velocity of the car is equal to the
velocity of scooter.
Velocity car = Velocity scooter = 40 m/s
from v = u + at
40 = 0 + 4t  t = 10 sec
The distance travelled by scooter in 10 sec.
is S1 = 10 × 40 = 400 m
The distance travelled by car in 10 sec. is
1 2 1
S2 = ut + at =0+ × 4 × (10)2 = 200 m
2 2
So maximum seperation = S1 – S2 = 400 – 200 = 200 m
* When they meet both will travel the same distance.
Questions 14, 15, 16 can be solved in a relatively easier way using the concept of relative motion
which is explained later
Now you can try Questions 1 to 13 in Ex. 1 and 1 to 6 in Exercise II

Corporate Head Office : Motion Education Pvt. Ltd., 394 - Rajeev Gandhi Nagar, Kota-5 (Raj.)
KINEMATICS Page # 131

2. MOTION UNDER GRAVITY :

I FORMAT : (When a body is thrown vertically upward) B


It includes two types of motion
(i) Deaccelerated motion from A to B because the direction upward
of velocity and acceleration is opposite. So speed motion downward motion
decreases (accelerated motion)
(deaccelerated
(ii) Accelerated motion from B to C because the direction of motion)
velocity and acceleration is same (downward). So speed u
increases A C
(a) Time of flight :
It is the time taken by the particle to reach the ground. If the particle is thrown vertically upward with
initial velocity u then
ui = u
a = – g (take downward direction negative)
from equation
1 2
S = ut + at  Snet = 0 (when particle again reaches the ground)
2
t = T (time of flight)
1 2 2u
0 = uT – gT  T =
2 g
(b) Maximum Height :
from v2 u2 + 2as
at maximum height v = 0, s = Hmax
u2
 0 = u2 –2 gHmax  Hmax =
2g
(c) Final velocity
from v = u + at
2u  2u 
v = vf a = – g t=T=  vf = u – g  g 
g
vf = – u
i.e. the body reaches the ground with the same speed with which it was thrown vertically upwards as
it thrown vertically upward.
(d) Time to reach half of the maximum height :-
ui = u a = – g
1 2
from S = ut + at Hmax
2
H 1 2 Hmax/2
= ut – gt
2 2
H = 2ut – gt2  gt2 – 2ut + H = 0 u

u2
2
2u  4u – 4gH 2u  4u2 – 4g   u2
 t= t= 2g  Hmax 
2g  2g
2g
u(2  2 )
t= ...(1)
2g
Equation 1 gives two value of time which corresponds to
u(2 – 2 )
t1 = (from ground to Hmax/2 in upward motion)
2g
u( 2  2 )
t2 = (from ground to Hmax/2 in downward motion)
2g

: 0744-2209671, 08003899588 | url : www.motioniitjee.com, : info@motioniitjee.com


Page # 132 KINEMATICS

(e) Time to reach any general height h


Let us assume that particle reaches from A to B in time t1 & from A to C is time t2.
1 2 1
So from S = ut + at  h = ut – gt 2
2 2
gt2 – 2ut + 2h = 0 B C

2u  4u 2 – 8gh
 t= h
2g
u
u – u2 – 2gh u  u2 – 2gh
So, t1 = , t2 = A
g g
 t1 + t2 = T (Time of flight)

II Format (Free fall) :


A body released near the surface of the earth is accelerated downward under the influence of force of
gravity.
(a) Time of Flight : (0,0)
u=0
1 2
from equation S = ut + at
2
S = – H, u = 0, a = – g
t = T (Let assume) H

1 2 2H
 – H = (0)T –gt  T =
2 g
(b) Final Velocity when body reaches the ground
from v2 – u2 = 2as
s=–H v = vf u = 0 a = – g
 v f 2 – 0 = 2 (–g) (–H)  vf = 2gH

Ex.17 A ball is thrown vertically upwards with a velocity u from the ground. The ball allains a maximum
height Hmax. Then find out the time and displacement at which ball have half of the maximum
speed.
Sol. Maximum speed of the ball is u
At point B and C ball have speed u/2 but direction
is opposite so from
B C
v = u + at
u/2 u/2
Let t1 is the time taken by the ball from point A to B and t2 is
the time taken by the ball from A to C h
u
From A to B  u – gt1 ...(i)
2 A
u
From A to C –  u – gt 2 ....(2)
2
u 3u
from (i) t1 = , from (ii) t2 =
2g 2g
from equation v2 – u2 = 2as
2
 u 2
 v = ± u/2, u = u, a = – g    – u = – 2gh
 2

3u 2  u 2  3
h=   Hmax   h= Hmax
8g  2g  4

Corporate Head Office : Motion Education Pvt. Ltd., 394 - Rajeev Gandhi Nagar, Kota-5 (Raj.)
KINEMATICS Page # 133

Ex.18 A ball thrown vertically upwards with a speed of 19.6 ms–1 from the top of a tower returns to
the earth in 6 s. Find the height of the tower.
Sol. Here u = 19.6 ms–1
g = –9.8 ms–2
Net displacement, s = – h
Negative sign is taken because displacement is in
the opposite direction of initial velocity.

Tower
1 2
As s = ut + gt h
2
1
 – h = 19.6 × 6 + × (–9.8) × 62
2
= 117.6 – 176.4 = –58.8
or h = 58.8 m

Ex.19 A ball is thrown vertically upwards with a velocity of 20 ms–1 from the top of a multistoreyed
building. The height of the point from where the ball is thrown is 25 m from the ground. (i) How
high will the ball rise and (ii) how long will it be before the ball hits the ground?
Sol. (i) Here u = +20 ms–1, g = –10 ms–2
At the highest point, v = 0
Suppose the ball rises to the height h from the point of projection.
As v2 – u2 = 2gs
 02 – 202 = 2 × (–10) × h or h = + 20 m.
(ii) Net displacement, s = –25 m
Negative sign is taken because displacement is in the opposite direction of initial velocity.
1 2
As s = ut + gt
2

1
 –25 = 20t + × (–10) × t2
2
or 5t2 – 20t – 25 = 0 or t2 – 4t – 5 = 0
or (t+ 1) (t – 5) = 0
As t  –1, so t = 5s.

Ex.20 A ball thrown up is caught by the thrower after 4s. How high did it go and with what velocity was it
thrown ? How far was it below the highest point 3 s after it was thrown?
Sol. As time of ascent = time of descent
 Time taken by the ball to reach the highest point = 2 s
For upward motion of the ball : u = ?, v = 0, t = 2s, g = – 9.8 ms–2
As v = u + gt
 0 = u – 9.8 × 2
or u = 19.6 ms–1
Maximum height attained by the ball is given by
1 2 1
s = ut + gt = 19.6 × 2 + × (9.8) × 22 = 19.6 m.
2 2
Displacement of the ball in 3 s,
1
s = 19.6 × 3 + × (–9.8) × 32 = 58.8 – 44.1 = 14.7 m
2
Distance of the ball from the highest point 3 s after it was thrown
= 19.6 – 14.7 = 4.9 m.

: 0744-2209671, 08003899588 | url : www.motioniitjee.com, : info@motioniitjee.com


Page # 134 KINEMATICS

Ex.21 A balloon is ascending at the rate of 9.8 ms–1 at a height of 39.2 m above the ground when a
food packet is dropped from the balloon. After how much time and with what velocity does it reach
the ground?
Take g = 9.8 ms–2.
Sol. Initially the food packet attains the upward velocity of the balloon, so
u = 9.8 ms–1, g = 9.8 ms–2 , s = –39.2 m
Here s is taken negative because it is in the opposite direction of initial velocity.
1 2
Using, s = ut + gt , we get
2
1
– 39.2 = 9.8 t – × 9.8 t2
2
or 4.9 t2 – 9.8 t – 39.2 = 0 or t2 – 2t – 8 =0
or (t – 4) (t + 2) = 0 or t = 4 s or – 2 s
As time is never negative, so t = 4s.
Velocity with which the food packet reaches the ground is
v = u + gt = 9.8 – 9.8 × 4 = – 29.4 ms–1.
Negative sign shows that the velocity is directed vertically downwards.

When a particle is dropped then it will automatically attains the velocity of the frame at that time.
Ex.22 Two balls are thrown simultaneously, A vertically upwards with a speed of 20 ms–1 from the
ground, and B vertically downwards from a height of 40 m with the same speed and along the
same line of motion. At what points do the two balls collide? Take g = 9.8 ms–2.
Sol. Suppose the two balls meet at a height of x from the ground after time t s from the start.
For upward motion of balls A :
u = 20 ms–1, g = – 9.8 ms–2 u=20 ms–1 B
1 2
s = ut + gt
2 40–x

40 m
1
x = 20 t – × 9.8 t2 = 20t – 4.9 t2 ...(i) C
2
For downward motion of ball B, x
1
40 – x = 20 × t + × 9.8 t2
2 u=20 ms–1 A
= 20t + 4.9 t2
... (ii)
Adding (i) and (ii), 40 = 40 t or t = 1 s
From (i), x = 20 × 1 – 4.9 × (1)2 = 15.1 m
Hence the two balls will collide after 1 s at a height of 15.1 m from the ground.

3. GRAPHS :

(i) Straight line :


A linear relation between y & x represents a straight line.
General equation of straight line
y = mx + c
m  slope of line
c  y intercept i.e. where the line cuts the y-axis. m=tan
Slope is defined as the tan of angle made by the m=tan
straight line with positive x-axis in anticlockwise  
direction.
m < 0   > 90°
m > 0   < 90°
0°   < 180°

Corporate Head Office : Motion Education Pvt. Ltd., 394 - Rajeev Gandhi Nagar, Kota-5 (Raj.)
KINEMATICS Page # 135

Ex.23 Draw the graph for the equation : 2y = 3x + 2


3 (0,1) 3
Sol. 2y = 3x + 2 y = x  1 tan 
2  2
3
m= > 0   < 90°
2
c = +1 > 0
 The line will pass through (0, 1)

Ex.24 Draw the graph for the equation : 2y + 4x + 2 = 0 tan = –2


Sol. 2y + 4x + 2 = 0  y = – 2x – 1 
m = – 2 < 0 i.e.,  > 90°
c = – 1 i.e.,
line will pass through (0, –1) (0,–1)

: (i) If c = 0 line will pass through origin.


(0,c)

(ii) y = c will be a line parallel to x axis.


(c,0)
(iii) x = c will be a line perpendicular to y axis (0,0)
(0,0)
(ii) Parabola
A general quadratic equation represents a parabola.
y = ax2 + bx + c a 0
if a > 0 ; It will be a opening upwards parabola.
if a < 0 ; It will be a opening downwards parabola.
if c = 0 ; It will pass through origin.

2
y=4x +3x

e.g. y = 4 x2 + 3x 2
y=–4x +3x

Average velocity & instantaneous velocity from Position vs time graph


Average velocity from t1 to t2

displacement x 2 – x1 B
= = t –t x2
time taken 2 1
x2–x1
= tan = slope of the chord AB
A 
x 2 – x1 x1
t2–t1
vinstantaneous = as lim
t2  t1 t 2 – t1 t1 t2
when t2 approaches t1 point B approaches Point A and the chord AB becomes tangent to the curve.
Therefore
vinstantaneous = Slope of the tangent x – t curve

: 0744-2209671, 08003899588 | url : www.motioniitjee.com, : info@motioniitjee.com


Page # 136 KINEMATICS

(iii) Reading of Graph


(A) Reading x v/s t graphs Explanation
x

x0
(1) Body is at rest at x0.

t
x

(2) Body starts from origin and is moving with speed tan  away from origin.

 t
x

(3) Body starts from rest from origin and moves away from origin with increasing
speed velocity and positive acceleration.

x
(4) Body starts from rest from x = x0 and moves away from origin with increasing
velocity or +ve acceleration.
x0

(5) x0 Body starts from x = x0 and is moving toward the origin with constant velocity
passes throw origin after same time and continues to move away from origin.

x
x0
(6) Body starts from rest at x = x0 and then moves with increasing speed towards
origin
 acceleration is –ve
t

x
(7) Body starts moving away from origin with some initial speed. Speed of body is
decreasing till t1 and it becomes 0 momentarily of t = t1 and At this instant. Its
reverses its direction and move towards the origin with increasing speed.

t2
t
O t1

Corporate Head Office : Motion Education Pvt. Ltd., 394 - Rajeev Gandhi Nagar, Kota-5 (Raj.)
KINEMATICS Page # 137

(8) x Body starts from origin moves away from origin in the –ve x-axis at t = t1 with
decreasing speed and at t= t1 it comes at rest momentarily, Reverses its direction
t1
t moves towards the origin the increasing speed. Crosses the origin at t = t2.
t2

x
(9) Body starts from origin from rest and moves away from origin with increasing
t speed.

(B) V-T GRAPHS


v

(1) Body is always at rest.

t
v

(2) v0 Body is moving with constant velocity v0

t
v

(3) Body is at rest initially then it starts moving with its velocity increasing at a
constant rate i.e. body is moving with constant acceleration.

v t

(4) Body starts its motion with initial velocity v0 and continues to move with its
v0 velocity increasing at a constant rate i.e. acceleration of the body is constant.

t
v
(5) Body starts its motion with initial velocity v0. Then it continues to move with its
velocity decreasing at a constant rate i.e. acceleration of the body is negative
v0
and constant. At t = t0 the body comes to rest instantaneously and reverses its
t0 direction of motion and then continues to move with decreasing velocity or increasing
t speed.
For 0 < t < t0 motion of the body is deaccelerated ( speed is decreassing)
t > t0 motion of the body is accelerated ( speed is increasing)
v

(6) Body is at rest initially. Then it starts moving with increasing velocity. As time
increases its velocity is increasing more rapidly. i.e. the moving with increasing
acceleration.
t

: 0744-2209671, 08003899588 | url : www.motioniitjee.com, : info@motioniitjee.com


Page # 138 KINEMATICS

v
(7) v0 Body starts its motion with initial velocity v0. Its velocity is decreasing with
time and at t = t0 . It becomes zero after body reverse its direction of motion and
continues to move with decreasing velocity or increasing speed. Since velocity
of the body is decreasing for whole motion. Therefore, its acceleration is
t0 t negative.For 0 < t < t0 motion of the body is deaccelerated (speed is
decreassing) t > t0 motion of the body is accelerated ( speed is increasing)

(C) READING OF a - t GRAPHS

(1) acceleration of the body is zero that means the body is moving constant velocity.

(2) Acceleration of the body is constant and positive.


t

t
(3) Acceleration of the body is constant and negative

(4) Initially the acceleration of the body is zero. Then its acceleration is increasing
at a constant rate.
t

(5) t
The body starts accelerating(initial acceleration zero) at t = 0. Its acceleration is
negative for whole of its motion and is decreasing at a constant rate.

(6) Initially acceleration of the body is zero. Its acceleration is positive for whole of
its motion. Its acceleration is increasing for whole of its motion.
t

Corporate Head Office : Motion Education Pvt. Ltd., 394 - Rajeev Gandhi Nagar, Kota-5 (Raj.)
KINEMATICS Page # 139

(IV) Drawing of graphs on the basis of given information.


(a) If acceleration of the body is zero.
(i) If the velocity of the body is v0 and it starts from origin.
x t v

v0
0
u
x=

t t
(ii) If at t = 0, x = x0 then
x v
t
+v
0

x0 x 0
v0
x=

t t
(iii) If at t = 0, x = – x0 then
x v

+ v 0t t
v0
–x 0
x=
t t
–x0

(b) If a body has constant acceleration :


For this section
(i) u0, x0 & a0 are positive constants. (ii) u  initial velocity (iii) v  velocity at any time t.
(iv) x  Position at any time t.
xi  initial position
(i) if u = 0, a = a0
1
if xi = 0, x = at 2 if xi = x0, x = x0 + (1/2)at2
2 x
x x
This is wrong because
it suggest the body don't
have some initial velocity
x0
t
t t
v a

a0
slope = tan
= a0

t t
v = a0t
(ii) If u = u0 , a = a0
1
x = xi + u0t + a0 t 2 v = u0 + a0t
2
x x
v a

a0

if xi = 0 t if xi = x0 t t t

: 0744-2209671, 08003899588 | url : www.motioniitjee.com, : info@motioniitjee.com


Page # 140 KINEMATICS

(iii) if u = u0, a = – a0
1 2
x = xi + u0t – a0 t
2
x
x

x0
if xi = 0 if xi = x0
t t
t0

v
a
u0
t
t0
t –a0

(iv) if u = – u0 , a = + a0
1
x = xi – u0t + a0 t 2
2
x

x0

if xi = 0 if xi = x0
v
a

a0
t
–u0 t
(v) If u = u0, a = – a0
1
x = xi – u0t – a0 t 2
2
x x
x0

t t

if xi = 0 if xi = x0
v
a

t t
–v0
–a0

Corporate Head Office : Motion Education Pvt. Ltd., 394 - Rajeev Gandhi Nagar, Kota-5 (Raj.)
KINEMATICS Page # 141

Ex.25 Draw the


(a) position vs time graph
(b) velocity vs time graph
(c) acceleration vs time graph
for the following cases
(i) If a body is projected vertically upwards with initial velocity u. Take the projection point to be
origin and upward direction as positive.
1 2
x = ut – gt
2
x v
a
2
u u
u 2u
2g
g g
t t
t
u 2u
g g –u –g

(ii) If a body is dropped from a height h above the ground. Take dropping point to be origin and
upward direction as +ve.
1 2
x = – gt
2
x v a=–g
a
2h 2h
g g
t t
t
v = – gt

–h – 2gh –g

(iii) If a body is projected vertically upwards from a tower of height h with initial velocity u. Take
the projection point to be origin and upward direction as +ve.
x
u2 v
2g a
2u
g
t u/2g t
u v = u – gt t
g
–g
–h

(iv) A car starting from rest accelerates uniformly at 2 ms–2 for 5 seconds and then moves with
constant speed acquired for the next 5 seconds and then comes to rest retarding at 2 ms–2.
Draw its
(a) Position vs time graph
(b) Velocity vs time graph
(c) acceleration vs time graph

: 0744-2209671, 08003899588 | url : www.motioniitjee.com, : info@motioniitjee.com


Page # 142 KINEMATICS

acceleration vs time graph


x (in m)
x
100
a
+2 v
75
10 15 –1
t 10ms
5
15 25
–2 t
5 10
t (in sec)
5 10 15
acceleration vs time graph velocity vs time graph Position vs time graph

(v) A part icle st arts from x = 0 and initial speed 10 ms –1 and moves with constant speed 10ms–1 for
20 sec. and then retarding uniformly comes to rest in next 10 seconds.
acceleration vs time graph
a
v

–1
10ms
20 30
t (sec)

–2 t (sec)
–1ms 20 30
Acceleration vs time graph velocity vs time graph

x
250m

200m

t (sec)
20 30sec

Position vs time graph

(V) Conversion of velocity v/s time graph to speed v/s time graph.
As we know that magnitude of velocity represent speed therefore whenever velocity goes –ve take its
mirror image about time axis.

velocity
speed
e
ag
im
or
Ex-26 irr
m
t (sec)
t (sec)

velocity speed e
ag
im
or
irr
Ex-27 m
t t

Corporate Head Office : Motion Education Pvt. Ltd., 394 - Rajeev Gandhi Nagar, Kota-5 (Raj.)
KINEMATICS Page # 143

(VI) Conversion of displacement vs time graph to distance vs time graph


D

Dist./Displacement
For distance time graph just make the mirror image of the displacement
Dist.-time
time graph from point of zero velocity onwards. C

(VII) Conversion of v - t graphs in to x-t and a-t graphs B Disp-time


v x A
Time

v0 v
t0
(i) 
x=

t t
v x a

a0
0
a

n

(ii)  a - t graph
ta

t t t

v x
a
tan   – a0
t
(iii) 

t0 t
t –a0
t0
at t = t0 velocity reverses its direction.

v
(iv) x – t graph
From t = 0 to t = t1 acceleration = 0 therefore v0
from t = 0 to t = t1, x - t graph will be a straight line.
From t = t1 to t2 acceleration is negative
 It will be an opening downward parabola
x t1 t2 t

t1 t2 t
v
(v) upto t = t1 acceleration is +ve
t1 < t < t2 acceleration is zero.
t > t2 acceleration is –ve
x t
t1 t2

x - t graph
t1 t2 t

: 0744-2209671, 08003899588 | url : www.motioniitjee.com, : info@motioniitjee.com


Page # 144 KINEMATICS

Some important points :


dv
• a=
dt 
 dv = adt 
 v = area under the a - t curve
dx
• v=
dt 
 dx  vdt
 x = area under the v - t curve
 displacement = area under the v - t curve
a
Ex-28 If at t = 0 u = 5 ms–1 then velocity at t = 10 sec
= u + change in velocity –2
5ms
= 5 + area of the shaded part
= 5 + 10 × 5
= 55 ms–1 10 sec t
a
Ex-29 if at t = 0, u = 2 ms–2 find out it maximum velocity 5ms–2
Since whole motion is accelerating. Therefore velocity
will be max at the end of the motion which will be
1
=2+ × 5 × 10 = 27 ms–1 t
2 10sec
Ex-30 if at t = 0, u = 4 ms–1 a
Find out v at
t = 10 sec, t = 20 sec & t = 30 sec. –2
10ms
Since for whole motion acceleration of the body is positive
1
vt= 10 sec = 4 + × 10 × 10 = 54 ms–1
2 t
1 10sec 20sec 30sec
vt = 20 sec = 4 + × 10 × 10 + 10 × 10
2
= 154 ms –1

1
vt = 30 sec = 154 + × 10 × 10 = 204 ms–1
2
(VIII) Reading of graphs if the motion of two bodies are sketched on the same axes.
(a) Reading of x - t graphs
x

x3
B

x2

x1
A

O t1 t2 t3 t
Conclusions :
(i) Body A Start its motion at t = 0 from origin and is moving away from the origin with constant velocity.
Finally it ends its motion at a distance of x2m from origin at t = t3.
(ii) Body B starts its motion at t = t1 from origin and is moving away from origin with constant velocity.
Finally it ends its motion at a distance of x3m from origin at t = t3
(iii) Since slope of B is greater than slope of A. Therefore velocity of B is greater than velocity of A.
(iv) A t = t2, Both A & B are at the same distance from starting point that means B overtakes A at t = t2
(v)  velocity of both A & B are constant
 acceleration of both the bodies are zero.
(vi)  x3 > x2
 At the end of the motion B is at a greater distance from the starting point.

Corporate Head Office : Motion Education Pvt. Ltd., 394 - Rajeev Gandhi Nagar, Kota-5 (Raj.)
KINEMATICS Page # 145

x2 A

B
x1
Ex-31 x0

t0 t1 t
Conclusion :
(i) Body A starts its motion at t = 0 from origin and is moving away from the origin with constant
velocity. Finally its motion ends at t = t1 at x = x2 m.
(ii) Body B starts its motion at t = 0 from x = x0 and then moves with constant velocity away from the
origin. Finally it ends its motion at t = t1.
(iii) Velocity of A is greater than that of B.
(iv) At t = t0 A overtakes B
(v) acceleration of both A & B is zero.
(vi)  x2 > x1
 At the end of the motion A is at a greater distance from the starting point then B

x
B
A

Ex-32

t1 t2 t
Conclusions :
(i) Both A & B starts their motion at same time t = 0 and from same point x = 0.
(ii) Both are moving away from the starting point.
(iii) A is moving with constant velocity while B starts its motion from rest and its velocity is increasing
with time i.e. it has some positive acceleration.
(iv)  At t = t1 the tangent on B's graph becomes parallel to the A's graphs
 At t = t1 velocity of both A & B is same.
(v) For t < t1 velocity of A is greater than velocity of B. Therefore up to t = t1, separation between A
& B increases with time.
(vi) For t > t1 velocity of B is greater than velocity of A. Therefore after t = t1 separation between A &
B starts decreasing and it becomes zero at t = t2 where B overtakes A.

Now you can try Questions 14 to 38 in Exercise I and Ques. 7 to 11 in Ex.II

4. TWO DIMENSIONAL MOTION OR MOTION IN A PLANE


Motion in a plane can be described by vector sum of two independent 1D motions along two mutual
perpendicular directions (as motions along two mutual directions don’t affect each other).
Consider a particle moving in X-Y plane, then its equations of motions for X and Y axes are
vx = ux + axt, vy = uy + ayt
1 1
x = uxt + axt2, and ; y = uyt + ayt2, and
2 2
v 2v  u2x  2a x x v 2y  u 2y  2a y y
where symbols have their usual meanings. Thus resultant motion would be described by the equations
 
r  x i  y j and xv  v i  v j
y

: 0744-2209671, 08003899588 | url : www.motioniitjee.com, : info@motioniitjee.com


Page # 146 KINEMATICS

4.1 PROJECTILE MOTION


It is the best example to understand motion in a plane. If we project a particle obliquely from the
surface of earth, as shown in the figure below, then it can be considered as two perpendicular 1D
motions - one along the horizontal and other along the vertical.

Y u

 u sin +
u cos 

O x
Assume that effect of air friction and wind resistance are negligible and value of ‘acceleration due to

gravity g is constant.
Take point of projection as origin and horizontal and vertical direction as +ve X and Y-axes, respectively.
For X-axis For Y - axis
ux = u cos, uy = u sin
ax = 0, ay = – g,
vx = u cos, and vy = u sin – gt, and
1
x = u cos × t y = u sint – gt2
2
It is clear from above equations that horizontal component of velocity of the particle remains constant
while vertical component of velocity is first decreasing, gets zero at the highest point of trajectory and
then increases in the opposite direction. At the highest point, speed of the particle is minimum.
The time, which projectile takes to come back to same (initial) level is called the time of flight (T).
At initial and final points, y = 0,
1
So u sint – gt2 = 0
2
2u sin 2u sin
 t = 0 and t  So, T=
g g
Range (R) The horizontal distance covered by the projectile during its motion is said to be range of the
projectile
u 2 sin 2
R = u cos × T =
g
For a given projection speed, the range would be maximum for  = 45°.
Maximum height attained by the projectile is
u 2 sin 2 
H=
2g
at maximum height the vertical component of velocity is 0.
u sin T
Time of ascent = Time of descent = =
g 2

Speed, kinetic energy, momentum of the particle initialy decreases in a projectile motion and attains a

minimum value (not equal to zero) and then again increases.


 is the angle between v and horizontal which decreases to zero. (at top most point) and again

increases in the negative direction

Corporate Head Office : Motion Education Pvt. Ltd., 394 - Rajeev Gandhi Nagar, Kota-5 (Raj.)
KINEMATICS Page # 147

Ex.33 A body is projected with a velocity of 30 ms–1 at an angle of 30° with the vertical. Find the
maximum height, time of flight and the horizontal range.
Sol. Here u = 30 ms–1,
Angle of projection,  = 90 – 30 = 60°
Maximum height,
u 2 sin 2  30 2 sin2 60
H= = = 34.44 m
2g 2  9.8
Time fo flight,
2u sin 2  30 sin 60
T= = = 5.3 s
g 9.8
Horizontal range,
u 2 sin 2 30 sin120 30 2 sin 60
R= = = = 79.53 m.
g 9.8 9.8
Ex.34 Find out t he relat ion bet w een u A, uB, uC (where uA, uB, uC are the initial velocities of particles A,
B, C, respectively)

B C
A

Sol.  Hmax is same for all three particle A, B, C


u2y
 Hmax =
2g
 uy is same for all  uyA = uyB = uyC

 2u y 
 TA = TB = TC  g 
 
2u xu y
from figure RC > RB > RA R =
g
 uxC > uxB > uxA  uA < uB< uC

(C) Coordinate of a particle after a given time t :


Particle reach at a point P after time t then Y vy
x = ucos .t v
x 
vx
1 2 P(x,y)
y = usin.t – gt usin
2
u y
Position vector 
O ucos X
  1 
r  (u cos .t ) î   (u sin )t – gt 2  ĵ
 2 

(D) Velocity and direction of motion after a given time :


After time 't' vx = ucos and vy = usin – gt

Hence resultant velocity v = v x2  v y 2 = u 2 cos 2   (u sin  – gt) 2

vy u sin  – gt –1  u sin  – gt 
tan  = =   = tan  
vx u cos  u cos  

: 0744-2209671, 08003899588 | url : www.motioniitjee.com, : info@motioniitjee.com


Page # 148 KINEMATICS

(E) Velocity and direction of motion at a given height :


At a height 'h', vx = ucos

And vy = u 2 sin 2  – 2gh


 Resultant velocity

v= v x2  v y 2 = (u cos ) 2  u2 sin2  – 2gh

v= u2 – 2gh
Note that this is the velocity that a particle would have at height h if it is projected vertically from
ground with u.

Ex.35 A body is projected with a velocity of 20 ms–1 in a direction making an angle of 60° with the
horizontal. Calculate its (i) position after 0.5 s and (ii) velocity after 0.5 s.
Sol. Here u = 20 ms–1,  = 60° , t = 0.5 s
(i) x = (u cos)t = (20 cos60°) × 0.5 = 5 m
1 2
y = (u sin ) t – gt = (20 × sin 60°) × 0.5
2
1
– × 9.8 × (0.5)2 = 7.43 m
2
(ii) vx = u cos  = 20 cos 60° = 10 ms–1
vy = u sin  – gt = 20 sin 60° – 9.8 × 0.5
= 12.42 ms–1
vy 12.42
 v= v 2x  v 2y = (10)2  (12.42) 2 =15.95 ms–1 tan  = v = = 1.242
x 10
  = tan–1 1.242 = 51.16°.

Equation of trajectory of a projectile.


Suppose the body reaches the point P(x, y) after time t.
Y
vy
v A Path of
x  v projectile
x

usin P(x,y)
Max.
u y height=h
m
 vx=u cos
O ucos X
B 
R v
uy
 The horizontal distance covered by the body in time t,
x = Horizontal velocity × time = u cos . t
x
or t =
u cos 
For vertical motion : u = u sin, a = –g, so the vertical distance covered in time t is given by

1 2 x 1 x2
s = ut + at or y = u sin . – g. 2
2 ucos 2 u cos 2 

1 x2
or y = x tan – g 2 ...(1)
2 u cos2 

Corporate Head Office : Motion Education Pvt. Ltd., 394 - Rajeev Gandhi Nagar, Kota-5 (Raj.)
KINEMATICS Page # 149

or y = px – qx2, where p and q are constants.


Thus y is a quadratic function of x. Hence the trajectory of a projectile is a parabola.

From equation (1)

 gxcos   gx 
y = x tan  1– 2 2   y = x tan  1 – 2u2 cos  sin  
 2u cos  sin   

 x
y = x tan  1 – R  ...(2)
 
Equation (2) is another form of trajectory equation of projectile

Ex.36 A ball is thrown from ground level so as to just clear a wall 4 m high at a distance of 4 m and falls
at a distance of 14 m from the wall. Find the magnitude and direction of the velocity.
Sol. The ball passes through the point P(4, 4). So its range = 4 + 14 = 18m.
The trajectory of the ball is,
Now x = 4m, y = 4m and R = 18 m
y
 4  7
 4 = 4 tan  1 – 18  = 4 tan . P(4,4)
  9 u

9 9 7
or tan  = , sin  = , cos = 4m
7 130 130

18  9.8  130
or u2 = = 182 4m x
297 14m

or u = 182 = 13.5 ms
–1

Also  = tan–1(9/7) = 52.1°

Ex.37 A particle is projected over a triangle from one end of a horizontal base and grazing the vertex
falls on the other end of the base. If  and  be the base angles and  the angle of projection,
prove that tan  = tan  + tan .
Sol. If R is the range of the particle, then from the figure we have

y y y(R – x)  xy
tan  + tan  = + = Y
x R–x x(R – x)

y R
or tan + tan  =  ...(1) P(x,y)
x (R – x)
Also, the trajectrory of the particle is
y

 x  
y = x tan  1– R  O
x
  x B A
R–x

y R
or tan = 
x (R – x)
From equations (1) and (2), we get
tan  = tan  + tan .

: 0744-2209671, 08003899588 | url : www.motioniitjee.com, : info@motioniitjee.com


Page # 150 KINEMATICS

4.2 Projectile fired parallel to horizontal. As shown in shown u


O
figure suppose a body is projected horizontally with velocity u x
from a point O at a certain height h above the ground level. y
The body is under the influence of two simultaneous independent x P vx
motions: h 
(i) Uniform horizontal velocity u. v
(ii) Vertically downward accelerated motion with vy
constant acceleration g.
R
Under the combined effect of the above two motions,
Y A Ground
the body moves along the path OPA.
Trajectory of the projectile. After the time t, suppose the body reaches the point P(x, y).
The horizontal distance covered by the body in time t is
x
x = ut  t=
u
The vertical distance travelled by the body in time t is given by
1 2
s = ut + at
2
1 2 1 2
or y=0×1+ gt = gt
2 2
[For vertical motion, u = 0]
2
1 x  g  2  x
or y g    2 x  t  u 
2 u  2u   
g
or y = kx2 [Here k = = a constant]
2u2
As y is a quadratic function of x, so the trajectory of the projectile is a parabola.
Time of flight. It is the total time for which the projectile remains in its flight (from 0 to A). Let T be its
time of flight.
For the vertical downward motion of the body,
we use
1 2
s = ut + at
2
1 2h
or h = 0 × T + gT 2 or T=
2 g
Horizontal range. It is the horizontal distance covered by the projectile during its time of flight. It is
equal to OA = R. Thus R = Horizontal velocity × time of flight = u × T
2h
or R=u
g
Velocity of the projectile at any instant. At the instant t (when the body is at point P), let the
velocity of the projectile be v. The velocity v has two rectangular components:
Horizontal component of velocity, vx = u
Vertical component of velocity, vy = 0 + gt = gt
 The resultant velocity at point P is

v v2x  v2y = u2  g2 t2
If the velocity v makes an angle  with the horizontal, then
vy gt  gt 
tan  = v = or  = tan–1  u 
x u

Corporate Head Office : Motion Education Pvt. Ltd., 394 - Rajeev Gandhi Nagar, Kota-5 (Raj.)
KINEMATICS Page # 151

Ex.38 A body is thrown horizontally from the top of a tower and strikes the ground after three seconds
at an angle of 45° with the horizontal. Find the height of the tower and the speed with which
the body was projected. Take g = 9.8 ms–2.
Sol. As shown in figure, suppose the body is thrown horizontally from the top O of a tower of height y with
velocity u. The body hits the ground after 3s. Considering verticlly downward motion of the body,
1 2 1
y = uyt + gt = 0 × 3 + ×9.8 × (3)2 = 44.1 m [ Initial vertical velocity, uy = 0]
2 2
Final vertical velocity,
vy = uy + gt = 0 + 9.8 × 3 = 29.4 ms–1
Final horizontal velocity, vx = u
As the resultant velocity u makes an angle of 45° with the horizontal, so
vy 29.4
tan 45° = or 1 = or u = 29.4 ms–1.
vx x

Ex.39 A particle is projected horizontally with a speed u from the top of plane inclined at an angle 
with the horizontal. How far from the point of projection will the particle strike the plane?
Sol. The horizontal distance covered in time t,
x
x = ut or t =
u u
The vertical distance covered in time t, 
1 2 1 x2
y=0+ gt = g  2 [using (1)] y D
2 2 u
y gx2 
Also  tan or y = x tan    x tan
x 2u 2 x=ut
 gx 
or x  2 – tan    0
 2u 
2u2 tan 
As x = 0 is not possible, so x =
g
The distance of the point of strike from the point of projection is
D= x2  y2 = x2  (x tan )2
2u2
=x 1  tan2  = x sec  or D = tan  sec 
g

Ex.40 A ball rolls off the top of a stairway with a constant horizontal velocity u. If the steps are h
2hu2
metre high and w meter wide, show that the ball will just hit the edge of nth step if n =
gw 2
Sol. Refer to figure. For n th step,
net vertical displacement = nh u
net horizontal displacement = n 1st
Let t be the time taken by the ball to reach the nth step. Then 2nd
R = ut h
n w
or n = ut or t =
u
1 2
Also, y = uy t + gt nth
2
R
2
1 2 1  n  2hu2
or nh = 0 + gt = g   or n =
2 2  u  g2

: 0744-2209671, 08003899588 | url : www.motioniitjee.com, : info@motioniitjee.com


Page # 152 KINEMATICS

4.3 Projectile at an angle  from height h


Consider the projectile as shown in the adjacent figure.
Take the point of projection as the origin the X and u
Y
Y-axes as shown in figure.
For X-axis, 
ux = u cos
ax = 0
vx = u cos, and h
x = u cos  × t
For Y-axis,
uy = u sin , x
ay = –g,
gt 2
vy = u sin  – gt, and y = u sin  t –
2
Ex.41 From the top of a tower 156.8 m high a projectile is projected with a velocity of 39.2 ms–1 in a
direction making an angle 30° with horizontal. Find the distance from the foot of tower where it
strikes the ground and time taken to do so.
Sol. The situation is shown –1
Here height of tower u=39.2 ms
OA = 156.8 m H
u = 39.2 ms–1
uy = usin
 = 30°
time for which projectile remain is air = t = ?
Horizontal distance covered R = OD = ? =30°
A
Now ux = u cos  and ux = u cos B
uy = u sin  be the components of velocity u .
Motion of projectile from O to H to D
1 156.8 m
Using equation y = uyt + ay t 2
2
Here : y = 156.8 m ; uy = – u sin
= 39.2 sin 30°
ay = 9.8 m/s2 ; t = ? O C D
156.8 = – 39.2 × 0.5 t + 4.9 t2
156.8 = – 19.6 t + 4.9 t2
or 4.9 t2 – 19.6 t – 156.8 = 0
or t2 – 4t – 32 = 0  (t – 8) (t + 4) = 0
We get t = 8 s; t = – 4s
t = – 4 s is not possible, thus we take t = 8s.
Now horizontal distance covered in this time
R = ux × t = u cos  × t = 39.2 × cos 30° × t
R = 271.57 m
4.4 Projectile Motion in Inclined Plane
Here, two cases arise. One is up the plane and the other is down the plane. Let us discuss both the
cases separately.
(i) Up the Plane : In this case direction x is chosen up the plane and direction y is chosen perpendicular
to the plane. Hence,
ux = u cos  , ax = – g sin 
uy = u sin  and ay = – g cos 

Corporate Head Office : Motion Education Pvt. Ltd., 394 - Rajeev Gandhi Nagar, Kota-5 (Raj.)
KINEMATICS Page # 153

y x

B
u

gsin
 gcos

 g
 O
O
C
Now, let us derive the expressions for time of flight (T) and range (R) along the plane.

Time of flight
1
At point B displacement along y-direction is zero. So, substituting the proper values in sy = uyt + ay t 2 ,
2
we get
1 2u sin 
0 = ut sin  + (– g cos ) t2  t = 0 and g cos 
2
2u sin 
t = 0, corresponds to point O and t = corresponds to point B. Thus,
g cos 

2u sin 
T=
g cos 
Range

Range (R) or the distance OB is also equal to be displacement of projectile along x-direction in the
t = T. Therefore.
1 1
R = sa = uxT + ax T 2  R = u cos  T – sin  T2
2 2

(ii) Down the inclined plane : y


along x - axis y-axis
(1) ux = ucos  (1) uy = usin
(2) ax = g sin  (2) ay = g cos 
u sin 
u
velocity at P
vy = uy + ay T (0,0) 
vx = ux + axT uc
os
2u y 2u sin  
Time of flight T = ay =
g cos  
c os gs P
1 g  in
2 
Range Sx = ux T + a x T
2 
1 g
2
= u cos  T + g sin . T
2 x

: 0744-2209671, 08003899588 | url : www.motioniitjee.com, : info@motioniitjee.com


Page # 154 KINEMATICS

Ex.42 A particle is projected at an angle  with horizontal from the foot of a plane whose inclination to
horizontal is  . Show that it will strike the plane at right angles if cot = 2 tan ( – )
Sol. Let u be the velocity of projection so that u cos ( – ) and u sin ( – ) are the initial velocities
respectively parallel and perpendicular to the inclined plane. The acceleration in these two directions
are (–g sin ) and (–g cos ).
The initial component of velocity perpendicular to PQ is u sin ( – ) and the acceleration in this
direction is (–g cos). If T is the time the particle takes to go from P to Q then in time T the space
described in a direction perpendicular to PQ is zero.
1 u
0 = u sin ( – ).T – g cos .T2
2 Q

2u sin( – )
T=
g cos 

If the direction of motion at the instant when the particle 
hits the plane be perpendicular to the plane, then the
velocity at that instant parallel to the plane must be zero. P N
 u cos ( – ) – g sin T = 0
u cos( – ) 2u sin( – )
=T=
g sin  g cos 
 cos = 2 tan ( – )

Ex.43 Two inclined planes OA and OB having inclinations 30° and 60° with x
y
the horizontal respectively intersect each other at O, as shwon in
u v B
figure. a particle is projected from point P with velocity u = 10 3 m / s
along a direction perpendicular to plane OA. If the particle strikes A Q

plane OB perpendicular of flight, then calculate. P


h
30° 60°
(a) time of flight
O
(b) velocity with which the particle strikes the plane OB,
(c) height h of point P from point O
(d) distance PQ. (Take g = 10 m/s2)
Sol. Let us choose the x and y directions along OB and OA respectively. Then,
ux = u = 10 3 m/s, uy = 0
ax = – g sin 60° = – 5 3 m/s2
and ay = – g cos 60° = – 5 m/s2
(a) At point Q, x-component of velocity is zero. Hence, substituting in
vx= ux + axt
10 3
0 = 10 3 – 5 3t  t= = 2s Ans.
5 3
(b) At point Q, v = vy = uy + ayt
 v = 0 – (5) (2) = –10 m/s Ans.
Here, negative sign implies that velocity of particle at Q is along negative y direction.
(c) Distance PO = |displacement of particle along y-direction| = |sy|
1 1
Here, sy = uyt + ay t2 = 0 – (5)(2)2 = – 10 m
2 2
 PO = 10 m

Corporate Head Office : Motion Education Pvt. Ltd., 394 - Rajeev Gandhi Nagar, Kota-5 (Raj.)
KINEMATICS Page # 155

1
Therefore, h = PO sin 30° = (10)   or h = 5m Ans.
2

(d) Distance OQ = displaement of particle along


x-direction = sx
1 1
Here, sx = uxt + ax t2 = (10 3)(2) – (5 3)(2)2 = 10 3 m
2 2

or OQ = 10 3 m

PQ = (PO)2  (OQ)2 = (10)2  (10 3)2 = 100  300  400


PQ = 20 m Ans.

Now you can try Questions 45 to 68 in Exercise I and Ques. 12 to 20 in Ex.II

5. RELATIVE MOTION

The word 'relative' is a very general term, which can be applied to physical, nonphysical, scalar or
vector quantities. For example, my height is five feet and six inches while my wife's height is five feet
and four inches. If I ask you how high I am relative to my wife, your answer will be two inches. What
you did? You simply subtracted my wife's height from my height. The same concept is applied everywhere,
whether it is a relative velocity, relative acceleration or anything else. So, from the above discussion

we may now conclude that relative velocity of A with respect of B (written as 


v AB ) is
  
v AB  v A – v B
Similarly, relative acceleration of A with respect of B is
  
a AB  a A – a B
If it is a one dimensional motion we can treat the vectors as scalars just by assigning the positive sign
to one direction and negative to the other. So, in case of a one dimensional motion the above
equations can be written as
vAB = vA – vB
and aAB = aA – aB
Further, we can see that
   
v AB  – v BA or a BA  – a AB

Ex.44 Seeta is moving due east with a velocity of 1 m/s and Geeta is moving the due west with a
velocity of 2 m/s. What is the velocity of Seeta with respect to Geeta?
Sol. It is a one dimensional motion. So, let us choose the east direction as positive and the west as
negative. Now, given that
vs = velocity of Seeta = 1 m/s
and vG = velocity of Geeta = – 2m/s
Thus, vSG = velocity of Seeta with respect to Geeta
= vS – vG = 1 – (–2) = 3 m/s
Hence, velocity of Seeta with respect to Geeta is 3 m/s due east.

: 0744-2209671, 08003899588 | url : www.motioniitjee.com, : info@motioniitjee.com


Page # 156 KINEMATICS

IMPORTANT NOTE :
PROCEDURE TO SOLVE THE VECTOR EQUATION.
  
A BC ...(1)
(a) Their are 6 variables in this equation which are following :

(1) Magnitude of A and its direction

(2) Magnitude of B and its direction

(3) Magnitude of C and its direction.
(b) We can solve this equation if we know the value of 4 varibales [Note : two of them must be directions]
(c) If we know the two direction of any two vectors then we will put them on the same side and other on
the different side.
For example
  
If we know the directions of A and B and C' s direction is unknown then we make equation as follows : -
  
C  A –B
(d) Then we make vector diagram according to the equation and resolve the vectors to know the
unknown values.

Ex.45 Car A has an acceleration of 2 m/s2 due east and car B, 4 m/s2 due north. What is the acceleration
of car B with respect to car A?
Sol. It is a two dimensional motion. Therefore, N

a BA = acceleration of car B with respect to car A
  W E
= a B– a A

Here, a B = acceleration of car
S
B = 4 m/s2 (due north)

and a A = acceleration of car A = 2 m/s2 (due east)
 
 a BA a B  4m / s2
| a BA | (4)2  (2)2  2 5m / s2

4
and   tan–1   = tan–1(2)
2 
 
Thus, a BA is 2 5 m/s2 at an angle of  = tan–1(2) – a A  2m / s 2
from west towards north.

Ex.46 Three particle A, B and C situated at the vertices of an equilateral triangle starts moving simul-
taneously at a constant speed "v" in the direction of adjacent particle, which falls ahead in the
anti-clockwise direction. If "a" be the side of the triangle, then find the time when they meet.
A
Sol. Here, particle "A" follows "B", "B" follows "C" and "C" follows
"A". The direction of motion of each particle keeps chang-
ing as motion of each particle is always directed towards
other particle. The situation after a time "t" is shown in the
figure with a possible outline of path followed by the par- O
ticles before they
meet. B C

Corporate Head Office : Motion Education Pvt. Ltd., 394 - Rajeev Gandhi Nagar, Kota-5 (Raj.)
KINEMATICS Page # 157

This problem appears to be complex as the path of motion


is difficult to be defined. But, it has a simple solution in A
component analysis. Let us consider the pair "A" and "B".
The initial component of velocities in the direction of line v
joining the initial position of the two particles is "v" and
"vcos" as shown in the figure here :
The component velocities are directed towards eachother. v cos O v
Now, considering the linear (one dimensional) motion in the 60°
direction of AB, the relative velocity of "A" with respect to B C
v
"B" is :
vAB = vA – vB
vAB = v – (– v cos ) = v + vcos
In equilateral triangle,  = 60°
v 3v
vAB = v + vcos60° = v + =
2 2
The time taken to cover the displacement "a" i.e. the side of the triangle
2a
t=
3v
QUESTIONS BASED ON RELATIVE MOTION ARE USUALLY OF FOLLOWING FOUR TYPES :
(a) Minimum distance between two bodies in motion
(b) River-boat problems
(c) Aircraft-wind problems
(d) Rain problems
(a) Minimum distance between two bodies in motion

When two bodies are in motion, the questions like, the minimum distance between them or the time
when one body overtakes the other can be solved easily by the principle of relative motion. In these
type of problems one body is assumed to be at rest and the relative motion of the other body is
considered. By assuming so two body problem is converted into one body problem and the solution
becomes easy. Following example will illustrate the statement.

Ex.47 Car A and car B start moving simultaneously in the same direction along the line joining them.
Car A with a constant acceleration a = 4 m/s2, while car B moves with a constant velocity v = 1
m/s. At time t = 0, car A is 10 m behind car B. Find the time when car A overtakes car B.
Sol. Given : uA = 0, uB = 1 m/s, aA = 4m/s2 and aB = 0
Assuming car B to be at rest, we have
uAB = uA – uB = 0 – 1 = – 1 m/s
aAB = aA – aB = 4 – 0 = 4 m/s2
Now, the problem can be assumed in simplified form as follow :
2 2
a=4m/s v=1m/s
A 10m B

+ve
Substituting the proper values in equation
2
uAB= –1m/s, aAB= 4m/s
A 10m B

At rest

: 0744-2209671, 08003899588 | url : www.motioniitjee.com, : info@motioniitjee.com


Page # 158 KINEMATICS

1 2
s = ut + at
2
1
we get 10 = – t + (4)(t2 ) or 2t2 – t – 10 = 0
2
1  1  80 1  81 19
or t   or t = 2.5 s and – 2 s
4 4 4
Ignoring the negative value, the desired
time is 2.5s. Ans.
Not e : The above problem can also be solved without using the concept of relative motion as under.
At the time when A overtakes B,
sA = sB + 10
1
  4  t 2  1  t  10
2
or 2t2 – t – 10 = 0
Which on solving gives t = 2.5 s and – 2 s, the same as we found above.
As per my opinion, this approach (by taking absolute values) is more suitable in case of two body
problem in one dimensional motion. Let us see one more example in support of it.

Ex.48 An open lift is moving upwards with velocity 10m/s. It has an upward acceleration of 2m/s2. A
ball is projected upwards with velocity 20 m/s relative to ground. Find :
(a) time when ball again meets the lift.
(b) displacement of lift and ball at that instant.
(c) distance travelled by the ball upto that instant. Take g = 10 m/s2
Sol. (a) At the time when ball again meets the lift,
s L = sB
1 1 2m/s2 10m/s 20m/s
 10t + × 2 × t2 = 20 t – × 10t2
2 2 +ve
Solving this equation, we get Ball

5 10m/s2
t=0 and t s
3 L Lift B Ball

5
 Ball will again meet the lift after s.
3
(b) At this instant
2
5 1 5 175
sL  sB  10   2   m  19.4 m
3 2 3
  9

(c) For the ball u  a . Therefore, we will first find t0, the time when its velocity becomes zero.

u 20
t0    2s
a 10

 5 
As t   s   t0 , distance and displacement are equal
 3 
or d = 19.4 m Ans.
Concept of relative motion is more useful in two body problem in two (or three) dimensional motion.
This can be understood by the following example.

Corporate Head Office : Motion Education Pvt. Ltd., 394 - Rajeev Gandhi Nagar, Kota-5 (Raj.)
KINEMATICS Page # 159

Ex.49 Two ships A and B are 10 km apart on a line running south to north. Ship A farther north is
streaming west at 20 km/h and ship B is streaming north at 20km/h. What is their distance
of closest approach and how long do they take to reach it ?
Sol. Ships A and B are moving with same speed 20 km/h in
the directions shown in figure. It is a two dimensional, N
two body problem with zero acceleration. Let us find vA A E

vBA
   vB
vBA  vB  v A
 B
Here, | vBA | (20)2  (20)2 AB=10km

 20 2 km / h


i.e., vBA is 20 2 km / h at an angle of 45º from east
towards north. Thus, the given problem can be simplified as :
45º


A is at rest and B is moving with vBA in the direction shown in figure. A
Therefore, the minimum distance between the two is C
vBA
45º
smin = AC = AB sin 45º
B
 1 
 10   km = 5 2 km Ans.
 2
and the desired time is
BC 5 2
t   (BC = AC = 5 2 km )
| vBA | 20 2
1
 h  15 min Ans.
4

(B) River - Boat Problems

In river-boat problems we come across the following three terms :


B B
 v br cos
v br W y
 
A  x
vr vbr sin A
vr
 = absolute velocity of river
vr

v br = velocity of boatman with respect to river or velocity of boatman is still water
and  = absolute velocity of boatman.
vb
 
• Here, it is important to note that v br is the velocity of boatman with which he steers and v b is the
actual velocity of boatman relative to ground.
  
Further, v b = v br  v r
Now, let us derive some standard results and their special cases.

: 0744-2209671, 08003899588 | url : www.motioniitjee.com, : info@motioniitjee.com


Page # 160 KINEMATICS


A boatman starts from point A on one bank of a river with velocity v br in the direction shown in fig.

River is flowing along positive x-direction with velocity v r . Width of the river is w, then
  
v b = v br  v r
Therefore, vbx = vrx + vbrx = vr – vbr sin
and vby = vry + vbry
= 0 + vbr cos = vbr cos
Now, time taken by the boatman to cross the river is :
w w
t= 
v by v br cos 
w
or t = v cos  ...(i)
br
Further, displacement along x-axis when he reaches on the other bank (also called drift) is :
w
x = vbx t = (vr – vbr sin ) v cos 
br
w
or x = (vr – vbr sin) v cos  ...(ii)
br
Three special are :

(i) Condition when the boatman crosses the river in shortest interval of time
B
From Eq.(i) we can see that time (t) will be minimum when  = 0°,
i.e., the boatman should steer his boat perpendicular to the river 
current. v br
w
Also, tmin = v as cos  = 1 A 
br
vr
(ii) Condition when the boatman wants to reach point B, i.e., at a point just opposite from where
he started
In this case, the drift (x) should be zero.
 x=0 B
w 
or (vr – vbr sin) v cos   0 v br
br

or vr = vbr sin 
vr A 
–1  v  vr
or sin = v or  = sin  r 
br  v br 

–1  v 
Hence, to reach point B the boatman should row at an angle  = sin  r  upstream from AB.
 v br 
Further, since sin not greater than 1.
So, if vr  vbr, the boatman can never reach at point B. Because if vr = vbr, sin  = 1 or  = 90° and it is
just impossible to reach at B if  = 90°. Moreover it can be seen that vb = 0 if vr = vbr and  = 90°.
Similarly, if vr > vbr, sin > 1, i.e., no such angle exists. Practically it can be realized in this manner that
it is not possible to reach at B if river velocity (vr) is too high.
(iii) Shortest path
Path length travelled by the boatman when he reaches the opposite shore is
s= w 2  x2
Here, w = width of river is constant. So for s to be minimum modulus of x (drift) should be minimum.
Now two cases are possible.

Corporate Head Office : Motion Education Pvt. Ltd., 394 - Rajeev Gandhi Nagar, Kota-5 (Raj.)
KINEMATICS Page # 161

When vr < vbr : In this case x = 0,

–1  v r  –1  v r 
when  = sin  v  or smin = w at  = sin  
 br   v br 
dx
When vr > vbr : In this case x is minimum, where 0
d
d  w 
or  (vr – vbr sin )  0
d  vbr cos  
or –vbr cos2 – (vr – vbr sin) (– sin) = 0
or – vbr + vr sin = 0
 vbr 
or  = sin–1  v 
 r 
Now, at this angle we can find xmin and then smin which comes out to be
 vr  –1  vbr 
smin = w  v  at  = sin  v 
 br   r 

Ex.50 A man can row a boat with 4 km/h in still water. If he is crossing a river where the current is 2
km/h.
(a) In what direction will his boat be headed, if he wants to reach a point on the other bank, directly
opposite to starting point?
(b) If width of the river is 4 km, how long will the man take to cross the river, with the condition in
part (a)?
(c) In what direction should he head the boat if he wants to cross the river in shortest time and
what is this minimum time?
(d) How long will it take him to row 2 km up the stream and then back to his starting point ?
Sol. (a) Given, that vbr = 4 km/h and vr = 2 km/h

 vr  2 1
  = sin–1  v  = sin–1   = sin–1   = 30°
 br  4 2
Hence, to reach the point directly opposite to starting point he should head the boat at an angle of
30° with AB or 90° + 30° = 120° with the river flow.
(b) Time taken by the boatman to cross the river
w = width of river = 4 km
vbr = 4 km/h and  = 30°
4 2
 t= = h Ans.
4 cos 30 3
(c) For shortest time  = 0°
w 4
and tmin = v cos 0 = = 1h
br 4
Hence, he should head his boat perpendicular to the river current for crossing the river in shortest time
and this shortest time is 1 h.

vbr–vr vbr+vr
D C D C

(d) t = tCD + tDC


CD DC 2 2 1 4
or t  =   1  h Ans.
v db – vr vbr  v r 4–2 42 3 3

: 0744-2209671, 08003899588 | url : www.motioniitjee.com, : info@motioniitjee.com


Page # 162 KINEMATICS

Ex.51 A man can swim at a speed of 3 km/h in still water. He wants to cross a 500 m wide river
flowing at 2 kh/h. He keeps himself always at an angle of 120° with the river flow while swim-
ming.
(a) Find the time he takes to cross the river.
(b) At what point on the opposite bank will he arrive ?
Sol. The situation is shown in figure

Here vr,g = velocity of the river with respect to the ground
Y
 B C
vm,r = velocity of the man with respect to the river

vm,g = velocity of the man with respect to the ground.

,g
vm
(a) We have

°
  

30
vm,g  vm,r  v r,g ...(i) vm,r = 3km/h

Hence, the velocity with respect to the ground is along AC.
Taking y-components in equation (i), A vr,g = 2km/h
 3 3
vm,g sin = 3 km/h cos 30° + 2 km/h cos 90° = km/h
2
Time taken to cross the river
displacement along the Y - axis 1/ 2km 1
= =  h
velocity along the Y - axis 3 3 / 2 km / h 3 3
(b) Taking x-components in equation (i),
 1
vm,g cos = –3km/h sin 30° + 2 km/h  km / h
2
Displacement along the X-axis as the man crosses the river
= (velocity along the X-axis) (time)

 1km   1  1
=  2h  ×  h = km
3 3  6 3

Ex.52 A boat moves relative to water with a velocity v and river is flowing with 2v. At what angle the
boat shall move with the stream to have minimum drift?
(A) 30° (B) 60° (C) 90° (D) 120°
Sol. (D) Let boat move at angle  to the normal as shown in
1
figure then time to cross the river =
v cos 
ucos 
1 ub  u
drift x = (2v – v sin ) for x to be minimum I = width of river
v cos 
dx
= 0 = 1 (2 sec  tan  – sec2) or sin  = 1/2
d
u sin ur=2v
or  = 30° and  = 90 + 30 = 120°

(C) Aircraft Wind Problems


 
This is similar to river boat problem. The only difference is that v br is replaced by v aw (velocity of
 
aircraft with respect to wind or velocity of aircraft in still air), v r is replaced by v w (velocity of wind)

    
and v b is replaced by v a (absolute velocity of aircraft). Further,, v a = v aw  v . The following
w
example will illustrate the theory.

Corporate Head Office : Motion Education Pvt. Ltd., 394 - Rajeev Gandhi Nagar, Kota-5 (Raj.)
KINEMATICS Page # 163

NOTE : SHORT - TRICK 


A   
C  A B
 
If their are two vectors A and B and their resultent
 
make an anlge  with A and  with B . 
then A sin  =  sin  A sin  
 B
Means component of A perpendicular to resultant is equal in B sin

magnitude to the compopent of B also perpendicular to resultant.

  
Ex.53 If two vectors A and B make angle 30° and 60° B

with their resultent and B has magnitude equal to
 60°
10, then find magnitude of A . 30° 
Bsin60° A
So B sin 60° = A sin 30°
 10 sin 60° = A sin 30° A sin 30
 A = 10 3

Ex.54 An aircraft flies at 400 km/h in still air. A wind of 200 2 km/h is blowing from the south. The
pilot wishes to travel from A to a point B north east of A. Find the direction he must steer
and time of his journey if AB = 1000 km.
Sol. Given that vw = 200 2 km/h
 
vaw = 400 km/h and v a should be along AB or in north-east direction. Thus, the direction of v aw
 
should be such as the resultant of v w and v aw is along AB or in north - east direction.
 N
Let v aw makes an angle  with AB as shown in figure.
B
Applying sine law in triangle ABC, we get

AC

BC v a 45°  v w  200 2km / h
sin 45 sin 
 BC   200 2  1 1 45°  C
or 
sin  =  AC  sin 45° =  400    A v aw  400 km / h
  2 2 E
  = 30°
Therefore, the pilot should steer in a direction at an angle of (45° + ) or 75° from north towards
east.

| v a| 400  sin 105 km
Further, = or | v a | = sin 45 × (400) h
sin(180–45–30 ) sin 45

 cos 15  km  0.9659  km


=  sin 45  (400) =  0.707  (400)
h h
= 546.47 km/h
 The time of journey from A to B is
AB 1000
t=   h  t = 1.83 h
546.47
| v a|
(D) Rain Problems
  
In these type of problems we again come across three terms v r , vm and vrm , Here,

v r = velocity of rain

: 0744-2209671, 08003899588 | url : www.motioniitjee.com, : info@motioniitjee.com


Page # 164 KINEMATICS


vm = velocity of man (it may be velocity of cyclist or velocity of motorist also)


and vrm = velocity of rain with respect to man.


Here, v is the velocity of rain which appears to the man. Now, let us take one example of this.
rm

Ex.55 A man standing on a road has to hold his umbrella at 30° with the vertical to keep the rain
away. He throws the umbrella and starts running at 10 km/h. He finds that raindrops are hitting
his head vertically. Find the speed of raindrops with respect to (a) the road, (b) the moving man.
Sol. When the man is at rest with respect to the ground, the rain comes to him at an angle 30° with the
vertical. This is the direction of the velocity of raindrops with respect to the ground. The situation
when the man runs is shown in the figure

30° vm,g

30°

v r,m vr ,g
(b)
(a)


Here vr,g = velocity of the rain with respect to the ground
 
vm,g = velocity of the man with respect to the ground and vr,m = velocity of the rain with respect to
the man.
  
We have, vr,g  vr,m  vm,g ...(i)
Taking horizontal components, equation (i) gives

10 km / h
vr,g sin30° = um,g = 10 km/h or, v,g =  20km / h
sin30
Taking vertical components, equation (i) gives

3
vr,g cos30° = vr,m or, vr,m = (20 km/h) = 10  3 km/h.
2
Ex.56 To a man walking at the rate of 3 km/h the rain appears to fall vertically. When the increases
his speed to 6 km/h it appears to meet him at an angle of 45° with vertical. Find the speed of rain.
Sol. Let i and j be the unit vectors in horizontal and vertical directions respectively..
Let velocity of rain Vertical ( j)

v r  aiˆ  bj
ˆ ...(i)
Then speed of rain will be

| v r | a2  b2
Horizontal ( i )

Corporate Head Office : Motion Education Pvt. Ltd., 394 - Rajeev Gandhi Nagar, Kota-5 (Raj.)
KINEMATICS Page # 165


In the first case v m = velocity of man = 3i
  
 v rm  v r – v m  (a – 3)iˆ  bj
ˆ
It seems to be in vertical direction. Hence,
a – 3 = 0 or a = 3

In the second case v m = 6 i

 
 ˆ = – 3 i + b j
v rm  (a – 6)iˆ  bj
This seems to be at 45° with vertical.
Hence, |b| = 3
Therefore, from Eq. (ii) speed of rain is

| v r | (3)2  (3)2 = 3 2 km / h Ans.

Relative Motion between Two Projectiles


Let us now discuss the relative motion between two projectiles or the path observed by one projectile
of the other. Suppose that two particles are projected from the ground with speeds u1 and u2 at angles
1 and 2 as shown in Fig.A and B. Acceleration of both the particles is g downwards. So, relative
acceleration between them is zero because
a12 = a1 – a2 = g – g = 0

Y Y
u1 u2

1 2
X X
(A) (B)

i.e., the relative motion between the two particles is uniform. Now
u1x = u1 cos 1, u2x = u2 cos 2
u1y = u1 sin 1 and u2y = u2 sin 2
Therefore, u12x = u1x – u2x = u1 cos 1– u2cos 2 y
and u12y = u1y – u2y = u1 sin 1– u2 sin 2
u12x and u12y are the x and y components of relative
u12y
velocity of 1 with respect to 2.
Hence, relative motion of 1 with respect to 2 is a straight u12
a12=0

u  x
line at an angle   tan 1 12 y  with positive x-axis. u12x
 u12 x 

Now, if u12x = 0 or u1 cos 1 = u2 cos 2, the relative motion is along y-axis or in vertical direction
(as  = 90º). Similarly, if u12y = 0 or u1 sin 1 = u2 sin 2, the relative motion is along x-axis or in
horizontal direction (as  = 0º).
Note : Relative acceleration between two projectiles is zero. Relative motion between them is uniform.
Therefore, condition of collision of two particles in air is that relative velocity of one with respect to

the other should be along line joining them, i.e., if two projecticles A and B collide in mid air, then VAB

should be along AB or VBA along BA.

: 0744-2209671, 08003899588 | url : www.motioniitjee.com, : info@motioniitjee.com


Page # 166 KINEMATICS

Condition for collision of two projectiles : Consider the situation shown in the figure. For projectiles to
collide, direction of velocity of A with respect to B has to be along line AB.
Here, vABx = u1 cos 1 + u2 cos 2
u2
vABy = u1 sin 1 – u2 sin 2 B
Y
Let, direction of velocity vector of A(wrt B) is making an u1
h2
angle  with +ve X-axis, which is given by X
A
v ABy u1 sin 1  u2 sin 2 h1
tan   
v ABx u1 cos 1  u2 cos 2
x
For collision to take place,
h2  h1
tan  = tan  
x
Ex.57 A particle A is projected with an initial velocity of 60 m/s. at an angle 30º to the horizontal. At
the same time a second particle B is projected in opposite direction with initial speed of 50 m/s
from a point at a distance of 100 m from A. If the particles collide in air, find (a) the angle of
proj ect ion  of particle B, (b) time when the collision takes place and (c) the distance of P from A,
where collision occurs. (g = 10 m/s2)
60m/s 50m/s

30º
A B
100m
Sol. (a) Taking x and y directions as
shown in figure. Y
 
Here, A  gˆ , ˆ
a j a B  g j
uAx = 60 cos 30º = 30 3 m / s
uAy = 60 sin 30º = 30 m/s X
uBx = – 50 cos  u AB

and uBy = 50 sin 


 
Relative acceleration between the two is zero as a A  aB . Hence, the relative motion between the two

is uniform. It can be assumed that B is at rest and A is moving with u AB . Hence, the two particles will

collide, if u AB is along AB. This is possible only when
uAy = uBy
i.e., component of relative velocity along y-axis should be zero.
or 30 = 50 sin 
  = sin–1 (3/5) Ans.

  4
(b) Now, | uAB | uAx – uBx = (30 3 + 50 cos)m/s =  30 3  50  5  m / s = (30 3 + 40) m/s

Therefore, time of collision is

AB 100
t = | | = or t = 1.09 s Ans.
uAB 30 3  40
(c) Distance of point P from A where collision takes place is
2 2
 1   1 
s= (u Ax t)2   u Ay t – gt 2  = . )2   30  109
( 30 3  109 . –  10  109
.  109
.  or s = 62.64 m Ans.
 2   2 

Corporate Head Office : Motion Education Pvt. Ltd., 394 - Rajeev Gandhi Nagar, Kota-5 (Raj.)
KINEMATICS Page # 167

5m/s
Ex.58 Two projectile are projected simultaneously from a point on the A
ground "O" and an elevated position "A" respectively as shonw in the
figure. If collision occurs at the point of return of two projectiles on
H
the horizontal surface, then find the height of "A" above the ground 10m/s
and the angle at which the projectile "O" at the ground
should be projected.
 x
O C

Sol. There is no initial separation between two projectile is x-direction. For collision to occur, the relative
motion in x-direction should be zero. In other words, the component velocities in x-direction should be
equal to that two projetiles cover equal horizontal distance at any given time. Hence,
uOx = uAx

uA 5 1
 u0cos = uA  cos = u = = = cos60°   = 60°
O 10 2
We should ensure that collision does occur at the point of return. It means that by the time projectiles
travel horizontal distances required, they should also cover vertical distances so that both projectile are
at "C" at the same time. In the nutshell, their times of flight should be equal.
For projectile from "O".
2uO sin 
T=
g
For projectile from "A",
 2H 
T  
 g 
For projectile from "A",
2uo sin   2H 
T=   
g  g 
Squaring both sides and putting values,
4u2O sin2  4  102 sin2 60
 H=  H
2g 2  10
2
 3
H  20   15m
 2 
 
We have deliberately worked out this problem taking advantage of the fact that projectiles are colliding
at the end of their flights and hence their times of flight should be equal. We can, however, proceed to
analyze in typical manner, using concept of relative velocity. The initial separation between two projectiles
in the vertical direction is "H". This separation is covered with the component of relative in vertical
direction.
3
 vOAy = uOy – uAy = u0 sin60° – 0 = 10  = 5 3m/s
2
Now, time of flight of projectile from ground is :
2uO sin  2x10x sin 60
T= = = 3
g 10
Hence, the vertical displacement of projectile from "A" before collision is :

 H = vOAy X T = 5 3 x 3 = 15 m/s

: 0744-2209671, 08003899588 | url : www.motioniitjee.com, : info@motioniitjee.com


Page # 168 KINEMATICS

Ex.59 Two projectiles are projected simultaneously from two towers as shwon in figure. If the projectiles
collide in the air, then find the distance "s" between the towers.

10 m/s B

10 2 m / s

45° 30m
A

10m

Sol. We see here that projectiles are approaching both horizontally and vertically. Their movement in two
component directions should be synchronized so that they are at the same position at a particular given
time. For collision, the necessary requirement is that relative velocity and displacement should be in the
same direction.
It is given that collision does occur. It means that two projectiles should cover the displacement with
relative velocity in each of the component directions. Y 10 m/s B
In x-direction,

1 10 2 m / s
vABx = uAx – uBx = 10 2 cos 45 – (–10) = 10 2  10 = 20 m/s
2 30m
A 45°
If "t" is time after which collision occurs, then
 s = vAy – uBy 10m

1
 vABy = ucos45° – 0 = 10 2   10m / s O S x
2
The initial vertical distance between points of projection is 30 – 10 = 20 m. This vertical distance is
covered with component of relative velocity in vertical direction. Hence, time taken to collide, "t", is :

20
t= 2
10
Putting this value in the earlier equation for "s", we have :
 s = 20t = 20x2 = 40 m

Now you can try all the questions related to relative motion.

Corporate Head Office : Motion Education Pvt. Ltd., 394 - Rajeev Gandhi Nagar, Kota-5 (Raj.)
KINEMATICS Page # 169

QUESTIONS FOR SHORT ANSWER



1. A vector a is turned without a change in its length 5. Can you have zero displacement and a non zero
 average velocity? Can you have a zero displacement
through a small angle d. What are | a| and a?
and a non zero velocity? Illustrate your answer on a
Sol.
x-t graph.
Sol.

2. Does the speedometer of a car measure speed or


velocity ? Explain
6. At which point on its path a projectile has the
Sol.
smallest speed?
Sol.

7. A person standing on the edge of a cliff at some


3. When a particle moves with constant velocity, its
height above the ground below throws one ball straight
average velocity and its instantaneous velocity & speed
up with initial speed u and then throws another ball
are equal. Comment on this statement.
straight down with the same initial speed. Which ball,
Sol.
if either, has the larger speed when it hits the ground?
Neglect air resistance.
Sol.

4. In a given time interval, is the total displacement


8. An airplane on floor relief mission has to drop a
of a particle equal to the product of the average
sack of rice exactly in the center of a circle on the
velocity and the time interval, even when the velocity
ground while flying at a predetermined height and
is not constant? Explain.
speed. What is so difficult about that? Why doesn’t it
Sol.
just drop the sack when it is directly above the circle.
Sol.

: 0744-2209671, 08003899588 | url : www.motioniitjee.com, : info@motioniitjee.com


Page # 170 KINEMATICS

9. Which of the following graphs cannot possibily 12. Give an example from your own experience in which
represent one dimensional motion of a particle? the velocity of an object is zero for just an instant of
x  time, but its acceleration is not zero.
|v| l
Sol.
t t
t

x - displacement l - length of path


Sol.

13. A ball is dropped from rest from the top of a building


and strikes the ground with a speed vf. From ground
level, a second ball is thrown straight upward at the
10. Can you suggest a suitable situation from same instant that the first ball is dropped. The initial
observation around you for each of the following ? speed of the second ball is v0 = vf, the same speed
x with which the first ball will eventually strike the ground.
x
Ignoring air resistance, decide whether the balls cross
paths at half the height of the building above the
halfway point, or below the halfway point. Give your
t reasoning.
t Sol.
x - displacement
x

Sol.

14. The muzzle velocity of a gun is the velocity of the


bullet when it leaves the barrel. The muzzle velocity
of one rifle with a short barrel is greater than the
muzzle velocity of another rifle that has a longer barrel.
In which rifle is the acceleration of the bullet larger?
11. One of the following statements is incorrect. Explain your reasoning.
(a) The car traveled around the track at a constant Sol.
velocity
(b) The car traveled around the track at a constant
speed. Which statement is incorrect and why ?
Sol.

Corporate Head Office : Motion Education Pvt. Ltd., 394 - Rajeev Gandhi Nagar, Kota-5 (Raj.)
KINEMATICS Page # 171

15. On a riverboat cruise, a plastic bottle is accidentally 18. A child is playing on the floor of a recreational
dropped overboard. A passenger on the boat estimate vehicle (RV) as it moves along the highway at a
that the boat pulls ahead of the bottle by 5 meters constant velocity. He has a toy cannon, which shoots
each second. Is it possible to conclude that the boat a marble at a fixed angle and speed with respect to
is moving at 5 m/s with respect to the shore? Account the floor. The connon can be aimed toward the front
for your answer. or the rear of the RV. Is the range towards the front
Sol. the same as, less than, or greater than the range
towards the rear? Answer this question (a) from the
child’s point of view and (b) from the point of view of
an observer standing still on the ground. Justify your
answers.
Sol.

16. A wrench is accidentally dropped from the top of


the mast on a sailboat. Will the wrench hit at the
same place on the deck whether the sailboat is at
rest or moving with a constant velocity? Justify your
answer.
Sol.
19. Three swimmers can swim equally fast relative to
the water. They have a race to see who can swim
across a river in the least time. Swimmer A swims
perpendicular to the current and lands on the far shore
downstream, because the current has swept him in
that direction. Swimmer B swims upstream at an angle
to the current and lands on the far shore directly
opposite the starting point. Swimmer C swims
downstream at an angle to the current in an attempt
to take advantage of the current. Who crosses the
river in the least time? Account for your answer.
Sol.
17. Is the acceleration of a projectile equal to zero
when it reaches the top of its trajectory? If not, why
not?
Sol.

: 0744-2209671, 08003899588 | url : www.motioniitjee.com, : info@motioniitjee.com


Page # 172 KINEMATICS

Exercise - I OBJECTIVE PROBLEMS (JEE MAIN)


1. A particle covers half of the circle of radius r. Then 4. A car travels a distance of 2000 m. If the first half
the displacement and distance of the particle are distance is covered at 40 km/hour and the second
respectively - half at velocity v and if the average velocity is 48 km/
(A) 2r, 0 (B) 2r, r hour, then the value of v is -
(A) 56 km/hour (B) 60 km/hour
r
(C) , 2r (D) r, r (C) 50 km/hour (D) 48 km/hour
2
Sol.
Sol.

5. At an instant t , the co-ordinates of a particle are


x = at2, y = bt2 and z = 0 , then its velocity at the
instant t will be
2. A hall has the dimensions 10m × 10m × 10 m. A fly
starting at one corner ends up at a diagonally opposite (A) t a2  b2 (B) 2t a2  b2
corner. The magnitude of its displacement is nearly
(C) a2  b2 (D) 2t2 a2  b2
(A) 5 3 m (B) 10 3 m (C) 20 3 m (D) 30 3 m
Sol. Sol.

6. A car runs at constant speed on a circular track of


radius 100 m taking 62.8 s on each lap. What is the
average speed and average velocity on each complete
lap ?
3. A car travels from A to B at a speed of 20 km h–1 (A) velocity 10 m/s speed 10 m/s
and returns at a speed of 30 km h–1. The average (B) velocity zero, speed 10 m/s
speed of the car for the whole journey is (C) velocity zero, speed zero
(A) 5 km h–1 (B) 24 km h–1 (C) 25 km h–1(D) 50 km h–1 (D) velocity 10 m/s, speed zero
Sol. Sol.

Corporate Head Office : Motion Education Pvt. Ltd., 394 - Rajeev Gandhi Nagar, Kota-5 (Raj.)
KINEMATICS Page # 173
7. The displacement of a body is given by 2s = gt 2
Sol.
where g is a constant. The velocity of the body at
any time t is
(A) gt (B) gt/2 (C) gt2/2 (D) gt3/3
Sol.

11. The variation of velocity of a particle moving along


straight line is shown in the figure. The distance
travelled by the particle in 4 s is
8. A particle is moving so that its displacement s is
v(m/s)
given as s = t3- 6t2 + 3t + 4 meter. Its velocity at the
instant when its acceleration is zero will be -
(A) 3 m/s (B) -12 m/s 20
(C) 42 m/s (D) -9 m/s
10
Sol.
t(s)
1 2 3 4
(A) 25m (B) 30m (C) 55m (D) 60m
Sol.

9. A body starts from rest and is uniformly accelerated


for 30 s. The distance travelled in the first 10s is x1,
next 10 s is x2 and the last 10 s is x3. Then x1 : x2 : x3
is the same as
(A) 1 : 2 : 4 (B) 1 : 2 : 5
(C) 1 : 3 : 5 (D) 1 : 3 : 9
Sol.
12. The displacement time graphs of two particles A
and B are straight lines making angles of respectively
30º and 60º with the time axis. If the velocity of A is
vA
vA and that of B is vB then the value of v is
B

(A) 1/2 (B) 1 / 3 (C) 3 (D) 1/3


10. The displacement-time graph of a moving particle Sol.
is shown below. The instantaneous velocity of the
particle is negative at the point
x
D

E F
C
t
(A) C (B) D (C) E (D) F

: 0744-2209671, 08003899588 | url : www.motioniitjee.com, : info@motioniitjee.com


Page # 174 KINEMATICS
13. The v-t graph of a linear motion is shown in Sol.
adjoining figure. The distance from origin after 8
seconds is -

(A) 18 meters (B) 16 meters


(C) 8 meters (D) 6 meters
Sol. 16. If position time graph of a particle is sine curve as
shown, what will be its velocity-time graph

v v
14. The adjoining curve represents the velocity-time (A) (B)
graph of a particle, its acceleration values along OA, t t
AB and BC in metre/sec2 are respectively-
v
v
(C) (D)
t
t
Sol.

(A) 1, 0, -0.5 (B) 1, 0, 0.5


(C) 1, 1, 0.5 (D) 1, 0.5, 0
Sol.

17. A particle, after starting from rest , experiences,


constant acceleration for 20 seconds. If it covers a
15. In the following velocity-time graph of a body, distance of S1, in first 10 seconds and distance S2 in
the distance and displacement travelled by the body next 10 sec, then
in 5 second in meters will be - (A) S2 = S1/2 (B) S2 = S1
(C) S2 = 2S1 (D) S2 = 3S1
Sol.

(A) 75,115 (B) 105, 75


(C) 45, 75 (D) 95, 55

Corporate Head Office : Motion Education Pvt. Ltd., 394 - Rajeev Gandhi Nagar, Kota-5 (Raj.)
KINEMATICS Page # 175

21. A body is dropped from a height h under


acceleration due to gravity g. If t1 and t2 are time
intervals for its fall for first half and the second half
distance, the relation between them is
(A) t1 = t2 (B) t1 = 2t2
(C) t1 = 2.414 t2 (D) t1 = 4t2
18. A body sliding on a smooth inclined plane requires Sol.
4sec to reach the bottom after starting from rest at
the top. How much time does it take to cover one
fourth the distance starting from the top
(A) 1sec (B) 2 sec
(C) 0.4sec (D) 1.6 sec
Sol.

22. Two bodies of different masses ma and mb are


dropped from two different heights, viz a and b. The
ratio of times taken by the two to drop through these
distances is
ma b
19. The initial velocity of a particle is 10 m/sec and (A) a : b (B) m : a
b
its retardation is 2 m/sec2. The distance covered in
the fifth second of the motion will be (C) a: b (D) a2 : b2
(A) 1m (B) 19m Sol.
(C) 50m (D) 75m
Sol.

23. A body is thrown upward and reaches its maximum


20. A body starts from rest, the ratio of distances height. At that position-
travelled by the body during 3rd and 4th seconds is : (A) its velocity is zero and its acceleration is also zero
(A) 7/5 (B) 5/7 (B) its velocity is zero but its acceleration is maximum
(C) 7/3 (D) 3/7 (C) its acceleration is minimum
(D) its velocity is zero and its acceleration is the
Sol.
acceleration due to gravity
Sol.

: 0744-2209671, 08003899588 | url : www.motioniitjee.com, : info@motioniitjee.com


Page # 176 KINEMATICS
24. Two trains each of length 50 m are approaching 27. A ball is thrown upwards. It returns to ground
each other on parallel rails. Their velocities are 10 m/ describing a parabolic path. Which of the following
sec and 15 m/sec. They will cross each other in - remains constant ?
(A) 2 sec (B) 4 sec (A) speed of the ball
(C) 10 sec (D) 6 sec (B) kinetic energy of the ball
Sol. (C) vertical component of velocity
(D) horizontal component of velocity.
Sol.

25. A car A is going north-east at 80 km/hr and another 28. The angle of projection of a body is 15º . The
car B is going south-east at 60 km/hr. Then the
other angle for which the range is the same as the
direction of the velocity of A relative to B makes with
the north an anglesuch that tan  is - first one is equal to-
(A) 1/7 (B) 3/4 (A) 30º (B) 45º
(C) 4/3 (D) 3/5 (C) 60º (D) 75º
Sol. Sol.

29. A ball is thrown at an angle of 45º with the


26. An object A is moving with 10 m/s and B is moving horizontal with kinetic energy E. The kinetic energy
with 5 m/s in the same direction of positive x-axis. A at the highest point during the flight is-
is 100 m behind B as shown. Find time taken by A to (A) Zero (B) E/2
Meet B (C) E (D) (2)1/2E
10m/s 5m/s Sol.
A B

100m
(A) 18 sec.(B) 16 sec. (C) 20 sec. (D) 17 sec.
Sol.

30. The maximum range of a projectile is 22 m.


When it is thrown at an angle of 15º with the
horizontal, its range will be-
(A) 22 m (B) 6 m
(C) 15 m (D) 11 m

Corporate Head Office : Motion Education Pvt. Ltd., 394 - Rajeev Gandhi Nagar, Kota-5 (Raj.)
KINEMATICS Page # 177
Sol. Sol.

34. A marble A is dropped vertically, another


identical marble B is projected horizontally from the
same point at the same instant
(A) A will reach the ground earlier than B
5x 2 (B) B will reach the ground earlier than A
31. The equation of projectile is y = 16x – .
4 (C) both A and B will reach the ground at the same
The horizontal range is- instant
(A) 16 m (B) 8 m (D) none of the above
(C) 3.2 m (D) 12.8 m Sol.
Sol.

35. An aeroplane is flying at a height of 1960


m in horizontal direction with a velocity of 360 km/
hr. When it is vertically above the point. A on the
ground, it drops a bomb. The bomb strikes a point
32. If four balls A, B, C, D are projected B on the ground, then the time taken by the bomb
with same speed at angles of 15º, 30º, 45º to reach the ground is-
and 60º with the horizontal respectively, the (A) 20 2 sec (B) 20 sec
two balls which will fall at the same place
(C) 10 2 sec (D) 10 sec
will be-
(A) A and B (B) A and D Sol.
(C) B and D (D) A and C
Sol.

36.A ball is hit by a batsman at an angle of 37º as


shown in figure. The man standing at P should run at
what minimum velocity so that he catches the ball
before it strikes the ground. Assume that height of
man is negligible in comparison to maximum height of
projectile.
33. The velocity at the maximum height of a projectile
is half of its initial velocity u. Its range on the horizontal
plane is
2u2 3 u2 u2 u2
(A) (B) (C) (D)
3g 2g 3g 2g
(A) 3 ms–1 (B) 5 ms–1 (C) 9 ms–1 (D) 12 ms–1

: 0744-2209671, 08003899588 | url : www.motioniitjee.com, : info@motioniitjee.com


Page # 178 KINEMATICS
Sol. 40. From the top of a tower of height h
a body of mass m is projected in the horizontal
direction with a velocity v, it falls on the
ground at a distance x from the tower. If
a body of mass 2 m is projected from the
top of another tower of height 2h in the horizontal
direction so that it falls on the ground at
a distance 2x from the tower, the horizontal
velocity of the second body is-

(A) 2v (B) 2v
37. Two projectiles A and B are thrown with the same
speed such that A makes angle  with the horizontal v v
(C) (D)
and B makes angle  with the vertical, then - 2 2
(A) Both must have same time of flight Sol.
(B) Both must achieve same maximum height
(C) A must have more horizontal range than B
(D) Both may have same time of flight
Sol.

41. One stone is projected horizontally from a 20 m


38. Suppose a player hits several baseballs. Which high cliff with an initial speed of 10 ms–1. A second
baseball will be in the air for the longest time? stone is simultaneously dropped from that cliff. Which
(A) The one with the farthest range. of the following is true ?
(B) The one which reaches maximum height (A) Both strike the ground with the same velocity
(C) The one with the greatest initial velocity (B) The ball with initial speed 10ms–1 reaches the
(D) The one leaving the bat at 45° with respect to ground first
the ground. (C) Both the balls hit the ground at the same time
Sol. (D) One cannot say without knowing the height of
the building
Sol.

39. A ball is projected from top of a tower with a


velocity of 5 m/s at an angle of 53º to horizontal. Its
speed when it is at a height of 0.45 m from the point 42. An aeroplane flying at a constant velocity releases
of projection is a bomb. As the bomb drops down from the aeroplane.
(A) 2 m/s (B) 3 m/s (A) it will always be vertically below the aeroplane
(C) 4 m/s (D) data insufficient (B) it will always be vertically below the aeroplane
Sol. only if the aeroplane is flying horizontally
(C) it will always be vertically below the aeroplane
only if the aeroplane is flying at an angle of 45° to the
horizontal.
(D) it will gradually fall behind the aeroplane if the
aeroplane is flying horizontally

Corporate Head Office : Motion Education Pvt. Ltd., 394 - Rajeev Gandhi Nagar, Kota-5 (Raj.)
KINEMATICS Page # 179
Sol. 45. A swimmer’s speed in the direction of flow of river
is 16 km h–1. Against the direction of flow of river, the
swimmer’s speed is 8 km h–1. Calculate the swimmer’s
speed in still water and the velocity of flow of the
river.
(A) 12 km/h, 4 km/h (B) 10 km/h, 3 km/h
(C) 10 km/h, 4 km/h (D) 12 km/h, 2 km/h
Sol.

43. A helicopter is flying south with a speed of 50


kmh–1. A train is moving with the same speed towards
east. The relative velocity of the helicopter as seen
by the passengers in the train will be towards.
(A) north east (B) south east 46. A man is walking on a road with a velocity
(C) north west (D) south west 3 km/hr. Suddenly rain starts falling. The velocity of
Sol. rain is 10 km/hr in vertically downward direction. The
relative velocity of the rain is -
(A) 13 km/hr (B) 7 km/hr

(C) 109 km/hr (D) 13 km/hr


Sol.

44. Two particles are moving with velocities v1 and


v2. Their relative velocity is the maximum, when the
angle between their velocities is
(A) zero (B) /4 (C) /2 (D) 
Sol.

: 0744-2209671, 08003899588 | url : www.motioniitjee.com, : info@motioniitjee.com


Page # 180 KINEMATICS

Exercise - II
(SINGLE CORRECT) 4. The co-ordinates of a moving particle at a time t,
1. A body covers first 1/3 part of its journey with a are given by, x = 5 sin 10 t, y = 5 cos 10 t. The speed
velocity of 2 m/s, next 1/3 part with a velocity of 3 m/s of the particle is -
and rest of the journey with a velocity 6m/s. The (A) 25 (B) 50 (C) 10 (D) None
average velocity of the body will be Sol.
11 8 4
(A) 3 m/s (B) m/s (C) m/s (D) m/ s
3 3 3
Sol.

5. A body moves with velocity v = lnx m/s where x is


its position. The net force acting on body is zero at .
(A) 0 m (B) x = e2m (C) x = em (D) x = 1 m
2. A particle has a velocity u towards east at t = 0. Sol.
Its acceleration is towards west and is constant, Let
xA and xB be the magnitude of displacements in the
first 10 seconds and the next 10 seconds.
(A) xA < xB (B) xA = xB (C) xA > xB
(D) The information is insufficient to decide the relation
of xA with xB.
Sol.

6. A body of mass 1 kg is acted upon by a force



F  2 sin 3 ti + 3 cos 3 t j find its position at t = 1 sec
if at t = 0 it is at rest at origin.
3. A stone is dropped into a well in which the level of
 3 3   2 2 
water is h below the top of the well. If v is velocity of (A)  2 ,  (B)  2 , 
 3 92   3 32 
sound, the time T after which the splash is heard is
given by
 2 2 
2h h (C)  ,  (D) none of these
 3 32 
(A) T = 2h/v (B) T  g

v
Sol.
2h h h 2h
(C) T   (D) T  
g 2v 2g v
Sol.

Corporate Head Office : Motion Education Pvt. Ltd., 394 - Rajeev Gandhi Nagar, Kota-5 (Raj.)
KINEMATICS Page # 181
7. A force F = Be acts on a particle whose mass is
–Ct
10. A particle is projected vertically upwards from a
m and whose velocity is 0 at t = 0. It’s terminal velocity point A on the ground. It takes t1 time to reach a
(velocity after a long time) is : point B but it still continues to move up. If it takes
further t2 time to reach the ground from point B then
C B BC B
(A) (B) (C) (D) – height of point B from the ground is
mB mC m mC
1
Sol. (A) g( t1  t 2 ) 2 (B) g t1 t2
2
1 1
(C) g( t1  t 2 )2 (D) gt 1 t 2
8 2
Sol.

8. A particle starts moving rectilinearly at time t = 0


such that its velocity ‘v’ changes with time ‘t’ according
to the equation v = t2 – t where t is in seconds and v
is in m/s. The time interval for which the particle retards
is
(A) t < 1/2 (B) 1/2 < t < 1
(C) t > 1 (D) t < 1/2 and t > 1
Sol.
11. Balls are thrown vertically upward in such a way
that the next ball is thrown when the previous one is
at the maximum height. If the maximum height is 5m,
the number of balls thrown per minute will be
(A) 40 (B) 50 (C) 60 (D) 120
Sol.

9. A ball is thrown vertically down with velocity of


5m/s. With what velocity should another ball be thrown
down after 2 seconds so that it can hit the 1st ball in 2
seconds
(A) 40 m/s (B) 55 m/s (C) 15 m/s (D) 25 m/s
Sol.

12. A disc arranged in a vertical plane has two groves


of same length directed along the vertical chord AB
and CD as shown in the fig. The same particles slide
down along AB and CD. The ratio of the time tAB/tCD is

A C
60º
D

(A) 1 : 2 (B) 1: 2 (C) 2 : 1 (D) 2 :1

: 0744-2209671, 08003899588 | url : www.motioniitjee.com, : info@motioniitjee.com


Page # 182 KINEMATICS
Sol. 14. A man moves in x - y plane along the path shown.
At what point is his average velocity vector in the
same direction as his instantaneous velocity vector.
The man starts from point P.
y
C
PB D
A
x
(A) A (B) B (C) C (D) D
Sol.

13. Acceleration versus velocity graph of a particle


moving in a straight line starting form rest is as shown
in figure. The corresponding velocity-time graph would
be -
a
15. The acceleration of a particle which moves along
the positive x-axis varies with its position as shown.
If the velocity of the particle is 0.8 m/s at x = 0, the
v
velocity of the particle at x = 1.4 is (in m/s)
2
v v a (in m/s )

0.4
(A) (B)
0.2
t t
v v O 0.4 0.8 1.4 x (in m)

(A) 1.6 (B) 1.2 (C) 1.4 (D) none


(C) (D)
Sol.
t t
Sol.

Corporate Head Office : Motion Education Pvt. Ltd., 394 - Rajeev Gandhi Nagar, Kota-5 (Raj.)
KINEMATICS Page # 183
Question No. 16 to 18 (3 questions) 18. The speed-time graph of the particle is correctly
shown by
The x-t graph of a particle moving along a straight
line is shown in figure speed speed

(A) 0 (B) 0
x parabola T 2T t T 2T t

speed speed
0 T 2T 0 0
(C) T 2T t (D) T 2T t
16. The v-t graph of the particle is correctly shown
by
Sol.
v
v
T 2T
0 T 0
(A) 2T t (B) t

v v

(C) 0 T 2T t (D) 0 T 2T t

Sol.

Question No. 19 to 24 (6 questions)


The figure shows a velocity-time graph of a particle
moving along a straight line

v(ms–1)
17. The a-t graph of the particle is correctly shown 10
by
0
a a 2 4 6 8 t(s)
2T
(A)
0 t (B)
0 t –20
T
19. Choose the incorrect statement. The particle comes
to rest at
a v
(A) t = 0 s (B) t = 5 s (C) t = 8 s
(D) none of these
(C)
0 t (D)
0 t Sol.

Sol.

: 0744-2209671, 08003899588 | url : www.motioniitjee.com, : info@motioniitjee.com


Page # 184 KINEMATICS
20. Identify the region in which the rate of change of 23. The total distance travelled by the particle is
 (A) 66.6 m (B) 51.6 m (C) zero (D) 36.6 m
v
velocity of the particle is maximum Sol.
t
(A) 0 to 2s (B) 2 to 4s (C) 4 to 6s (D) 6 to 8 s
Sol.

21. If the particle starts from the position x0 = –15 m, 24. The correct displacement-time graph of the particle
then its position at t = 2s will be is shown as
(A) – 5m (B) 5m (C) 10 m (D) 15 m x x
Sol. (m) (m)

(A) (B)
0 2 4 6 8 t(s) 0 2 4 6 8 t(s)
x x
(m) (m)

(C) (D)
0 2 4 6 8 t(s) 0 2 4 6 8 t(s)
Sol.

22. The maximum of displacement of the particle is


(A) 33.3 m (B) 23.3 m (C) 18.3 (D) zero
Sol.

25. The velocity-time graph of a body falling from


rest under gravity and rebounding from a solid surface
is represented by which of the following graphs ?
V
V

(A) t (B)
t

V V

(C) (D)
t t

Corporate Head Office : Motion Education Pvt. Ltd., 394 - Rajeev Gandhi Nagar, Kota-5 (Raj.)
KINEMATICS Page # 185
Sol. Sol.

28. A body A is thrown vertically upwards with such a


velocity that it reaches a maximum height of h.
Simultaneously another body B is dropped from height
h. It strikes the ground and does not rebound. The
velocity of A relative to B v/s time graph is best
26. Shown in the figure are the displacement time represented by : (upward direction is positive)
graph for two children going home from the school.
Which of the following statements about their relative VAB VAB
motion is true after both of them started moving ?
(A) (B)
Their relative velocity:
X t t
C1
VAB VAB
C2
(C) (D) t
t
O t T
(A) first increases and then decreases Sol.
(B) first decreases and then increases
(C) is zero
(D) is non zero constant
Sol.

29. It takes one minute for a passenger standing on


an escalator to reach the top. If the escalator does
not move it takes him 3 minute to walk up. How long
will it take for the passenger to arrive at the top if he
walks up the moving escalator?
(A) 30 sec (B) 45 sec (C) 40 sec (D) 35 sec
Sol.
27. Shown in the figure are the v
P1
velocity time graphs of the two
particles P1 and P2. Which of P2
the following statements about
their relative motion is true ?
Theire relative velocity
(A) is zero O t T
(B) is non-zero but constant
(C) continuously decreases
(D) continuously increases

: 0744-2209671, 08003899588 | url : www.motioniitjee.com, : info@motioniitjee.com


Page # 186 KINEMATICS
30. A body is thrown up in a lift with a velocity u Sol.
relative to the lift and the time of flight is found to be
t. The acceleration with which the lift is moving up is

u – gt 2u – gt u  gt 2u  gt
(A) (B) (C) (D)
t t t t
Sol.

34. The equation of the trajectory is given by


(A) 180y = 240 x – x2 (B) 180 y = x2 – 240x
(C) 180y = 135x – x2 (D) 180y = x2 – 135x
Sol.

31. A point mass is projected, making an acute angle



with the horizontal. If angle between velocity v and

acceleration g is , then  is given by
(A) 0º <  < 90º (B)  = 90º
(C)  = 90º (D) 0º <  < 180º
Sol.

35. A particle is projected from a horizontal plane (x-


z plane) such that its velocity vector at time t is

given by V  a î  (b – ct ) ĵ . Its range on the horizontal
plane is given by
Question No. 32 to 34 ba 2ba 3 ba
(A) (B) (C) (D) None
A projectile is thrown with a velocity of 50 ms–1 at an c c c
angle of 53º with the horizontal Sol.

32. Choose the incorrect statement


(A) It travels vertically with a velocity of 40 ms–1
(B) It travels horizontally with a velocity of 30 ms–1
(C) The minimum velocity of the projectile is 30 ms–1
(D) None of these
Sol.

36. A ball is thrown from a point on ground at some


angle of projection. At the same time a bird starts
from a point directly above this point of projection at
a height h horizontally with speed u. Given that in its
flight ball just touches the bird at one point. Find the
distance on ground where ball strikes
33. Determine the instants at which the projectile is
at the same height h 2h 2h h
(A) t = 1s and t = 7s (B) t = 3s and t = 5s (A) 2u (B) u (C) 2u (D) u
g g g g
(C) t = 2s and t = 6s (D) all the above

Corporate Head Office : Motion Education Pvt. Ltd., 394 - Rajeev Gandhi Nagar, Kota-5 (Raj.)
KINEMATICS Page # 187
Sol. 39. The distance (in metres) of the particle from origin
at t = 2 sec.
(A) 60 2 (B) 100 (C) 60 (D) 120
Sol.

37. A projectile is fired with a speed u at an angle 


with the horizontal. Its speed when its direction of
motion makes an angle ‘’ with the horizontal is -
(A) u sec cos (B) u sec sin
(C) u cos sec (D) u sin sec
40. Particle is dropped from the height of 20 m from
Sol.
horizontal ground. There is wind blowing due to which
horizontal acceleration of the particle becomes 6 ms–2.
Find the horizontal displacement of the particle till it
reaches ground.
(A) 6 m (B) 10 m (C) 12 m (D) 24 m
Sol.

Question No. 38 & 39 (2 questions)


At t = 0 a projectile is fired from a point O (taken as
origin) on the ground with a speed of 50 m/s at an
angle of 53° with the horizontal. It just passes two
points A & B each at height 75 m above horizontal as
shown.
41. Find time of flight of projectile thrown horizontally
with speed 10 ms–1 from a long inclined plane which
50m/s

makes an angle of  = 45º from horizontal.


A B
(A) 2 sec (B) 2 2 sec (C) 2 sec (D) none
75m Sol.
53°
O
38. The horizontal separation between the points A
and B is -
(A) 30 m (B) 60 m (C) 90 m (D) None
Sol.

42 A particle is projected at angle 37º with the incline


plane in upward direction with speed 10 m/s. The angle
of incline plane is given 53º. Then the maximum height
above the incline plane attained by the particle will be
(A) 3m (B) 4m (C) 5m (D) zero

: 0744-2209671, 08003899588 | url : www.motioniitjee.com, : info@motioniitjee.com


Page # 188 KINEMATICS
Sol. 45. If time taken by the projectile to reach Q is T,
than PQ =

v
90° P


Q
(A) Tvsin (B) Tvcos (C) Tv sec (D) Tv tan
Sol.
43. On an inclined plane of inclination 30°, a ball is
thrown at an angle of 60° with the horizontal from the
foot of the incline with velocity of 10 3 ms–1. If g =
10 ms–2, then the time in which ball with hit the inclined
plane is -
(A) 1.15 sec. (B) 6 sec
(C) 2 sec (D) 0.92 sec
Sol.

Question No. 46 to 49 (4 questions)


Two projectiles are thrown simultaneously in
the same plane from the same point. If their velocities
are v1 and v2 at angles 1 and 2 respectively from the
horizontal, then answer the following questions.
44. A projectile is fired with a velocity at right angle 46. The trajectory of particle 1 with respect to particle
to the slope which is inclined at an angle  with the 2 will be
horizontal. The expression for the range R along the (A) a parabola
incline is - (B) a straight line
(C) a vertical straight line
2v 2 2v 2
(A) sec  (B) tan (D) a horizontal straight line
g g Sol.
2v 2 v2
(C) tan  sec  (D) tan2 
g g
Sol.

47. If v1 cos1 = v2 cos2, then choose the incorrect


statement
(A) one particle will remain exactly below or above
the other particle
(B) the trajectory of one with respect to other will be
a vertical straight line
(C) both will have the same range
(D) none of these

Corporate Head Office : Motion Education Pvt. Ltd., 394 - Rajeev Gandhi Nagar, Kota-5 (Raj.)
KINEMATICS Page # 189
Sol. 50. A ship is travelling due east at 10 km/h. A ship
heading 30º east of north is always due north from
the first ship. The speed of the second ship in km/h is
(A) 20 2 (B) 20 3 / 2 (C) 20 (D) 20 / 2
Sol.

48. If v1sin1 = v2sin2, then choose the incorrect


statement
(A) the time of flight of both the particles will be same
(B) the maximum height attained by the particles will 51. A particle is kept at rest at origin. Another particle
be same
(C) the trajectory of one with respect to another will starts from (5, 0) with a velocity of – 4 î  3 ĵ . Find
be a horizontal straight line their closest distance of approach.
(D) none of these (A) 3 m (B) 4 m (C) 5 m (D) 2 m
Sol. Sol.

49. If v1 = v2 and 1 > 2, then choose the incorrect


statement
(A) Particle 2 moves under the particle 1
(B) The slope of the trajectory of particle 2 with
respect to 1 is always positive 52. Four particles situated at the corners of a square
(C) Both the particle will have the same range if 1 > of side ‘a’ move at a constant speed v. Each particle
45° and 2 < 45° and 1 + 2 = 90° maintains a direction towards the next particle in
(D) none of these succession. Calculate the time particles will take to
Sol. meet each other.
a a a 2a
(A) (B) (C) (D)
v 2v 3v 3v
Sol.

: 0744-2209671, 08003899588 | url : www.motioniitjee.com, : info@motioniitjee.com


Page # 190 KINEMATICS
53. A pipe which can rotate in a vertical plane is 55. A boat having a speed of 5 km/hr. in still water,
mounted on a cart. The cart moves uniformly along a crosses a river of width 1 km along the shortest possible
horizontal path with a speed v1 = 2 m/s. At what path in 15 minutes. The speed of the river in Km/hr.
angle  to the horizontal should the pipe be placed so (A) 1 (B) 3 (C) 4 (D) 41
that drops of rain falling with a velocity v2 6 m/s move
parallel to the walls of the pipe without touching them ? Sol.
consider the velocity of the drops as constant due to
the resistance of the air.

v1

–1  1 
(A) tan –1( 3) (B) tan  3 
56. A flag is mounted on a car moving due North with
–1 1  velocity of 20 km/hr. Strong winds are blowing due
(C) tan  2  (D) None of these East with velocity of 20 km/hr. The flag will point it
direction
Sol.
(A) East (B) North-East
(C) South-East (D) South-West
Sol.

54. A swimmer swims in still water at a speed = 5 km/


hr. He enters a 200 m wide river, having river flow speed
= 4 km/hr at point A and proceeds to swim at an angle
of 127° (sin37° = 0.6) with the river flow direction.
Another point B is located directly across A on the
other side. The swimmer lands on the other bank at a
57. A man is crossing a river flowing with velocity of 5
point C, from which he walks the distance CB with a
m/s. He reaches a point directly across at a distance
speed = 3 km/hr. The total time in which he reachrs
of 60 m in 5 sec. His velocity in still water should be
from A to B is
(A) 12 m/s (B) 13 m/s (C) 5 m/s (D) 10 m/s
(A) 5 minutes (B) 4 minutes
Sol.
(C) 3 minutes (D) None
Sol.

Corporate Head Office : Motion Education Pvt. Ltd., 394 - Rajeev Gandhi Nagar, Kota-5 (Raj.)
KINEMATICS Page # 191
58. Wind is blowing in the north direction at speed of Sol.
2 m/s which causes the rain to fall at some angle with
the vertical. With what velocity should a cyclist drive
so that the rain appears vertical to him
(A) 2 m/s south (B) 2 m/s north
(C) 4 m/s west (D) 4 m/s south

61. Mark the correct statements for a particle going


on a straight line
(A) if the veloci ty is zero at any instant, the
acceleration should also be zero at that instant
(B) if the velocity is zero for a time interval, the
acceleration is zero at any instant within the time
(MULTIPLE CORRECT) interval
59. The displacement x of a particle depend on time t (C) if the velocity and acceleration have opposite sign,
as x = t2 – t3 the object is slowing down
(A) particle will return to its starting point after time /. (D) if the position and velocity have opposite sign,
2 the particle is moving towards the origin.
(B) the particle will come to rest after time Sol.
3
(C) the initial velocity of the particle was zero but its
initial acceleration was not zero.

(D) no net force act on the particle at time
3
Sol.

62. A particle initially at rest is subjected to two forces.


One is constant, the other is a retarding force
proportion at to the particle velocity. In the subsequent
motion of the particle.
(A) the acceleration will increase from zero to a
constant value
(B) the acceleration will decrease from its initial value
to zero
(C) the velocity will increase from zero to maximum &
then decrease
(D) the velocity will increase from zero to a constant
value.
Sol.

60. A particle has intial velocity 10 m/s. It moves due


to constant retarding force along the line of velocity
which produces a retardation of 5 m/s2. Then -
(A) the maximum displacement in the direction of initial
velocity is 10 m
(B) the distance travelled in first 3 seconds is 7.5 m
(C) the distance travelled in first 3 seconds is 12.5 m
(D) the distance travelled in first 3 seconds is 17.5 m

: 0744-2209671, 08003899588 | url : www.motioniitjee.com, : info@motioniitjee.com


Page # 192 KINEMATICS
  Sol.
63. Let v and a denote the velocity and acceleration
respectively of a body in one-dimensional motion
 
(A) | v| must decrease when a  0

(B) Speed must increase when a  0
 
(C) Speed will increase when both v and a are < 0
 
(D) Speed will decrease when v  0 and a  0
Sol.

66. A bead is free to slide down a A


sm ooth wi re ti g ht l y st ret ched
between points A and B on a vertical 
B R
circle. If the bead starts from rest
at A, the highest point on the circle
(A) its velocity v on arriving at B is proportional to
cos
64. Which of the following statements are true for a (B) its velocity v on arriving B is proportional to tan
moving body? (C) time to arrive at B is proportional to cos
(A) If its speed changes, its velocity must change (D) time to arrive at B is independent of 
and it must have some acceleration Sol.
(B) If its velocity changes, its speed must change
and it must have some acceleration
(C) If its velocity changes, its speed may or may not
change, and it must have some acceleration
(D) If its speed changes but direction of motion does
not changes, its velocity may remain constant
Sol.

67. Velocity-time graph for a car is semicircle as shown


here. Which of the following is correct :
v

65. Let v and a denote the velocity and acceleration 1m/s


respectively of a body
(A) a can be non zero when v = 0
2 sec
(B) a must be zero when v = 0 (A) Car must move in circular path
(C) a may be zero when v  0 (B) Acceleration of car is never zero
(D) The direction of a must have some correlation (C) Mean speed of the particle is /4 m/s.
with the direction of v (D) The car makes a turn once during its motion

Corporate Head Office : Motion Education Pvt. Ltd., 394 - Rajeev Gandhi Nagar, Kota-5 (Raj.)
KINEMATICS Page # 193
Sol. Sol.

68. The figure shows the velocity (v) of a particle


plotted against time (t)

+v0
v 70. A particle moves with constant speed v along a
T regular hexagon ABCDEF in the same order. Then the
O magnitude of the average velocity for its motion from
t 2T
–v0 A to -
(A) F is v/5 (B) D is v/3
(A) The particle changes its direction of motion at (C) C is v 3/2 (D) B is v
some point Sol.
(B) The acceleration of the particle remains constant
(C) The displacement of the particle is zero
(D) The initial and final speeds of the particle are the
same
Sol.

71. An observer moves with a constant speed along


the line joining two stationary objects. He will observe
that the two objects
(A) have the same speed
(B) have the same velocity
(C) move in the same direction
69. A block is thrown with a velocity of 2 ms–1 (relative (D) move in opposite directions
to ground) on a belt, which is moving with velocity 4 Sol.
ms–1 in opposite direction of the initial velocity of block.
If the block stops slipping on the belt after 4 sec of
the throwing then choose the correct statements(s)
(A) Displacement with respect to ground is zero after
2.66 sec and magnitude of displacement with respect
to ground is 12 m after 4 sec.
(B) Magnitude of displacement with respect to ground
in 4 sec is 4 m.
(C) Magnitude of displacement with respect to belt in
4 sec is 12 m.
(D) Displacement with respect to ground is zero in 8/
3 sec.

: 0744-2209671, 08003899588 | url : www.motioniitjee.com, : info@motioniitjee.com


Page # 194 KINEMATICS
72. A man on a rectilinearly moving cart, facing the 74. Choose the correct alternative (s)
direction of motion, throws a ball straight up with (A) If the greatest height to which a man can throw a
respect to himself stone is h, then the greatest horizontal distance upto
(A) The ball will always return to him which he can throw the stone is 2h.
(B) The ball will never return to him (B) The angle of projection for a projectile motion whose
(C) The ball will return to him if the cart moves with range R is n times the maximum height is tan–1(4/n)
constant velocity (C) The time of flight T and the horizontal range R of
(D) The ball will fall behind him if the cart moves with a projectile are connected by the equation gT2 =
some acceleration 2Rtan where  is the angle of projection.
Sol. (D) A ball is thrown vertically up. Another ball is thrown
at an angle  with the vertical. Both of them remain in
air for the same period of time. Then the ratio of
heights attained by the two ball 1 : 1.
Sol.

73. A projectile of mass 1 kg is projected with a velocity 75. If T is the total time of flight, h is the maximum
height & R is the range for horizontal motion, the x & y
of 20 m/s such that it strikes on the same level as co-ordinates of projectile motion and time t are related
the point of projection at a distance of 3 m. Which as :
of the following options are incorrect.  t t  X  X
(A) the maximum height reached by the projectile can (A) y  4h  1   (B) y  4h  1  
 T  T  R  R
be 0.25 m
(B) the minimum velocity during its motion can be
 T  T  R  R
(C) y  4h  1   (D) y  4h  1  
15 m/s  t t  X  X
3 Sol.
(C) the time taken for the flight can be sec.
5
(D) maximum potential energy during its motion can
be 6J.
Sol.

76. A particle moves in the xy plane with a constant


acceleration ‘g’ in the negative y-direction. Its equation
of motion is y = ax – bx2, where a and b constants.
Which of the following are correct?
(A) The x-component of its velocity is constant.
(B) At the origin, the y-component of its velocity is
g
a
2b
(C) At the origin, its velocity makes an angle tan–1(a)
with the x-axis
(D) The particle moves exactly like a projectile.

Corporate Head Office : Motion Education Pvt. Ltd., 394 - Rajeev Gandhi Nagar, Kota-5 (Raj.)
KINEMATICS Page # 195
Sol. 78. A particle is projected from the ground with velocity
u at angle  with horizontal. The horizontal range,
maximum height and time of flight are R, H and T
respectively. They are given by,

u 2 sin 2 u 2 sin2  2u sin


R= ,H= and T =
g 2g g

Now keeping u as fixed,  is varied from 30° to 60°.


Then,
(A) R will first increase then decrease, H will increase
and T will decrease
(B) R will first increase then decrease while H and T
both will increase
(C) R will decrease while H and T will increase
(D) R will increase while H and T will increase
Sol.
77. A ball is rolled off along the edge of a horizontal
table with velocity 4 m/s. It hits the ground after time
0.4s. Which of the following are correct?
(A) The height of the table is 0.8 m
(B) It hits the ground at an angle of 60° with the
vertical
(C) It covers a horizontal distance 1.6 m from the
table
(D) It hits the ground with vertical velocity 4 m/s
Sol.

: 0744-2209671, 08003899588 | url : www.motioniitjee.com, : info@motioniitjee.com


Page # 196 KINEMATICS

Exercise - III (JEE ADVANCED)

1. The position vector of a particle moving in x-y Sol.



plane is given by r  ( t 2  4)i  ( t  4)j . Find
(a) Equation of trajectory of the particle
Sol.

(b) Time when it crosses x-axis and y-axis 4. The velocity time graph of a body moving in a
Sol. straight line is shown. Find its
y

velocity in m/sec
60°

30°
x
2. A p arti cl e move s al ong the sp ac e curv e 2.5 sec
 time in sec 2
r  ( t 2  t) i  (3 t  2) j  ( 2t 3  4 t 2 ) k . (t in sec, r in m) Find (a) instantaneous velocity at t = 1.5 sec
at time t = 2 the (a) velocity, (b) acceleration, (c) Sol.
speed or magnitude of velocity and (d) magnitude of
acceleration.
Sol.

(b) average acceleration from t = 1.5 sec. to


t = 2.5 sec
Sol.

3. At time t the position vector of a particle of mass


 (c) draw its acceleration time graph from t = 0 to
m = 3kg is given by r  6 t i  t 3 j  cos tk . Find the re-
t = 2.5 sec
 Sol.
sultant force F ( t) , magnitude of its acceleration when


t , & speed when t = .
2

Corporate Head Office : Motion Education Pvt. Ltd., 394 - Rajeev Gandhi Nagar, Kota-5 (Raj.)
KINEMATICS Page # 197

5. The curvilinear motion of a particle is defined by vx 8. A particle goes from A to B with a speed of 40 km/
= 50 – 16t and y = 100 – 4t2 , where vx is in metres h and B to C with a speed of 60 km/h. If AB = 6BC the
ave rage speed i n k m/h betwe en A and C i s
per second, y is in metres and t is in seconds. It is
____________
also known that x = 0 at t = 0. Determine the velocity
(v) and acceleration (a) when the position y = 0 is total dis tan ce travelled
[Hint : Average speed = ]
reached. time taken
Sol. Sol.

6. Velocity of car v is given by v = at – bt2, where a 9. A particle is moving along x-axis. Initially it is located
and b are positive constants & t is time elapsed. Find 5 m left of origin and it is moving away from the origin
value of time for which velocity is maximum & also and slowing down. In this coordinate system, the signs
corresponding value of velocity. of the initial velocity and acceleration, are
Sol. + y
– + v0 a
(0, 0) x


Sol.

7. The force acting on a body moving in a straight line 10. Find the change in velocity of the tip of the minute
is given by F = (3t2 – 4t + 1) Newton where t is in hand (radius = 10 cm) of a clock in 45 minutes.
sec. If mass of the body is 1kg and initially it was at Sol.
rest at origin. Find displacement between time t = 0
and t = 2 sec
Sol.

: 0744-2209671, 08003899588 | url : www.motioniitjee.com, : info@motioniitjee.com


Page # 198 KINEMATICS
11. At a distance L = 400 m from the traffic light 14. A stone is dropped from a height h. Simultaneously
brakes are applied to a locomotive moving at a velocity another stone is thrown up from the ground with such
v= 54 km/hr. Determine the position of the locomotive a velocity that it can reach a height of 4 h. Find the
relative to the traffic light 1 min after the application time when two stones cross each other.
of the breaks if its acceleration is –0.3 m/sec2. Sol.
Sol.

12. A train starts from rest and moves with a constant 15. A bal loon is ascending vertical ly with an
acceleration of 2.0 m/s2 for half a minute. The brakes acceleration of 0.2 m/s2 Two stones are dropped from
are then applied and the train comes to rest in one it at an interval of 2 sec. Find the distance between
minute. Find them 1.5 sec after the second stone is released (use
(a) the total distance moved by the train, g = 9.8 m/s2).
(b) the maximum speed attained by the train and Sol.
(c) the position (s) of the train at half the maximum
speed.
Sol.

16. From the velocity-time plot shown in figure, find


the distance travelled by the particle during the first
40 seconds. Also find the average velocity during this
period.
V
5m/s
13. A car is moving along a straight line. It is taken t(s)
from rest to a velocity of 20 ms–1 by a constant 0 20 40
acceleration of 5ms–2. It maintains a constant velocity –5m/s
of 20 ms–1 for 5 seconds and then is brought to rest
again by a constant acceleration of –2 ms–2. Draw a Sol.
velocity-time graph and find the distance covered by
the car.
Sol.

Corporate Head Office : Motion Education Pvt. Ltd., 394 - Rajeev Gandhi Nagar, Kota-5 (Raj.)
KINEMATICS Page # 199
17. The velocity-time graph of the particle moving 19. A particle is projected in the X-Y plane. 2 sec
along a straight line is shown. The rate of acceleration after projection the velocity of the particle makes an
and deceleration is constant and it is equal to 5 ms–2. angle 45º with the X-axis. 4 sec after projection, it
If the average velocity during the motion is 20ms–1, moves horizontally. Find the velocity of projection (use
then find the value of t. g = 10 ms–2).
Sol.

o t 25 sec
Sol.

20. A particle is projected upwards with a velocity of


100 m/sec at an angle of 60º with the vertical. Find
the time when the particle will move perpendicular to
its initial direction, taking g = 10 m/sec2.
Sol.

18. The fig. shows the v-t graph of a particle moving


in straight line. Find the time when particle returns to
the starting point.
v
20

10

10 20 25 t
Sol. gx2
21. The equation of a projectile is y  3 x  . The
2
angle of projectile is ________ and initial velocity is
_______.
Sol.

: 0744-2209671, 08003899588 | url : www.motioniitjee.com, : info@motioniitjee.com


Page # 200 KINEMATICS
22. A ball is projected at an angle of 30º above with 25. A ball is projected on smooth inclined plane in
the horizontal from the top of a tower and strikes the direction perpendicular to line of greatest slope with
ground in 5 sec at an angle of 45º with the horizontal. velocity of 8m/s. Find it’s speed after 1 sec.
Find the height of the tower and the speed with which
it was projected. [g =10 m/s2]
8 m/s
Sol.
37º

Sol.

23. A rocket is launched at an angle 53º to the


horizontal with an initial speed of 100 ms–1. It moves
along its initial line of motion with an acceleration of 26. Find range of projectile on the inclined plane which
30 ms–2 for 3 seconds. At this time its engine falls & is projected perpendicular to the incline plane with
the rocket proceeds like a free body. Find : velocity 20m/s as shown in figure.
(i) the maximum altitude reached by the rocket u = 20 ms
-1

(ii) total time of flight


(iii) the horizontal range. [sin 53º = 4/5]
Sol.
37º
Sol.

24. A ball is thrown horizontally from a cliff such that


it strikes ground after 5 sec. The line of sight from the 27. The horizontal range of a projectiles is R and the
point of projection to the point of hitting makes an maximum height attained by it is H. A strong wind now
angle of 37º with the horizontal. What is the initial begins to blow in the direction of motion of the
velocity of projection. projectile, giving it a constant horizontal acceleration
= g/2. Under the same conditions of projection, find
37º the horizontal range of the projectile.
Sol.

Sol.

Corporate Head Office : Motion Education Pvt. Ltd., 394 - Rajeev Gandhi Nagar, Kota-5 (Raj.)
KINEMATICS Page # 201

28. A butterfly is flying with velocity 10 i  12j m / s Sol.


and wind is blowing along x axis with velocity u. If
butterfly starts motion from A and after some time
reaches point B, find the value of u.
y
B

A 37°
x
Sol.

31. Two particles are moving along two long straight


lines, in the same plane, with the same speed = 20
cm/s. The angle between the two lines is 60°, and
their intersection point is O. At a certain moment, the
two particles are located at distance 3m and 4m from
O, and are moving towards O. Find the shortest
distance between them subsequently?

Sol.
29. In the figure shown, the two projectiles are fired
simultaneously. What should be the initial speed of
the left side projectile for the two projectile to hit in
mid-air ?

20m/s
60º 45º
\\\\\\\\\\\\\\\\\\\\\\\\\\
10m
Sol.

32. Rain is falling vertically with a speed of 20 ms–1


relative to air. A person is running in the rain with a
velocity of 5 ms–1 and a wind is also blowing with a
speed of 15 ms–1 (both towards east). Find the angle
with the vertical at which the person should hold his
umbrella so that he may not get drenched.
Sol.

30. In the figure shown, the two projectiles are fired


simultaneously. Find the minimum distance between
them during their flight?
20 3 m / s
20 m/s

60° 30°
20 m

: 0744-2209671, 08003899588 | url : www.motioniitjee.com, : info@motioniitjee.com


Page # 202 KINEMATICS
33. A glass wind screen whose inclination with the 35. A man crosses a river in a boat. If he crosses the
vertical can be changed is mounted on a car. The car river in minimum time he takes 10 minutes with a drift
moves horizontally with a speed of 2 m/s. At what 120 m. If he crosses the river taking shortest path,
angle  with the vertical should the wind screen be he takes 12.5 minute, find -
placed so that the rain drops falling vertically (i) width of the river
downwards with velocity 6 m/s strike the wind screen (ii) velocity of the boat with respect to water
perpendicularly? (iii) speed of the current.
Sol. Assume vb/r > vr
Sol.

34. A man with some passengers in his boat, starts


perpendicular to flow of river 200m wide and flowing
with 2m/s. Boat speed in still water is 4m/s. When he
reaches half the width of river the passengers asked
him they want to reach the just opposite end from
where they have started.
(a) Find the direction due to which he must row to
reach the required end.
(b) How many times more total time, it would take to
that if he would have denied the passengers.
Sol.

Corporate Head Office : Motion Education Pvt. Ltd., 394 - Rajeev Gandhi Nagar, Kota-5 (Raj.)
KINEMATICS Page # 203

1. A speeder in an automobile passes a stationary 7. A, B & C are three objects each moving with constant

policeman who is hiding behind a bill board with a velocity. A’s speed is 10 m/sec in a direction PQ . The
motorcycle. After a 2.0 sec delay (reaction time) the
velocity of B relative to A is 6 m/sec at an angle of,
policeman acceleraties to his maximum speed of 150
cos–1(15/24) to PQ. The velocity of C relative to B is
km/hr in 12 sec and catches the speeder 1.5 km beyond 
the billboard. Find the speed of speeder in km/hr. 12 m/sec in a direction QP , then find the magnitude
of the velocity of C.
2. A large number of bullets are fired in all direction
with the same speed v. What is the maximum area on 8. A particle is projected from point P with velocity
ground on which these bullets can spread?
5 2 m/s perpendicular to the surface of a hollow
3. The speed of a particle when it is at its greatest right angle cone whose axis is vertical. It collides at Q
normally. Find the time of the flight of the particle.
height is2 / 5 times of its speed when it is at its half
y
the maximum height. The angle of projection is
_________ and the velocity vector angle at half the
maximum height is _________.

4. A projectile is to be thrown horizontally from the P Q


45° x
top of a wall of height 1.7m. Calculate the initial
velocity of projection if it hits perpendicularly an incline
of angle 37° which starts from the ground at the bottom 9. A glass wind screen whose inclination with the
of the wall. The line of greatest slope of incline lies in vertical can be changed, is mounted on a cart as
the plane of motion of projectile. shown in figure. The cart moves uniformly along the
horizontal path with a speed of 6 m/s. At what maximum
5. Two inclined planes OA and OB having inclination angle  to the vertical can the wind screen be placed
(with horizontal) 30° and 60° respectively, intersect so that the rain drops falling vertically downwards
each other at O as shown in figure. A particle is with velocity 2 m/s, do not enter the cart?
projected from point P with velocity u  10 3 ms –1 along

a direction perpendicular to plane OA. If the particle
strikes plane OB perpendicularly at Q, calculate
A v=6m/s
u B

Q
P
h
30° 60° 10. A hunter is riding an elephant of height 4m moving
O in straight line with uniform speed of 2m/sec. A deer
(a) velocity with which particle strikes the plane OB, running with a speed V in front at a distance of 4 5 m
(b) time of flight, moving perpendicular to the direction of motion of the
(c) vertical height h of P from O, elephant. If hunter can throw his spear with a speed
(d) maximum height from O attained by the particle of 10 m/sec. relative to the elephant, then at what
and
angle  to it’s direction of motion must he throw his
(e) distance PQ
spear horizontally for a successful hit. Find also the
speed ‘V’ of the deer.
6. A particle is thrown horizontally with relative velocity
10 m/s from an inclined plane, which is also moving
with acceleration 10 m/s2 vertically upward. Find the
time after which it lands on the plane (g = 10 m/s2)

2
10 m/s

30°

: 0744-2209671, 08003899588 | url : www.motioniitjee.com, : info@motioniitjee.com


Page # 204 KINEMATICS

Exercise - IV PREVIOUS YEAR QUESTIONS

LEVEL - I JEE MAIN


1. If a body losses half of its velocity on penetrating 3. A ball whose kinetic energy is E, is projected at an
3 cm in a wooden block, then how much will it angle of 45° to the horizontal. The kinetic energy of
penetrate more before coming to rest ? the ball at the highest point of its flight will be :
[AIEEE 2002] [AIEEE 2002]
(A) 1 cm (B) 2cm (C) 3cm (D) 4cm
Sol.
E E
(A) E (B) (C) (D) zero
2 2
Sol.

2. From a building two balls A and B are thrown such 4. Speeds of two identical cars u and 4u at a specific
that A is thrown upwards and B downwards (both instant. The ratio of the respective distances at which
vertically with same velocity). If vA and vB are their the two cars are stopped from that instant is
respective velocities on reaching the ground,then : [AIEEE 2002]
[AIEEE 2002] (A) 1 : 1 (B) 1 : 4
(A) vB  v A (B) v A  vB (C) v A  vB (C) 1 : 8 (D) 1 : 16
Sol.
(D) their velocities depend on their masses
Sol.

5. A lift is moving down with acceleration a. A man in


the lift drops a ball inside the lift. The acceleration of
the ball as observed by the man in the lift and a man
standing stationary on the ground are respectively
[AIEEE 2002]
(A) g, g (B) g - a, g - a
(C) g - a, g (D) a, g

Corporate Head Office : Motion Education Pvt. Ltd., 394 - Rajeev Gandhi Nagar, Kota-5 (Raj.)
KINEMATICS Page # 205
Sol. Sol.

6. The coordinates of a moving particle at any time t 8. A car moving with a speed of 50 km/h, can be
stopped by brakes after at least 6m. If the same car
are given by x  t 3 and y  t 3 .The speed of the
is moving at a speed of 100 km/h, the minimum stopping
particle at time t is given by [AIEEE 2003] distance is [AIEEE 2003]
(A) 12 m (B) 18 m
(A) 3t  2   2 (B) 3t 2  2   2 (C) 24 m (D) 6 m
Sol.
(C) t 2  2   2 (D)  2  2
Sol.

9. A ball is thrown from a point with a speed v0 at an


angle of projection  . From the same point and at
7. A boy playing on the roof of a 10 m high building the same instant, a person starts running with a
throws a ball with a speed of 10 m/s at an angle 30°
with the horizontal. How far from the throwing point v0
constant speed to catch the ball. Will the person
will the ball be at the height of 10 m from the ground 2
? [g = 10 m/s2, sin 30° = 1/2, cos 30° = be able to catch the ball? If yes, what should be the
3 / 2]
angle of projection ? [AIEEE 2004]
[AIEEE 2003]
(A) Yes, 60° (B) Yes, 30°
(A) 5.20 m (B) 4.33 m
(C) No (D) Yes, 45°
(C) 2.60 m (D) 8.66 m

: 0744-2209671, 08003899588 | url : www.motioniitjee.com, : info@motioniitjee.com


Page # 206 KINEMATICS
Sol. Sol.

10.A particle is acted upon by a force of constant 12.A ball is released from the top of a tower of height
magnitude which is always perpendicular to the velocity h metre. It takes T second to reach the ground. What
of the particle. The motion of the particle takes place
in a plane, it follows that [AIEEE 2004]
T
is the position of the ball in s? [AIEEE 2004]
(A) its velocity is constant 3
(B) its acceleration is constant (A) h/9 m from the ground
(C) its kinetic energy is constant (B) 7h/9 m from the ground
(D) it moves in straight line (C) 8h/9 m from the ground
Sol. (D) 17h/18 m from the ground
Sol.

11.An automobile travelling with a speed of 60 km/h, 13.A parachutist after bailing out falls 50 m without
can brake to stop within a distance of 20 m. If the car friction. When parachute opens, it decelerates at 2
is going twice as fast, i.e. 120 km/h, the stopping ms–2. He reaches the ground with a speed of 3 ms–1.
distance will be [AIEEE 2004] At what height, did he bail out ? [AIEEE 2005]
(A) 20 m (A) 91 m
(B) 40 m (B) 182 m
(C) 60m (C) 293 m
(D) 80 m (D) 111 m

Corporate Head Office : Motion Education Pvt. Ltd., 394 - Rajeev Gandhi Nagar, Kota-5 (Raj.)
KINEMATICS Page # 207
Sol. Sol.

14.A particle is moving eastwards with a velocity of 5


ms –1 . In 10 s the vel oci ty changes to 5 ms – 1
northwards. The average acceleration in this time is
16.The relation between time t and distance x is t =
[AIEEE 2005]
ax2 + bx, where a and b are constants. The acceleration
1 is [AIEEE 2005]
(A) ms 2 towards north-east (A) –2abv2 (B) 2bv3
2
(C) –2av3 (D) 2av2
1 2 Sol.
(B) ms towards north
2
(C) zero
1
(D) ms 2 towards north-west
2
Sol.

17.A projectile can have the same range R for two


angles of projection. If t1 and t2 be the times fo flights
in the two cases, then the product of the two times
of flights is proportional to [AIEEE 2005]
15.A car, starting from rest, accelerates at the rate f
1 1
through a distance s, then continues at constant speed (A) R2 (B) (C) (D) R
for time t and then decelerates at the rate f/2 to R2 R
come to rest. If the total distance travelled is 15 s, Sol.
then [AIEEE 2005]
1 2
(A) s  ft
72
1 2
(B) s  ft
36
1 2
(C) s  ft
2
1 2
(D) s  ft
4

: 0744-2209671, 08003899588 | url : www.motioniitjee.com, : info@motioniitjee.com


Page # 208 KINEMATICS
18.A particle located at x = 0 at time t = 0, starts Sol.
moving along the positive x-direction with a velocity
v that varies as v   x . The displacement of the
particle varies with time as [AIEEE 2006]
(A) t2 (B) t (C) t1/2 (D) t3
Sol.

19.A particle is projected at 60° to the horizontal


with a kinectic energy K. The kinetic energy at the 21.A body is at rest at x = 0. At t = 0, it starts
highest point is [AIEEE 2007] moving in the positive x-direction with a constant
K K accleration. At the same instant another body passes
(A) K (B) zero (C) (D) through x = 0 moving in the positive x- direction with
4 2 a constant speed. The position of the first body is
Sol. given by x1 (t) after time t and that of the second
body by x2(t) after the same time interval.Which of
the following graphs correctly describes (x1 – x2) as a
function of time?
[AIEEE 2008]
(x1 –x2) (x1 –x2)

(A) (B)

t t
O O

(x1 –x2)
(x1 –x2)
2
20.The velocity of a particle is v  v0  gt  ft . If its
position is x = 0 at t = 0, then its displacement after
unit time (t = 1) is [AIEEE 2007]
g f (C) (D)
(A) v0  2 g  3 f (B) v0  
2 3 t
O t
g O
(C) v0  g  f (D) v0   f
2

Corporate Head Office : Motion Education Pvt. Ltd., 394 - Rajeev Gandhi Nagar, Kota-5 (Raj.)
KINEMATICS Page # 209
Sol. 
23. A particle has an initial velocity 3iˆ  4 j and an

acceleration of 0.4iˆ  0.3 j . Its after 10 s is
[AIEEE 2009]
(A) 10 unit (B) 7 2 unit
(C) 7 unit (D) 8.5 unit
Sol.

22.Consider a rubber ball freely falling from a height h


= 4.9 m onto a horizontal elastic plate. Assume that
the duration of collision is negligible and the collision
with the plate is totally elastic. Then the velocity as
a function of time the height as function of time will
be [AIEEE 2009]

V V1

V1 24.A particle is moving with velocity v  k(yiˆ  xj),
ˆ
O t where k is a constant. The general equation for its
(A) (B) path is [AIEEE 2010]
(A) y = x2 + constant (B) y2 = x + constant
O t
–V1 (C) xy = constant (D) y2 = x2 + constant
Sol.

V V

V1 V1

O t O t
(C) t1 2t1 3t1 4t1 (D) t1 2t1 3t1 4t1

–V1 –V1

Sol.

25.A water fountain on the ground sprinkles water all


around it. If the speed of water coming out of the
fountain is v, the total area around the fountain that
gets wet is [AIEEE 2011]

v4  v4 v2 v2
(A)  (B) (C)  (D) 
g2 2 g2 g2 g

: 0744-2209671, 08003899588 | url : www.motioniitjee.com, : info@motioniitjee.com


Page # 210 KINEMATICS
Sol. Sol.

28. A particle of mass m is at rest at the origin at


time t = 0. It is subjected t o a force F(t) = F0e-bt in
the x direction. It speed v(t) is depicted by which of
the following curves? [AIEEE 2012]
Fo
Fo
mb
26.An object, moving with a speed of 6.25 m/s, is mb

dv
decelerated at a rate given by  2.5 v where v (A) (B)
dt V(t)

is the instantaneous speed. The time taken by the V(t)

object, to come to rest, would be t t


[AIEEE 2011] Fo
Fo
(A) 2 s (B) 4 s (C) 8 s (D) 1 s mb ( )
mb
Sol.

(C) (D)
V(t)

V(t)
t t
Sol.

27.A boy can throw a stone up to a maximum height


of 10 m. The maximum horizontal distance that the
boy can throw the same stone up to will be
[AIEEE 2012]
(A) 20 2 m (B) 10 m (C) 10 2 m (D) 20 m

Corporate Head Office : Motion Education Pvt. Ltd., 394 - Rajeev Gandhi Nagar, Kota-5 (Raj.)
KINEMATICS Page # 211

LEVEL - II JEE ADVANCED


1. In 1.0 sec. a particle goes from point A to point B 3. An object A is kept fixed at the point x = 3 m and
moving in a semicircle of radius 1.0 m. The magnitude y = 1.25 m on a plank P raised above the ground. At
of average velocity is - [JEE ‘99] time t = 0 the plank starts moving along the +x direction
A with an acceleration 1.5 m/ s2. At the same instant a
stone is projected from the origin with a velocity u as
1m shown. A stationary person on the ground observes
B the stone hitting the object
y
(A) 3.14 m/sec (B) 2.0 m/sec during its downward motion A
(C) 1.0 m/sec (D) zero at an angle of 45° to the 1.25m P
Sol. horizontal. All the motions
are in x-y plane. Find u and u
the time after which the
stone hits the object. Take O 3.0 m x
g = 10 m/s 2
[JEE 2000]
Sol.

2. A ball is dropped vertically from a height d above 4. On a frictionless horizontal surface, assumed to be
the ground it hits the ground and bounces up vertically the x-y plane, a small trolley A is moving along a
to a height d/2. Neglecting subsequent motion and air straight line parallel to the y-axis (see figure) with a
resistances, its velocity v varies with the height h
constant velocity of ( 3 – 1) m/s. At a particular
above the ground as - [JEE’ 2000 (Scr)]
instant, when the line OA makes an angle of 45° with
v v
the x-axis, a ball is thrown along the surface from the
d origin O. Its velocity makes an angle  with the x-axis
h h
(A) (B) d
and it hits the trolley.
y
A
v v

d d
h h
(C) (D)
45°
O x
Sol. (a) The motion of the ball is observed from the frame
of trolley. Calculate the angle  made by the velocity
vector of the ball with the x-axis in this frame.
(b) Find the speed of the ball with respect to the
4
surface, if  = . [JEE 2002]
3
Sol.

: 0744-2209671, 08003899588 | url : www.motioniitjee.com, : info@motioniitjee.com


Page # 212 KINEMATICS
5. A particle starts from rest. Its acceleration (a) Sol.
varsus time (t) is as shown in the figure. The maximum
speed of the particle will be - [JEE’ 2004 (Scr)]
a
2
10m/s

11 t(s)
(A) 110 m/s (B) 55 m/s (C) 550 m/s (D) 660 m/s
Sol.

8. STATEMENT-1
For an observer looking out through the window of a
fast moving train, the nearby objects appear to move
in the opposite direction to the train, while the distant
objects appear to be stationary.
6. A small block slides without friction down an inclined STATEMENT-2
plane starting from rest. Let Sn be the distance If the observer and the object are moving at velocities
 
Sn V1 and V2 respectively with reference to a laboratory
travelled from time t = n – 1 to t = n. The S is - frame, the velocity of the object with respect to the
n1  
[JEE’ 2004 (Scr)] observer is V2 – V1
(A) STATEMENT-1 is True, STATEMENT-2 is True;
2n – 1 2n  1 2n – 1 2n
(A) (B) (C) (D) STATEMENT-2 is a correct explanation for STATEMENT-1
2n 2n – 1 2n  1 2n  1
(B) STATEMENT-1 is True, STATEMENT-2 is True’
Sol. STATEMENT-2 is NOT a correct explanation for
STATEMENT-1
(C) STATEMENT-1 is True, STATEMENT-2 is False
(D) STATEMENT-1 is False, STATEMENT-2 is True
[JEE’ 2008]
Sol.

7. The velocity displacement graph of a particle moving


along a straight line is shown. The most suitable
acceleration-displacement graph will be -
[JEE’ 2005 (Scr)]
v
v0
9. A train is moving along a straight line with a con-
stant acceleration 'a'. A boy standing in the train
throws a ball forward with a speed of 10 m/s, at an
x0 x angle of 60° to the horizontal. The boy has to move
a a forward by 1.15 m inside the train to catch the ball
x back at the initial height. The acceleration of the train
in m/s2 is [JEE’ 2011]
(A) (B) Sol.
x
a a
x x

(C) (D)

Corporate Head Office : Motion Education Pvt. Ltd., 394 - Rajeev Gandhi Nagar, Kota-5 (Raj.)
KINEMATICS Page # 213

ANSWER KEY

QUESTIONS FOR SHORT ANSWER


 
1. Magnitude | a| will remain uncharged. B  a

 
a  B  A  a2  a 2  2a 2 cos(   d)

–a
 2a2 (1  cos )  2a 2 (1  1  2 sin 2d / 2) = 2a sin d /2

2 Speedometer measure speed of car as it only gives the magnitude.


3 When particle is moving with constant velocity its average velocity and instantaneous velocity will be
same and magnitude of instantaneous velocity will also be same.

 S 
4 VAvg  , S  VAvg  t
t

 S  
5 VAvg  with zero displacement non zero VAvg is not x
t S  0

possible zero displacement and non zero V is possible if particle
t
is reversing and coming to starting point. Show on x-t graph by an example.

6 Speed of projectile is smallest at the highest point.


7 Both the ball will hit the ground with same speed.
8 If sack of rice is dropped when it is just above the centre it will fall ahead of circle because sack will
have velocity same as plane in horizontal direction.
9 Ist Curve : at particular time x has more than one value hence not a 1-D motion.

IInd Curve : | V| cannot be negative

IIIrd Curve : Length of a moving body can not decrease with time
10 Ist Curve : A ball moving forward collides with surface rebounds and stops after IInd collision
IInd Curve : A ball repeatedly making inelastic collisions with floor.
IIIrd Curve : Collision of a ball with surface. {Surface has large velocity for short time}

11 (a) is incorrect car can not travel around track with constant velocity as direction is continuously
changing.
(b) correct

12 Ball at maximum height V = 0 for just an instant but acceleration due to gravity.
1
13 Vf  2gH . Let balls meet after t sec. h1
1 2 1 2 H X
h1  gt and h2 = Vf t  gt
2 2 h2
H V0=Vf
h1 + h2 = H = Vf t H  2gH t t  2
2g
1 H H
 h1  g  hence they will meet above half height of building.
2 2g 4

: 0744-2209671, 08003899588 | url : www.motioniitjee.com, : info@motioniitjee.com


Page # 214 KINEMATICS

14 Initially bullet is at rest u = 0 V2 = 0 + 2as

V2
 a muzzle velocity is more for short barrl and S is also less hence acceleration will be more in that case.
2S

15 Hence we can not conclude that velocity of boat is 5 m/sec w.r.t. shore

VBottle  Vriver ; VB – VR = 5
16 Yes wrench will hit at the same place on the deck irrespective of that boat is at rest or moving because
when boat is at rest wrench will have zero horizontally velocity and when boat is moving both will have
same horizontal velocity.
17 Acceleration of the projectile remains constant throughout the journey = g
18 (a) In child point of view range will be same in both the cases.
(b) In ground frame of reference
VCT = VC – VT
VC = VCT + VT
For front range Vcannon = VC cos  + VT Range will be more
For Rear range Vcannon = VC cos  – VT Range will be less

d
19 d t for tmin cos  = 1 maximum Hence A will reach opposite end in least time
Vbr Vbr cos 

Exercise - I OBJECTIVE PROBLEMS (JEE MAIN)


1. B 2. B 3. B 4. B 5. B 6. B 7. A

8. D 9. C 10. C 11. C 12. D 13. A 14. A

15. B 16. C 17. D 18. B 19. A 20 B 21. C

22. C 23. D 24. B 25. A 26. C 27. D 28. D

29. B 30. D 31. D 32. C 33. B 34. C 35. B

36. B 37. D 38. B 39. C 40. B 41. C 42. A

43. D 44. D 45. A 46. C

Exercise - II
1. A 2. D 3. B 4. B 5. D 6. C 7. B

8. B 9. A 10. D 11. C 12. B 13. D 14. C

15. B 16. B 17. D 18. C 19. B 20 C 21. A

22. A 23. A 24. C 25. A 26. D 27. D 28. C

Corporate Head Office : Motion Education Pvt. Ltd., 394 - Rajeev Gandhi Nagar, Kota-5 (Raj.)
KINEMATICS Page # 215

29. B 30. B 31. D 32. A 33. D 34. A 35. B

36. C 37. C 38. B 39. A 40. C 41. C 42. A

43. C 44. C 45. D 46. B 47. C 48. D 49. B

50. C 51. A 52. A 53. A 54. B 55. B 56. C

57. B 58. B 59. A,B,C,D 60. A,C 61. B,C,D 62. B,D 63. C,D

64. A,C 65. A,C 66. A,D 67. C 68. A,B,C,D 69. B,C,D 70. A,C,D

71. A,B,C 72. C,D 73. D,C 74. A,B,C,D 75. A,B 76. A,B,C,D 77. A,C,D

78. B

CExercise - III (JEE ADVANCED)

1. (a) y2 + 8y + 12 = x ; (b) crosses x axis when t = 4 sec, crosses y axis when t = ± 2 sec.

2. (a) 5i + 3j + 8k, (b) 2i + 16k, (c) 7 2 , (d) 2 65 3. –18 tj – 3 cos tk ; 3 ; 3 4   4

1 3  
4. (a) m / s , (b) m / s 2 , (c) 5. v  –30 i – 40 j, a  –16 i – 8 j 6. a/2b, a2/4b
3 2

2 v0 a vel   2
7. m 8. 42 km/hr 9. 10.  3  cm/min 11. 25 m
3 – +  

 h
12. (a) 2.7 km; (b) 60 m/s; (c) 225 m and 2.25 km 13. 240 m 14.   15. 50 m
 8g

16. 100 m, zero 17. 5 s 18. 36.2 sec. 19. 20 5 20. 20 sec 21. 60, 2 m/sec.

22. u = 50 ( 3 – 1) m/sec., H = 125 (– 3 + 2)m 23. (i) 1503.2 m (ii) 35.54 sec (iii) 3970.56 m

24. 100/3 m/s 25. 10 m/s 26. 75 m 27. R + 2H 28. 6 m/s 29. 20 × 2/3

–1  1  4
30. 10 m 31. 50 3 cm 32. tan–1 (1/2) 33. tan–1(3) 34.   tan  2  , 3

35. 200 m, 20 m/min, 12 m/min

: 0744-2209671, 08003899588 | url : www.motioniitjee.com, : info@motioniitjee.com


Page # 216 KINEMATICS

v 4
1. 122.7 km/hr 2.
g2
3. 60°, tan
–1
 3/2  4. u = 3m/s 5. (a) 10 ms–1,(b) 2 sec, (c)5 m, (d)

1
16.25 m, (e) 20 m 6. sec 7. 5 m/sec 8. 1 sec 9. 2 tan–1 (1/3) 10.  = 37°, v = 6 m/s
3

Exercise - IV PREVIOUS YEAR QUESTIONS

LEVEL - I JEE MAIN

1. A 2. B 3. C 4. D 5. C 6. B 7. D
8. C 9. A 10. C 11. D 12. C 13. C 14. D
15. A 16. C 17. D 18. A 19. C 20. B 21. B
22. C 23. B 24. D 25. A 26. A 27. D 28. C

LEVEL - II JEE ADVANCED

1. B 2. A 3. u = 7.29 m/s, t = 1 sec 4. (a) 45°, (b) 2m/sec

5. B 6. C 7. B 8. B 9. 5 m/s2

Corporate Head Office : Motion Education Pvt. Ltd., 394 - Rajeev Gandhi Nagar, Kota-5 (Raj.)

S-ar putea să vă placă și